Far-1 Autumn 2023 - 2

You might also like

You are on page 1of 340

CAF-01

Financial Accounting
and Reporting I

Vol. I

Practice Manual Examination


Questions & Answers

Book Contains Translations of


QURANIC AYATS & AHADITHS,
Therefore, Handle Carefully.
Table OF Contents
Chapter Topics Page No.
No.

1 Disposal and Exchange of PPE 01

2 Revaluation 18

3 Statement of Changes in Equity 59

4 IAS-23 74

5 IAS-8 120

6 Ratios 161

7 IAS-20 224

8 IAS-40 255

9 All Test Including Spring 2020 282


Property, Plant & Equipment [PPE] (IAS 16)
These are those tangible items that:
• Are held for use in business for production or supply of goods or for administrative purposes; and
• Are expected to be used during more than one accounting period.
 The asset are capitalized and then depreciated over their useful life.
 Examples of these assets include:
• Land & Building
• Plant & Machinery
• Furniture & Fixtures
• Motor Vehicles
• Computer Equipment etc

Depreciation is the systematic allocation of depreciable amount of an asset over its useful life.
Depreciable Amount = Cost –Residual Value
Cost means purchase price and all costs incurred in bringing the asset into working condition as intended by
management.
Residual Value is the amount expected to be obtained from an asset at the end of its useful life.
Useful Life is the period over which asset is expected to be used by an entity.
Methods of depreciation:
➢ Straight line method:
Annual Depreciation = Cost – Residual Value
Estimated useful life
or
Rate of depreciation= 1 x 100
Useful life
In this method, depreciation will be same for every year.

➢ Reducing Balance method (Diminishing Balance Method):


In this method, a rate of depreciation is calculated as follows:

s
[1- n ] x 100
c
n = Useful Life
s = Scrap Value/ Residual Value
c = Cost of asset
• The rate will then be multiplied in first year on the cost of asset ( without deducting residual value).
• From second accounting period onwards, rate of depreciation is multiplied with WDV at the beginning of
the year, to calculate depreciation for the year.
• If however, a rate of depreciation is given then simply use that rate.

1
➢ Sum of year’s digit method:
Depreciation expense is calculated as follows:
Sum of digits = n(n+1)
2
n = Useful life
Depreciation =Factor of current year x [cost – R.V]
Sum of digits
➢ Machine Hour method:

Machine Hour Rate = cost – R.V .


Effective total working Hours

Depreciation for the year = Rate x Actual hours used during the period

➢ Production Units Method:

Cost – R.V
Production unit rates =
𝑇𝑜𝑡𝑎𝑙 𝑈𝑛𝑖𝑡𝑠 𝑡𝑜 𝑏𝑒 𝑝𝑟𝑜𝑑𝑢𝑐𝑒𝑑

Depreciation for the year = Rate x Actual units produced during the period

2
DISPOSAL / SALE OF ASSET
Machine
Cost = 750,000
Accumulated Depreciation = 600,000
Written down value = 150,000
Suppose this machine is now sold for 180,000.
Gain / Loss = different between WDV and Sale proceeds.
= 180,000 – 150,000
Gain = 30,000

ACCOUNTING ENTRIES OF DISPOSAL


(a) Calculating Gain / Loss by Entries
If there is no separate ledger accumulated If there is a separate ledger of accumulated
depreciation depreciation
Cash 130,000 Cash 180,000
Machin (At WDV) 150,000 Account Depreciation 600,000
Gain (Bal.) 30,000 Machine (At Cost) 750,000
Gain (Bal.) 30,000
Difference in this entry is gain or loss to be recognised Difference in this entry is gain or loss to be recognized
in Income Statement in Income Statement

(b) Calculating Gain / Loss by Disposal Account


Disposal account is a ledger which is used to calculate the gain or loss on disposal of an asset.
Disposal Account (if asset account is at WDV)
Machine 150,000 Cash 180,000

Gain (P.L) 30,000


Disposal Account(if asset account is at cost)
Machine 750,000 Cash 180,000
Accumulated Depreciation 600,000
Gain (P.L) 30,000

SUMMARY OF ACCOUNTING ENTRIES IF A DISPOSAL ACCOUNT IS REQUIRED TO BE PREPARED


No separate ledger of accumulated depreciation Separate ledger of accumulated depreciation
Cash XX Cash XX
Disposal A/c XX Disposal A/c XX
Disposal A/c XX Disposal A/c XX
Machine (At WDV) XX Machine (At Cost) XX
Acc Depreciation XX
Disposal A/c XX

3
Q. 1 The Machinery Account (at Cost) of a firm for the three years ended December 31, 2014 appears as follows
Machinery
2012 2012
Jan 1 to Cash (No-1) 50,000 Dec 31 Balance c/d 50,000
50,000 50,000

2013 2013
Jan 1 Balance b/d 50,000
July 1 to Cash (No-2) 20,000 Dec 31 Balance c/d 70,000
70,000 70,000

2014 2014
Jan 1 Balance b/d 70,000
July 1 to Cash (No-3) 15,000 Dec 31 Balance c/d 85,000
85,000 85,000

Depreciation @20% on the diminishing value basis was accumulated in provision for depreciation Account
On October 1, 2015 machines # 2 was damaged and has to be replaced by a new machine costing Rs. 25,000 . The
Machine was insured and insurance claim of Rs. 12,400 was received.
Required:
Prepare the 2015 Machinery Account, Provision for Depreciation Account and machinery disposal Account. All
workings are to be shown. Depreciation is provided on time basis.
Note: Provision for depreciation account is another name of accumulated depreciation account.

Q.2 A Company charges depreciation on Plant and Machinery under reducing balance method at the rate of 15%
p.a. On 1st April 1997, the balance in the ledger stood as 460,000 (Carrying amount). The following particulars are
given relating to the plant and machinery during the four years ending 31 March 2001.
Date
1-Sep-1997 A Machine purchased for Rs. 21,000 on 1 May 1995 was fully destroyed in an accident
1-Jul-1998 A new Machine was purchased costing 52,500. A sum of Rs .30,000 was paid on the same date
and remaining balance was paid in May 1999
31-Aug-1999 Plant Purchased on 1st April 1996 for Rs. 31,500 was disposed of for Rs. 36,000
1-Nov-2000 Some old Machine having book value of Rs 10,000 on 1st April1997 was sold for Rs. 4,000
Required:
Prepare plant and machinery account for four years ending 31 March 2001.

Q. 3 A trading organisation charges depreciation on its plant and machinery on a reducing balance method @ 15%
per annum. On 1 July 2011, the net book value in the ledger stood at Rs. 5,660,000. Movements in the plant and
machinery account during the two years ended 30 June 2013 were as follows:
Date Particulars
1 October 2011
A new machine costing Rs. 80,000 was purchased. A sum of Rs. 30,000 was paid on
the same date and the balance was paid on 31 March 2012.
1 December 2011 A machine that was purchased for Rs. 200,000 and installed at a cost ofRs. 10,000
on 1 August 2009 was fully destroyed in an accident.
1 February 2012
Some old machinery (book value on 1 July 2011 Rs. 20,000) was sold for Rs. 8,000.

4
30 November 2012
A machine imported on 1 July 2010 was disposed of for Rs. 63,000. The value of
machine was Rs. 70,000 whereas import levies amounted to Rs. 5,000.
Required:
Prepare the plant and machinery account for the years ended 30 June 2012 and 2013. (19)

Q.4 The following balances are appearing in the books of Faizan Ahmad & Co. as at 31 st December 2015
Rs.
Machinery – Cost 12,500,000
Machinery -Acc. Depreciation 2,505,000

Following additional information related to the year ended 31st December 2016 is also available:
(i) On 1st July 2016, second hand machinery was purchased for Rs. 900,000. An amount of Rs. 100,000
was spent on its overhauling before use. Overhauling was necessary to bring it to working condition.
(ii) On 1st August 2016, the machinery purchased on 1st June 2013 for Rs. 225,000 was destroyed by fire
and claim of Rs. 65,000 was received in cheque from JJ Insurance Company.
(iii) Machinery purchased on 1st January 2014 for Rs. 435,000 was sold on 30th November 2016 at Rs.
140,000 in cash.
(iv) Faizan Ahmad & Co. provides depreciation on machinery @ 10% on written down value.
(v) Depreciation on addition/deletion is provided in proportion to the period of use.
Required:
Prepare the following ledger for the year ended December 31st, 2016
(a) Machinery Account
(b) Accumulated Depreciation Account
(c) Disposal Account (12)
Summary of entries of disposal (if no disposal account is required to be prepared)

Q. 5 The written down value of plant and machinery of Azfar and Company as at 1 july 2011 is Rs. 2,110,443
Following additional information is also available:
i. On 1 July 2009, second-hand machinery was purchased for Rs. 600,000. An amount of Rs. 400,000 was
spent on its overhauling, before use.
ii. On 1 January 2010 machinery costing Rs. 500,000 was purchased.
iii. The machinery purchased on 1 July 2009 became obsolete and was sold for Rs. 200,000 on 1 January
2012. On the same date, new machinery was purchased at a cost of Rs. 1,200,000.
iv. Machinery purchased on 1 January 2010 was sold on 30 June 2013 at its book value plus Rs. 100,000.
Azfar and Company provides depreciation on machinery @ 15% on written down value.
Depreciation on addition / deletion is provided in proportion to the period of use.
Required:
a) Machinery Account from 1 July 2011 to 30 June 2013
b) Machinery Disposal Account for the years ended 30 June 2012 and 2013

5
Q. 6 Ziakot steel Works provides depreciation on plant and machinery at 20% per annum on diminishing balance
method.
On 1 July 2012 the balances in the plant and machinery account and accumulated depreciation accounts were Rs.
712,000 and Rs. 240,000 respectively.

A machine which was purchased on 1 January 2010 for Rs. 100,000 was sold on 31 March 2013 at Rs. 50,000.
Only one new machine was purchased during the year ended june 30 2013 costing Rs. 60,000. The machine was
received in the factory on October 1, 2012 and was installed on 1 January 2013.
Required:
Plant and machinery account and accumulated depreciation account for the year ended 30.06.2013.

Q.7 Mr. umer has provided you the following data


Furniture – WDV as on 01.01.2008 400,000
Following transactions took place during the year.
1) Additions made Rs. 30,000 on 1.6.2008.
2) An asset having cost of Rs.50,000 on 01.04.2006 is sold on 30.6.2008 for Rs.12,000
3) Rate of depreciation is 20% WDV.
Required:
1) Asset account for the year ended December 31,2008 and disposal account.
2) Also prepare asset a/c for year ended December 31, 2009 assuming no additions and deletion took place

6
A. 1
Machinery Account – At Cost
b/d 85,000 1-10 Disposal Account 20,000
1-10 Cash 25,000
c/d 90,000
Accumulated Depreciation Account
Disposal A/c (W-3) 7,760 b/d (W-1) 31,500
Depreciation (W-2) 11,230
c/d 34,970

*Disposal Account
Machinery 20,000 Cash 12,400
Acc Depreciation 7,760
Gain (P&L) 160
*It is prepared to find out the gain or loss on disposal of an asset. It is prepared for the year; therefore there is no
concept of opening and closing balance in this ledger (an income statement item).
WORKINGS:
(W-1) Accumulated Depreciation as on 1st January, 2015:
Machine No. Cost 2012 2013 2014 Total
1 (1-1-2012) 50,000 10,000 8,000 6,400 24,400
2 (1-1-2013) 20,000 -- 2,000 3,600 5,600
3 (1-1-2014) 15,000 -- -- 1,500 1,500
10,000 10,000 11,500 31,500

(W-2) Depreciation for the year 2015:


Machine # Calculations Depreciation
1 (50,000 – 24,400) × 20% 5,120
2 (20,000 – 5,600) × 20% × 9/12 2,160
3 (15,000 – 1,500) × 20% 2,700
4 25,000 × 20% × 3/12 1,250
11,230
(W-3) Accounting Depreciation of Asset Disposed Off:
5,600 + 2,160 = 7,760

A.2
Plant & Machinery Account (At WDV)
Date Particular Rs. Date Particular Rs.
1-4-97 b/d 460,000 1-9-97 Disposal a/c (W-1) 14,433
31-3-98 Depreciation (W-2) 67,653
c/d 377,914
1-4-98 b/d 377,914 31-3-99 Depreciation (W-3) 62,593
1-7-98 Cash 30,000
Payable 22,500
52,500 c/d 367,821
1-4-99 b/d 367,821 31-8-99 Disposal A/c (W-4) 18,136
31-3-2000 Depreciation A/c (W-5) 53,480
c/d 296,205
1-4-2000 b/d 296,205 1-11-2000 Disposal (W-6) 5,604
31-3-2001 Depreciation (W-7) 44,047
c/d 246,554

7
WORKINGS:
(W-1) WDV of Asset Destroyed:
Period Depreciation WDV
1-5-95 – 31-3-96 21,000 × 15% × 11/12 2,888 18,112
1-4-96 – 31-3-97 18,112 × 15% 2,717 15,395
1-4-97 – 1-9-97 15,395 × 15% × 5/12 962 14,433
(W-2) Depreciation for the year ended 31-3-98:
(460,000 – 15,395) × 15% = 66,691
Add: Depreciation of destroyed machine = 962
67,653
(W-3) Depreciation for the year ended 31-3-99:
377,914 x 15% = 56,687
52,500 × 15% × 9/12 = 5,966
62,593
(W-4) Disposal of Machine on 31-8-1999:
Purchase on 1-4-96 for Rs. 31,500
Period Depreciation WDV
1-4-96 – 31-3-97 31,500 × 15% 4,725 26,775
1-4-97 – 31-3-98 26,775 × 15% 4,016 22,759
1-4-98 – 31-3-99 22,759 × 15% 3,414 19,345
1-4-99 – 31-8-99 19,345 × 15% × 5/12 1,209 18,136
(W-5) Depreciation for the year ended 31-3-2000:
(367,821 – 19,345) × 15% = 52,271
Depreciation on disposed asset 1,209
53,480
(W-6) Disposal of Machine on 1-11-2000:
Book Value on 1-4-97 = Rs. 10,000
Period Depreciation WDV
1-4-97 – 31-3-98 10,000 × 15% 1,500 8,500
1-4-98 – 31-3-99 8,500 × 15% 1,275 7,225
1-4-99 – 31-3-2000 7,225 × 15% 1,084 6,141
1-4-2000 – 1-11-2000 6,141 x 15 % x 7 /12 537 5,604

(W-7) Depreciation for the year ended 31-3-2001:


(296,205 – 6,141) × 15% = 43,510
Add: Depreciation on disposal 537
44,047
A.3
Plant & Machinery A/c
Rs. Rs.
1-7-11 Balance b/d 5,660,000 1-12-11 Disposal account 144,334
1-10-11 Bank/creditors 80,000 01-02-12 18,250
Disposal account

30-6-12 Dep. Expense 843,279


30-6-12 Balance c/d 4,734,137
5,740,000 5,740,000
1-7-12 Balance b/d 4,734,137 30-11-12 Disposal account 50,800

30-6-13 Dep. Expense 705,380

8
30-6-13 Balance c/d 3,977,957
4,734,137 4,734,137

Workings:
Loss due to Accident: Rs.
Cost of purchases 210,000
Depreciation for 11 months in 2009-2010 (28,875)
Book value at 30-06-2010 181,125
Depreciation for the year 2010-11 (27,169)
Book value at 30-06-2011 153,956
Depreciation for five months up to 1-12-2011 (153,956 x 15% x 5/12 (9,622)
WDV 144,334
Disposal of old plant and machinery:
WDV as on 1-7-2011 20,000
Depreciation for seven months up to 1-02-2012 (1,750)
WDV 18,250

Depreciation for the year ended June 2012


Depreciation on addition (80,000 x 15% x 9/12) 9,000
Depreciation on deletion due to accident 9,622
Depreciation on disposal of old plant and machinery 1,750

Depreciation on opening assets still available [5,660,000 - 153,956 - 20,000] x 15% 822,907
Total depreciation
843,279

Disposal of imported plant and machinery:


Cost of purchases 75,000
Depreciation for the year 2010-11 (11,250)
Book value at 30-06-2011 63,750
Depreciation for the year 2011-12 (9,563)
Book value at 30-06-2012 54,187
Depreciation for five months up to 30-11-2012 (3,387)
WDV 50,800

Depreciation for the year ended June 2013


Depreciation on disposal of imported plant and machinery 3,387

Depreciation on opening assets still available [4,734,137 - 54,187] x 15% 701,993


Total depreciation
705,380

9
A. 4
Machinery – Cost
b/f 12,500,000 1/8/2016 Disposal 225,000
30/11/2016 Disposal 435,000
1/7/2016 Bank 1,000,000
(900+100) c/f 12,840,000

13,500,000 13,500,000

Acc. Depreciation – Machinery

1/8/2016 Disposal 63,392 b/f 2,505,000


30/11/2016 Disposal 114,949 31/12/2016 Dep. 1,039,413

c/f 3,366,072
3,544,413 3,544,413
Disposal
1/8/2016 Machinery – Cost 225,000 1/8/2016 Acc. Dep 63,392
30/11/2016 Machinery –Cost435,000 30/11/2016 Acc. Dep 114,949
1/8/2016 Bank 65,000
30/11/2016 Cash 140,000
Loss(P&L) 276,659
660,000 660,000
Workings:
W-1) Depreciation Expense for the year:
On B/F 947,103
(12,500,000 – 2,505,000 – 171,619 – 352,350) x 10%

On Additions: 50,000
(1,000,000/10 x 100 x 6/12)

On Disposals:
(225,000 – 53,381) x 10/100 x 7/12 10,011
(435,000 – 82,650) x 10/100 x 11/12 32,299
1,039,413
W-2)
(i) Machine – 1:
WDV
1 – 6 – 2013 225,000 x 10% x 7/12 = 13,125 = 211,875
1 – 6 – 2014 211,875 x 10% = 21,187 = 190,687
1 – 6 – 2015 190,687 x 10% = 19,069 = 171,618
1 – 8 – 2016 171,618 x 10% x 7/12 = 10,011 = 161,607

10
63,392

Machine – 2:

1 – 1 – 2014 435,000 x 10% = 43,500 = 391,500


1 – 1 – 2015 391,500 x 10% = 39,150 = 352,350
1 – 1 – 2016 352,350 x 10% x 11/12 = 32,299 = 320,051
114,949

A. 5
Machine WDV
b/d (bal) 2,110,443 Disposal 668,313
Cash 1,200,000 Depreciation 352,379
c/d 2,289,751
3,310,443 3,310,443

b/d (bal) 2,289,751 Disposal 284,033


Depreciation 343,462
c/d 1,662,256
2,289,751 2,289,751

Disposal account
Machine 668,313 Cash 200,000
Loss (P&L) 468,313
668,313 668,313

Disposal account
Machine 284,033 Cash (284,033 + 100,000) 384,033
Gain (P&L) 100,000
384,033 384,033

W-1) Machine Purchased on 01-07-2009:


Cost = purchases price 600,000
Overhauling cost 400,000
Total cost 1,000,000
Depreciation @15% (150,000)
30-06-2010 850,000
Depreciation @ 15% (127,500)
30-06-2011 722,500
Depreciation @ 15% x 6/12 (54,188)
01-01-2012 668,313

W-2) Machine Purchased on 01-01-2010:


Cost 500,000

11
Depreciation @ 15% x 6/12 (37,500)
30-06-2010 462,500
Depreciation @ 15% (69,375)
30-06-2011 393,125
Depreciation @ 15% (58,969)
30-06-2012 334,156
Depreciation @ 15% (50,123)
30-06-2013 284,033
W-3) Depreciation for 2012:
(2,110,443-722500) x 15% = 208,191
722,500 x 15% x 6/12 = 90,000
1,200,000 x 15% x 6/12 = 54,188
352,379
W-4) Depreciation for 2013:
(2,289,751 – 334,156) x 15% = 293,339
334,156 x 15% = 50,123
343,462
A. 6
Plant and machinery
b/d 712,000 Disposal 100,000
Cash 60,000
c/d 692,000
772,000 772,000

Accumulated depreciation account


b/d 240,000
Disposal (W 1) 51,040 Depreciation (W-2) 97,520
c/d 286,480
337,520 337,520

W-1 Accumulate depreciation of machine sold:


Cost (01.01.2010) 100,000
Depreciation @ 20% x 6/12 (10,000)
30 June 2010 90,000
Depreciation @ 20% (18,000)
30 June 2011 72,000
Depreciation @ 20% (14,400)
30 June 2012 57,600
Depreciation @ 20% x 9/12 (8,640)
31 March 2013 48,960
Total depreciation upto disposal (10,000 + 18,000 +14,400+8,640) 51,040
W-2 depreciation for the year:

Opening WDV (712,000 - 240,000) 472,000


Less opening WDV of Disposal (57,600)

12
414,400
X 20% 82,880
+57,500 x 20% x 9/12 8,640
+600,000x20% x 6/12 6,000
Total 97,520

A.7
Building A/C-2008(WDV)
b/d 400,000 Disposal (W-2) 30,600
Cash 30,000 Depreciation (W-1) 80,100
C/d 319,300
430,000 430,000

Disposal A/C
Building 30,600 Cash 12,000
Loss (P&L) 18,600
30,600 30,600

Building A/C-2009(WDV)
b/d 319,300 Deprecation (319,300 x 20%) 63,860
c/d 255,440
319,300 319,300

(W-1) Depreciation expenses


On opening excluding disposal 73,200
(400,000 - 34,000) * 20%
On addition (30000 * 20% * 7/12) 3,500
On disposal (34,000 * 20% * 6/12) 3,400
80,100

(W-2) WDV of disposal


Cost (1-04-06) 50,000
Less: deprecation (50000*20%*9/12) (7,500)
31-12-06 42,500
Depreciation (42500*20%) (8,500)
31-12-07 34,000
Depreciation (34000*20%*6/12) (3,400)
30-06-08 30,600

13
Exchange of assets and reverse working of figures:
Q.1 The following information is available in respect of machines of akaml brothers ;
I. The balance of cost and accumulated depreciation of machine as on 1 January 2017 were Rs. 800,000 and
Rs. 333,000 respectively
II. A machine acquired on 1 January 2014 having net book value of Rs. 31,935 on 1 January 2017 was sold
for Rs. 34,000 on 30 April 2017. Cost of disposal incurred was Rs. 5,000.
III. On 1 July 2017, a machine was exchanged for a new machine. The balance of the purchase price was paid
through a cheque of Rs. 80,000. The list price of the new machine was Rs. 130,000. The old Machine had
been acquired at a cost of Rs. 65,000 on 1 October 2015.
IV. Machine are depreciated at 15% per annum using the reducing balance method.

Required:
Prepare the following ledger accounts pertaining to the Machines for the year ended 31 December 2017 :
a) Cost (03)
b) Accumulated depreciation (05)
c) Gain/ Loss on disposal (04)

14
Test question
Q.1 Following information pertains to plant and machinery of Alpha Enterprises (AE):

(i) As at 1 January 2018, balances of cost and accumulated depreciation amounted


to Rs. 12,700,000 and Rs. 6,240,000 respectively.

(ii) On 1 April 2018, an old machine was exchanged for a new machine. The balance
of the purchase price was paid through a cheque of Rs. 680,000. The list price of
the new machine was Rs.1,020,000. The old machine had been acquired for Rs.
870,000 on 1 September 2015.

(iii) On 1 February 2018, a plant having a list price of Rs. 11,300,000 was acquired.
(iv) On 31 October 2018, another machine was sold for Rs. 334,000. It was acquired on
1 January 2015 and had a net book value of Rs. 512,000 on 1 January 2018. A cost of
Rs. 25,000 was incurred on its disposal.
(iv) AE depreciates plant and machinery at 20% per annum using the reducing balance
method

Required:
Prepare following ledger accounts pertaining to the plant and machinery for the year ended 31
December 2018:
(a) Cost (06)
(b) Accumulated depreciation (06)
(c) Assets disposal (04)

15
A.1 Alpha Enterprises
Plant and machinery -
Cost
Date Description Rs. in '000 Date Description Rs. in '000
1-Jan-2018 Balance 12,700 1-Apr-2018 Assets disposal 870
1-Apr-2018 Bank 680 31-Oct-2018 Assets disposal (W-2) 1,000
Disposal 340
1,020
1-Aug-2018 Cash 11,300 31-Dec-2018 Balance 23,150
25,020 25,020

Accumulated depreciation - Plant and


machinery
1-Apr-2018 Assets disposal (W-1) 376 1-Jan-2018 Balance 6,240
31-Oct-2018 Assets disposal (W-2) 573 31-Dec- Depreciation exp. (W-3) 2,292
2018
31-Dec-2018 Balance 7,583
8,532 8,532

Assets disposal - Plant and


machinery
1-Apr-2018 Old machine 870 1-Apr-2018 Acc. depreciation (W-1) 376
31-Oct-2018 Cost 1,000 1-Apr-2018 New machine 340
31-Oct-2018 Acc. depreciation (W-2) 573
31-Oct-2018 Cash (334-25) 309
31-Dec-2018 Loss on disposal (P.L) 272
1,870 1,870

W-1: Accumulated depreciation – Machine exchange Rs. in '000


Depreciation for 2015 870×0.2×4÷12 58
Depreciation for 2016 (870–58)×0.2 162
Depreciation for 2017 (870–58–162)×0.2 130
Accumulated depreciation upto 31-12-2017 (WDV = 870-350=520) 350
Depreciation for 2018 (870–350)×0.2×3÷12 26
376

16
W-2:Accumulated depreciation – Machine sold
1-1-2015 100
x 20% (20)
31-12-2015 80
x 20% (16)
31-12-2016 64
x 20% (12.8)
31-12-2017 51.2
512,000/51.2 x 100
Cost = 1,000,000
Acc.dep on 31-12-2017=1,000,000-512,000=488,000
+ 512,000 x 20% x 10/12=85,333
Acc.dep on 10-10-2018=573,333

W-3 : Depreciation for the year


Opening WDV (12,700 – 6,240) 6,460
Less:Opening WDV of Disposal ( 512 + 520) (1,032)
5,428
x 20% 1086
+depreciation on Disposals (26+85) 111
+depreciation on Additions
1,020 x 20% x 9/12 153
11,300 x 20% x 5/12 942
Total depreciation 2,292

17
Revaluation of property, plant and equipment:
Before Revaluation
Discussion of Retained earnings
Suppose: First year of Business of a sole proprietor:
Capital Account
b/d --
Drawings 80,000 Cash 500,000
Profit 150,000
c/d 570,000

Second year of Business


b/d 570,000
Drawings 130,000 Profit 125,000
c/d 565,000

COMPANY
(First year of business)
Capital Account
1st year
b/d --
Cash 500,000
c/d 500,000

Retained Earnings Account


b/d --
Dividend 80,000 Profit 150,000
c/d 70,000

Dividend = Drawings by Owner of Company


Statement of financial position
At the end of 1st year
Non current assets XX
Current assets XX
XX
Capital & Liabilities:
Capital 500,000
Retained Earnings 70,000
570,000

Page 1 of 36

18
Second year of business
Capital Account
b/d 500,000
c/d 500,000

Retained Earnings Account


b/d 70,000
Dividend 130,000 Profit 125,000
c/d 65,000

Revaluation Model

Subsequent Measurement: (Measurement after initial recognition)


Cost model Revaluation Model
Cost - Revalued Amount -
Less Acc Depreciation (-) Less Acc Depreciation (-)
Carrying Amount __-__ Carrying Amount __-__

Revaluation Model
If revaluation model is adopted, carrying amount is compared with fair value at the date of revaluation.
For example:
1. Revaluation Date is 30-6-2010
Machine
Cost 500,000
Less Acc depreciation (150,000)
Carrying Amount 350,000
Fair value 600,000
There is an unrealized gain or expected gain of Rs 250,000 which is called as revaluation surplus.
Machine 250,000
Revaluation surplus 250,000
(This entry has no effect on profit or loss for the year)
Revaluation surplus is presented within equity in statement of financial position.

2) Revaluation date is 30-6-2015

Building
Cost 1,500,000
Less Acc depreciation (400,000)
Carrying Amount 1,100,000
Fair value 150,000

There is an expected loss of Rs 950,000 which is termed as revaluation loss.


Revaluation loss 950,000
Building 950,000
Revaluation loss is recognized in statement of profit or loss in the period in which it arises.
Conclusion: As a result of revaluation there could be a revaluation surplus or revaluation loss. After revaluation, value of asset is equal
to its fair value. After revaluation, fair value is depreciated over the asset’s remaining useful life.

Page 2 of 36

19
Example
1st revaluation as on 30-6-2011; Revaluation surplus = Rs 300,000
Machine 300,000
Revaluation surplus 300,000
2nd revaluation as on 30-6-2013
(i) Ist scenario; Revaluation loss = Rs 200,000
(ii) 2nd scenario; Revaluation loss = Rs 350,000
Important point to remember
Revaluation is not compulsory every year as per IAS-16.Revaluation entries are only required whenever there is a material difference
between carrying amount and fair value.
If there is a loss on revaluation of an asset in an accounting period but there is a balance of any revaluation surplus related to same
asset because of any previous revaluation then first adjust the loss against the surplus and the balance of loss (if any) is recognized in
statement of profit or loss.
Keeping in view the above point; adjusting entries will be;
Scenario i
Revaluation Surplus 200,000
Machinery 200,000
(No effect of this entry on statement of profit or loss)

Scenario ii
Revaluation Surplus 300,000
Revaluation Loss (bal) 50,000
Machinery 350,000
(Rs 50,000 will be recognized in statement of profit or loss)

Treatment of Acc Depreciation at the time of revaluation:


Machinery as on 30-6-2013
Cost 500,000
Less Acc depreciation (150,000)
Carrying Amount 350,000
Fair value 250,000
Revaluation loss 100,000
The adjustment of accumulated depreciation is only required if the asset account and accumulated depreciation accounts are
separately prepared. In such a situation, asset is revalued in two steps:
a) Eliminate the existing accumulated depreciation against the amount stated against the asset.
Acc Depreciation 150,000
Machinery 150,000
(The effect of this adjustment is that asset account is converted into WDV)
b) Restate the balance of WDV upto fair value.
Revaluation loss 100,000
Machinery 100,000
Entry (a) always remains same while entry (b) can be different depending on whether there is surplus or loss.

Another Example
Cost of Equipment 1,500,000
Less Acc depreciation (700,000)
Carrying Amount 800,000
Fair value 1,100,000
Revaluation surplus 300,000
Prepare accounting entries related to revaluation:

Page 3 of 36

20
a)
Acc Depreciation 700,000
Equipment 700,000
b)
Equipment 300,000
Revaluation surplus 300,000

→ Before any further discussion please recall the calculation of retained earnings:
Opening balance of retained earnings -
Profit for the period -
Less: Dividend for the period (-)
Closing balance of retained earnings -

When the revaluation surplus is realized; (i.e transferred to retained earnings)


There are two methods:
a) Transfer the surplus (in full) when the asset is or b) Transfer the surplus as the asset is used and
disposed off depreciated (annually).

1) Transfer the surplus (in full) when the asset is disposed off:
Example
A machinery having a revalued amount of Rs 260,000 and accumulated depreciation amounting to Rs 13,000 is sold for Rs
253,000 on 30-6-2013. On this date, balance of revaluation surplus is 75,000.
Required:
Prepare necessary journal entries to account for the disposal.
Solution
a)
Cash 253,000
Acc Depreciation 13,000
Machinery 260,000
Gain (bal) 6,000
(Rs 6,000 is treated as an other income in statement of profit or
loss)

b)

Revaluation Surplus 75,000


Retained Earnings 75,000
(Transfer of whole surplus to retained earnings on disposal but not through statement of profit or loss)

Movement of retained earnings and presentation of transfer of revaluation surplus:


Opening balance of retained earnings -
Profit after tax -
Transfer of surplus -
Dividend for the period (-)
Closing balance of retained earnings -

Another Example:
An equipment has following values at the time of disposal:
Revalued Amount 600,000
Acc Depreciation (150,000)
450,000
Revaluation Surplus 300,000
It is sold for Rs 325,000

Page 4 of 36

21
a)
Cash 325,000
Acc Depreciation 150,000
Loss 125,000
Equipment 600,000
b)
Revaluation Surplus 300,000
Retained Earnings 300,000

2) Transfer the surplus annually:


In this case surplus is transferred to retained earnings as the asset is depreciated annually. The basis of calculation of amount of
transfer should be exactly similar to the basis of calculation of depreciation. The entry of transfer remains same as discussed
earlier; i.e
Revaluation Surplus Xxx
Retained Earnings xxx

If the question is silent regarding the policy of transfer of surplus then transfer the surplus as the asset is used and depreciated (to
adjusted the effect of extra depreciation annually; otherwise if the policy of transfer of surplus is on disposal then that effect will be
adjusted on disposal).

Revaluation model:
1) This model involves revaluing the asset’s carrying amount to its fair value (FV) (Also known as revalued amount).
2) If FV is more than carrying amount then revaluation surplus
3) If FV is less than carrying amount revaluation loss
4) If there is revaluation surplus already in existence for an asset because of a previous revaluation, then subsequent
revaluation loss is adjusted against surplus. If loss is more than surplus, difference is recorded in income statement.
5) Depreciation should be charged on cost of asset. If there is a surplus then effect of extra depreciation is transferred to
retained earnings. If there is a revaluation loss then effect of less charged depreciation should be charged when there is a
subsequent surplus in future.
6) After revaluation, revalued amount (FV) is depreciated over remaining useful life.
7) When revaluation surplus is realized (means transferred to retained earnings)
a) At the time of disposal/end of useful life; or
b) As the asset is used by the entity and depreciated (period wise)
If the question is silent then follow the (b) policy.
8) Treatment of accumulated depreciation at the time of revaluation:
a) Eliminate against cost (net replacement value method)
b) Restate up to revalued amount
9) If an asset is revalued, then all the assets in the class of asset need to be revalued. The following are examples of classes of
property, plant and equipment.
(i) Land
(ii) Building
(iii) Plant and Machinery
(iv) Motor Vehicles
(v) Furniture & Fixtures
(vi) Office Equipment.
10) Revaluation is not compulsory annually for items of property, plant and equipment carried out at revaluation model.
Instead revaluation is only required whenever there is a material differences between fair value and carrying amount.
[Para 34]
11) As the land is not depreciated in normal circumstances therefore its surplus is transferred at the time of disposal.

Page 5 of 36

22
Q.1 Faraday Pharmaceutical Limited (FPL) acquired a building for Rs. 200 million on July 1, 2005. The following information relating to the
building is available:
1) It is being depreciated on the straight line basis, over 20 years.
2) FPL uses the revaluation model for subsequent measurement of its property, plant and equipment and accounts for revaluations
on the *net replacement value method. The details of revaluation carried out by the independent valuer during the past years
are as follows:
Revaluation date Fair value
Rupees in million
July 1, 2006 230
July 1,2007 170
July 1, 2008 180
3) FPL transfers the maximum possible amount from the revaluation surplus to retained earnings on an annual basis.
4) There is no change in the useful life of the building.
Required:
a) Prepare the journal entries to record the above transactions from the date of acquisition of the building to the year ended June
30, 2009.
b) Prepare following ledgers for all relevant years.
• Building Account
• Accumulated Depreciation Account
• Revaluation Surplus Account
*Net replacement value method simply means eliminate the existing accumulated depreciation against asset.
Q.2 Following information is available to you

1) Cost of plant is Rs. 360,000 purchased on 1 July 2009.


2) It is depreciated on straight line basis over a period of 10 years.
3) Detail of revaluation was as follows
Revaluation Date FV
1 July 2010 400,000
1 July 2011 280,000
1 July 2012 290,000
Required: Prepare following ledgers from the date of purchase upto the year ended 30-6-2013.
a) Plant Account
b) Accumulated Depreciation Account
c) Revaluation Surplus Account

Q.3 Awesome Industries Limited (AIL) manufactures components for textile machinery. It purchased a plant on 1 July 2008 at a cost of Rs.
200 million. It has an estimated useful life of five years and no residual value.
AIL revalues its plant on an annual basis. The details of revaluations performed by Supreme
Valuation Service, an independent firm of valuer, are as follows:

Fair value - 1 July 2009 Rs. 180 million


Fair value - 1 July 2010 Rs. 108 million
Fair value - 1 July 2011 Rs. 88 million

Required:
Prepare journal entries for the year ended 30 June 2009, 2010, 2011 & 2012.

Q.4 MWL is a listed company and is engaged in the business of purifying and marketing of bottled water.
MWL purchased a bottling plant on 1-7-2006 at a cost Rs 90 million. The plant has a useful life of 10 years with no residual value.
Deprecation is charged on straight line method over plants’ useful life. MWL revalues its plant at the end of every two years.
The revalued amounts determined by Jet Values, an independent firm of valuers are as follows
i. On 30-6-2008 : Rs 64 million
ii. On 30-6-2010 : Rs 60 million
However there was no change in the expected useful life and residual value of the plant since the purchase of bottling plant

Required:
Prepare the relevant accounting entries up to the year end 30-6-2011 from the date of purchase of Bottling Plant

Page 6 of 36

23
Important Points to remember
Entry of elimination of accumulated depreciation On the date of revaluation

Depreciation on revalued amount (FV) After revaluation on the basis of remaining life

Transfer of surplus After revaluation in the periods in which depreciation is


charged on the basis of revalued amount.

Disclosures of property, plant and equipment:


• Schedule of PPE.
• Measurement basis
• Useful life / rate of depreciation
• Method of depreciation
• Detail of disposal of an item of PPE.
Disclosure requirements if an item of property, plant and equipment is revalued.
When items of property, plant and equipment are stated at revalued amounts the following must be disclosed:
• The effective date of the revaluation (if there are more than one revaluation dates then the latest date);
• Whether an independent valuer was involved and its name if available;
• For each revalued class of property, plant and equipment, the carrying amount that would have been recognised had
the asset been carried under the cost model; and
• The revaluation surplus, indicating the change for the period and any restrictions on the distribution of the balance to
shareholders.
Point to remember:
Revaluation surplus is presented in other comprehensive income in statement of comprehensive income.
Solutions:
A.1 Faraday
a) Journal Entries
Debit Credit
Date Particulars
Rs. in,000 Rs. in,000
1-Jul-05 Building 200,000
Bank 200,000
(Record purchase of plant)
30-Jun-06 Depreciation 10,000 10,000
Accumulated depreciation – Building
(Record depreciation for the year 2005-6)
Working: Rs. 200,000 20 = Rs. 10,000
1-Jul-06 Accumulated depreciation – Building 10,000
Building 10,000
(Reversal of prior year depreciation)
1-Jul-06 Building 40,000
Surplus on revaluation of fixed assets 40,000
(Increase in value through revaluation)
Working: Rs. 230,000 - Rs. 190,000 = Rs. 40,000
30-Jun-07 Depreciation 12,105
Accumulated depreciation - Building 12,105
(Record depreciation for the year 2006- 7)
Working: Rs. 230,000 19 = Rs. 12,105
30-Jun-07 Surplus on revaluation of fixed assets 2,105
Retained earnings/Profit & loss account 2,105
(transfer of surplus through retained earning to the extent of excess
depreciation)

Page 7 of 36

24
Working: Rs. 40,000 - 19 = Rs. 2,105
l-Jul-07 Accumulated depreciation - Building 12,105
Building 12,105
(Reversal of prior year depreciation)
l-Jul-07 Surplus on revaluation of fixed assets 37,895
Revaluation expense 10,000
Building 47,895
(Decrease in value through revaluation)
Working:
Reversal of Surplus balance (Rs. 40,000 - Rs. 2,105) Rs. 37,895.
Balancing figure of Rs. 10,000 charged to Profit and Loss
Building value decline: (Rs. 230,000 - Rs. 12,105) - Rs. 170,000 =Rs. 47,895
30-Jun-08 Depreciation 9,444
Accumulated depreciation - Building 9,444
(Record depreciation for the year 2007-8)
Working: Rs. 170,000 18 = Rs. 9,444
l-Jul-08 Accumulated depreciation - Building 9,444
Building 9,444
(Reversal of prior year depreciation)
l-Jul-08 Building 19,444
Reversal of Revaluation Loss 9,444
Surplus on revaluation of fixed assets (balancing) 10,000
(Reversal of prior year Revaluation Loss)
Working:
Revaluation income = Rs. 10,000 - [ Rs. 10,000 - Rs. 9,444] = Rs. 9,444
Building: [Rs. 170,000 - Rs. 9,444] - Rs. 180,000 =Rs. 19,444
30-Jun-09 Depreciation 10,588
Accumulated depreciation - Building 10,588
(Record depreciation for the year 2007-8)
Working: Rs. 180,000 17 = Rs. 10,588
30-Jun-09 Surplus on revaluation of fixed assets 588
Retained earnings 588
(Reverse the excess depreciation)
Working: Rs. 10,000 17 = Rs. 588

b)
Ledger Accounts
Building (cost)
Bank 200,000,000
C/D 200,000,000
B/D 200,000,000 Acc Dep 10,000,000
Revaluation 40,000,000
Surplus
C/D 230,000,000
B/D 230,000,000 Acc Dep 12,105,263
Revaluation Surplus 37,894,737
Revaluation Loss 10,000,000
C/D 170,000,000
B/D 170,000,000 Acc Dep 9,444,444
Revaluation 10,000,000
Surplus (W-1)
Reversal of Loss 9,444,444
C/D 180,000,000

Page 8 of 36

25
Accumulated Depreciation
C/D 10,000,000 Depreciation 10,000,000
(200 ÷ 20)
Building 10,000,000 B/D 10,000,000
C/D 12,105,263 Depreciation 12,105,263
(230 ÷ 19)
Building 12,105,263 B/D 12,105,263
C/D 9,444,444 Depreciation 9,444,444
(176 ÷ 18)
Building 9,444,444 B/D 9,444,444
C/D 10,588,235 Depreciation 10,588,235
(180 ÷ 17)

Revaluation Surplus

- -
R/E 2,105,263 Building 40,000,000
C/D 37,894,737
Building 37,894,737 B/D 37,894,737
C/D -
R/E 588,235 B/D -
C/D 9,411,765 Building 10,000,000

The amount of transfer of surplus can also be calculated as the difference between depreciation based on revalued amount and the
asset’s cost. Therefore, in questions, words “incremental depreciation on account of revaluation” will have same meaning i.e transfer
that amount from revaluation surplus to retained earnings.

A.2
Answer
Plant
1-7-2009
Cash 360,000
C/D 360,000
B/D 360,000 Acc Depreciation 36,000
Revaluation Surplus 76,000
C/D 400,000
B/D 400,000 Acc Depreciation 44,444
Revaluation Surplus 67,556
Revaluation Loss 8,000
C/D 280,000
B/D 280,000 Acc Depreciation 35,000
Reversal of loss (W-1) 7,000
Revaluation Surplus 38,000
C/D 290,000

Accumulated Depreciation
Depreciation 36,000
C/D 36,000
Plant 36,000 B/D 36,000
Depreciation 44,444
C/D 44,444

Page 9 of 36

26
Plant 44,444 B/D 44,444
Depreciation 35,000
C/D 35,000
Plant 35,000 B/D 35,000
Depreciation 41,429
C/D 41,429

Revaluation Surplus
Retained Earnings 8,444 Plant 76,000
C/D 67,556
Plant 67,556 B/D 67,556
C/D -
Retained Earnings 5,429 B/D -
Plant 38,000
C/D 32,571

A.3
Awesome Industries
1-7-2008
Plant 200 M
Cash 200 M
30-6-2009
Depreciation 40 M
Acc Depreciation 40 M (200 M ÷ 5)
1-7-2009
Acc Depreciation 40 M
Plant 40 M
1-7-2009
Plant 20 M
Revaluation Surplus 20 M (180- 160)
30-6-2010
Depreciation 45 M
Acc Depreciation 45 M (180M÷4)
30-6-2010
Revaluation Surplus 5M
Retained Earnings 5M (20M÷4)
1-7-2010
Acc Depreciation 45 M
Plant 45M
1-7-2010
Revaluation Surplus 15 M
Revaluation Loss 12 M
Plant 27 M [(180 – 45) -108]
30-6-2011
Depreciation 36 M
Acc Depreciation 36 M (108÷3)
30-6-2011
No transfer as no surplus

Page 10 of 36

27
1-7-2011
Acc Depreciation 36 M
Plant 36 M
1-7-2011
Plant 16 M
Revaluation Surplus 8M
Reversal of loss 8M

30-6-2012 Depreciation 44 M
Acc depreciation 44 M (88÷2)

30-6-2012 Revaluation Surplus 4M


Retained Earning 4M (8÷2)

A.4

Solution
MWL
Journal Entries
1-7-2006 Rs 000 Rs 000
Plant 90,000
Cash/Bank 90,000
30-6-2007
Depreciation 9,000
Acc Depreciation 9,000
(90,000÷10)
30-6-2008
Depreciation 9,000
Acc Depreciation 9,000
30-6-2008
Acc Depreciation 18,000
Plant 18,000
30-6-2008
Revaluation Loss 8,000
Plant 8,000
(WDV (90,000 – 18,000) = 72,000
FV = 64,000)
30-6-2009
Depreciation 8,000
Acc Depreciation 8,000
(64,000÷8)
30-6-2010
Depreciation 8,000
Acc Depreciation 8,000
30-6-2010
Acc Depreciation 16,000
Plant 16,000
30-6-2010 Plant 12,000
Reversal of loss (W-1) 6,000
Revaluation Surplus (W-1) 6,000

Page 11 of 36

28
30-6-2011
Depreciation 10,000
Acc Depreciation 10,000
(60,000÷6)
30-6-2011
Revaluation Surplus 1,000
Retained Earning 1,000
(6000 ÷ 6)

Self-Test Questions (Revaluation)

Q.1
Rupees
Cost of machine at 1/1/2011: 100,000
Fair value
• 1/1/2012 180,000
• 1/1/2013 60,000
• 1/1/2014 77,000
• 1/1/2015 120,000
Depreciation: 10% per annum to a nil residual value
The company’s policy is to transfer the realized portion of the revaluation surplus to retained earnings as the asset is used.
Required:
Prepare the asset’s account as a net carrying amount account (i.e. do not prepare separate cost and accumulated depreciation
accounts) as well as revaluation surplus accounts for the year ended 31-12-2011 to 31-12-2015.

Q.2 A company always revalues its fixed assets to fair value using net replacement vale method. Following information relates to
its specialized vehicle.

Date Rupees
Purchase price 1 July 2005 500,000
Fair value 1 January 2007 420,000
Fair value 1 January 2009 165,000

Useful life: five year


Required:
a)

Page 12 of 36

29
Solutions
Self-Test Questions (Revaluation)

A.1
Machinery Account – WDV
1-1-2001
Bank 100,000 Depreciation 10,000
C/D 90,000
B/D 90,000 Depreciation 20,000
Revaluation Surplus 90,000 (180,000 ÷ 9)
C/D 160,000
B/D 160,000 Revaluation Surplus 80,000
Revaluation Loss 20,000
Depreciation 7,500
(60,000 ÷ 8)
C/D 52,500
B/D 52,500 Depreciation 11,000
(77,000 ÷ 7)
Reversal of Loss (W-1) 17,500
Revaluation Surplus 7,000
C/D 66,000
B/D 66,000 Depreciation 20,000
(120,000 ÷ 6)
Revaluation Surplus (W-2) 54,000

C/D 100,000

W-1 WDV=52,500 W-2 WDV=66,000


HCA=70,000 FV=120,000
FV=77,000

Revaluation Surplus
1-1-2001 - -
C/D - B/D -
Retained Earnings 10,000 Machine 90,000
(90,000 ÷ 9)
C/D 80,000
Machine 80,000 B/D 80,000
C/D -
Retained Earnings 1,000 B/D -
(7,000÷7)
Machine 7,000
C/D 6,000
Retained Earnings 10,000 B/D 6,000
(60,000÷6)
Machine 54,000
C/D 50,000

Page 13 of 36

30
A.2

a) Journal Entries
1-7-2005
Vehicles 500,000
Bank 500,000
31-12-2003
Depreciation 50,000
Acc Depreciation 50,000
(500,000÷5 x6/12)
31-12-2006
Depreciation 100,000
Acc Depreciation 100,000
(500,000÷5)

1-1-2007
Acc Depreciation 150,000
Vehicle 150,000
1-1-2007
Vehicle 70,000
Revaluation Surplus 70,000
31-12-2007
Depreciation 120,000
Acc Depreciation 120,000
(420,000÷3.5)
31-12—2007
Revaluation Surplus 20,000
Retained Earnings 20,000
(70,000÷3.5)
31-12-2008
Depreciation 120,000
Acc Depreciation 120,000
31-12-2008
Revaluation Surplus 20,000
Retained Earnings 20,000
(70,000÷3.5)
1-1-2009
Acc Depreciation 240,000
Vehicle 240,000
1-1-2009
Revaluation Surplus 15,000
Vehicle 15,000
31-12-2009
Depreciation 110,000
Acc Depreciation 110,000
(165,000 ÷ 1.5)
31-12-2009
Revaluation Surplus 10,000
Retained Earnings 10,000
(15,000 ÷ 1.5)
Page 14 of 36

31
b)
Notes to Financial Statements
For the year ended 31-12-2009
Vehicles 2009 2008
Cost/Revalued Amount
Opening Balance 420,000 420,000
Revaluation Surplus/Loss (15000) -
Elimination (240,000) -
Closing Balance 165,000 420,000

Acc Depreciation
Opening Balance 240,000 120,000
Elimination (240,000) -
Depreciation for the year 110,000 120,000
Closing Balance 110,000 240,000
Carrying Amount 55,000 180,000

Page 15 of 36

32
Extra questions of revaluation
Question No. 1
On 1st January 2014 Omega Chemicals Limited (OCL) changed its valuation model from cost to revaluation for its buildings. The
following information pertains to its buildings as at 01-01-2014.

Estimated useful life Revalued amounts


Accumulated
as originally Cost as per valuation
depreciation
estimated report
------------------ Rs. In million ---------------
Factory buildings 20 years 100.00 37.50 52.00
Office buildings 25 years 164.50 26.32 149.94

As per the report of the professional valuer, the was no change in estimated useful life of the buildings:
On 1 July 2014, one of the office buildings was sold for Rs. 30 million. On 01-01-2014 written down value before valuation and
revalued amount of the sold building amounted to Rs. 27.72 million and Rs. 31.92 million respectively.
On 31 December 2014, factory buildings were revalued at Rs. 64 million where as there was no change in value of the office buildings.
OCL uses straight line method of depreciation which is charged from the date the asset is available for use upto the date of disposal.
Revaluation is to be accounted for by using net replacement value method.
Required:
In the light of the requirements of the international financial reporting standards, prepare accounting entries from the above
information for the year ended 31 December 2014. (17)
Question No. 2
Shahzad Textile Mills Limited (STML) purchased a plant for Rs. 500 million on 1 July, 2010. the plant has an estimated useful life of 10
years and no residual value.
STML uses revaluation model for subsequent measurement of its property, plant and equipment and accounts for revaluations on net
replacement value method. The details of revaluations performed by an independent firm of valuers are as follows:

Revaluation date Fair value


1 July 2011 Rs. 575 million
1 July 2012 Rs. 390 million
1 July 2013 Rs. 380 million

Required:
Prepare journal entries to record the above transactions from the date of acquisition of the plant to the year ended 30 June 2014.
(Ignore tax implications) (15)
Question 3
PQR Enterprises was incorporated on 1 July 2012. The company depreciates its property, plant and equipment on straight line basis
over their useful life. It uses revaluation model for subsequent measurement of the property, plant and equipment and has a policy of
revaluing these after every two years.
Following information pertains to its property, plant and equipment:

Value as determined
Cost as on 01- WDV as on
by professional valuer Useful life in years
07-2013 01-07-2013
on 30-06-2014
Assets
Remaining as
Original at
-------------- Rs. In million ------------------ determined by
acquisition
valuer
Office building 6,000 5,500 5,750 12 8
Factory building 4,400 3,960 3,320 10 9
Warehouse 4,500 4,050 3,350 10 8

Page 16 of 36

33
During the year there were no addition or deletion in the above assets.
As per policy, PQR transfers the maximum possible amount from the revaluation surplus to retained earnings on an annual basis.
Required:
Prepare necessary journal entries for the year ended 30 June 2014 and 2015. (12)
Question 4
The following information pertains to Sherdil Limited (SL):
(i) Buildings and equipment were acquired on 1 January 2014 for Rs. 450 million and Rs. 50 million respectively.
(ii) The relevant information relating to both assets is summarised below:

Subsequent
Assets Depreciation method Life/rate
measurement
Buildings Straight line 20 years Annual revaluation
Equipment Reducing balance 10% Cost

SL transfers the maximum possible amount from revaluation surplus to retained earnings on an annual basis.
(iii) The revalued amount of buildings as determined by Accurate Valuers (Private) Limited, an independent valuation company, on
1 January 2015 and 2016 was Rs. 456 million and Rs. 378 million respectively.
(iv) Equipment costing Rs. 35 million was purchased on 1 August 2015. Half of the equipment purchased on 1 January 2014 was
disposed off on 30 June 2016.
Required:
Prepare Building A/c and Equipment A/c as well as separate ledgers for accumulated depreciation for the year ended 31-12-2015 and
31-12-2016.
Question 5
The following information pertains to Sherdil Limited (SL):
(i) Buildings and equipment were acquired on 1 January 2014 for Rs. 450 million and Rs. 50 million respectively.
(ii) The relevant information relating to both assets is summarised below:

Subsequent
Assets Depreciation method Life/rate
measurement
Buildings Straight line 20 years Annual revaluation
Equipment Reducing balance 10% Cost

SL transfers the maximum possible amount from revaluation surplus to retained earnings on an annual basis.
(iii) The revalued amount of buildings as determined by Accurate Valuers (Private) Limited, an independent valuation company, on
1 January 2015 and 2016 was Rs. 456 million and Rs. 378 million respectively.
(iv) Equipment costing Rs. 35 million was purchased on 1 August 2015. Half of the equipment purchased on 1 January 2014 was
disposed off on 30 June 2016.
Required:
In accordance with International Financial Reporting Standards, prepare a note on ‘Property plant & equipment’ (including
comparative figures) for inclusion in SL’s financial statements for the year ended 31 December 2016. (18)

Page 17 of 36

34
Solution:
Answer No. 1
Omega Chemicals Limited
Accounting entries for the year ended 31 December 2014

Debit Credit
Date Description
Rs. Millions Rs. Millions
Factory buildings
1-1-2014 Accumulated depreciation - Factory buildings 37.50
Factory buildings 37.50
(Reversal of accumulated depreciation on revaluation of factory
buildings on 31 December 2013)
1-1-2014 Revaluation loss [52 – (100 – 37.5) 10.50
Factory buildings 10.50
(2013 Revaluation loss of factory buildings accounted for in
2014)
31-Dec-2014 Depreciation expense (52 ÷ 12.5 W-1) 4.16
Accumulated depreciation – Factory buildings 4.16
(Depreciation expenses for the year ended 31 December 2014)
31-Dec-2014 Accumulated depreciation – Factory buildings 4.16
Factory buildings 4.16
(Reversal of accumulated depreciation on revaluation of factory
buildings on 31 December 2014)
31-Dec-2014 Factory buildings [64 – (52 – 4.16)] 16.16
Reversal of revaluation loss [10.5-(10.55÷12.5W-1)] 9.66
Revaluation surplus – factory buildings (Bal.) 6.50
(revaluation of factory buildings on 31 December 2014 and
reversal of previous revaluation loss).
Office buildings
1-1-2014 Accumulated depreciation 26.32
Office building 26.32
1-1-2014 Office building 11.76
Revaluation surplus [149.94 – (164.5 – 26.32)] 11.76
1-Jul-2014 Depreciation expense [31.92 5 ÷ 21 × 0.5] 0.76
Accumulated depreciation-Office buildings 0.76
(Depreciation expenses for the six months ended 1 July 2014 for
the office building block sold)
1-Jul-2014 Revaluation surplus [(31.92-27.72=4.2) 5 ÷ 21 × 0.5] 0.10
Retained earnings 0.10
(Transfer of incremental depreciation for the six months ended
31 December 2014 to retained earnings)
1-Jul-2014 Bank 30.00
Accumulated depreciation 0.76
Loss on sale of Office building (bal) 1.16
Office buildings 31.92
(Sale of office building)

Page 18 of 36

35
1-Jul-2014 Revaluation surplus 4.1
Retained earnings 4.1
(4.2 - 0.1)
31-Dec-2014 Depreciation expense (149.94 – 31.92) ÷ 21 5.62
Accumulated depreciation-Office buildings 5.62
(Depreciation expenses for the year ended 31 December 2014)
31-Dec-2014 Revaluation surplus-Office building [149.94-(164.5-26.32)- 0.36
4.21÷21
Retained earnings 0.36
(Transfer of incremental depreciation for the year ended 31
December 2014 to retained earnings)
(W-1: Remaining useful life of the buildings on the revaluation date of 31 December 2013

Years

100 37.5 
Factory buildings:  = 5; = 7.5; 20 − 7.5 = 12.5 12.50
 20 5 

164.5 26.32 
Office buildings:  = 6.58; = 4; 25 − 4 = 21 21.00
 25 6.58 

Answer No. 2
“Rs. In Million”
Date Particulars
Debit Credit
1-7-2010 Plant 500
Bank 500
10-6-2011 Depreciation 50
Accumulated depreciation (500 ÷ 10) 50
1-7-2011 Accumulated depreciation 50
Plant 50
1-7-2011 Plant 125
Revaluation Surplus 125
30-6-2012 Depreciation 63.89
Accumulated depreciation [575/9] = 63.89 63.89
30-6-2012 Revaluation Surplus 13.89
Retained Earnings [125/9] = 13.89 13.89
1-7-2012 Accumulated depreciation 63.89
Plant 63.89
1-7-2012 Revaluation Surplus 111.11
Revaluation Loss 10
Plant 121.11
WDV = 575 – 63.89 = 511.11
FV = 390.00
30-6-2013 Depreciation 48.75
Accumulated depreciation [390÷8] = 48.75 48.75
1-7-2013 Accumulated depreciation 48.75
Plant 48.75

Page 19 of 36

36
1-7-2013 Plant 38.75
Reversal of Loss [10 – 1.25 (10/8)] 8.75
Rev. Surplus 30.00
WDV = 390 – 48.75 = 341.25
FV = 380.00
38.75
30-6-2014 Depreciation 54.29
Accumulated depreciation [380÷7] = 54.29 54.29
30-6-2014 Rev. Surplus 4.29
Retained Earnings [30÷7] = 4.29 4.29

Answer No. 3
PQR
Journal Entries for the year ended 30-6-2014
Office Buildings:

30-6-14 Depreciation 500 (6,000 ÷ 12)


Accumulated depreciation 500
30-6-14 Accumulated depreciation 1,000 (500 + 500)
Office building 1,000
30-6-14 Office building 750
R. Surplus 750
WDV = 5,500 – 500 = 5,000
FV = 5,750
Factory Buildings:
30-6-14 Depreciation 440 (4,400 ÷ 10)
Accumulated depreciation 440
30-6-14 Accumulated depreciation 880 (440 + 440)
Factory building 880
30-6-14 Revaluation Loss 200
Factory building 200
WDV = 3,960 – 440 = 3,520
FV = 3,320
200
Warehouse:
30-6-14 Depreciation 450 (4,500 ÷ 10)
Accumulated depreciation 450
30-6-14 Accumulated depreciation 900 (450 + 450)
Warehouse 900
30-6-14 Revaluation Loss 250
Warehouse 250
WDV = 4,050 – 450 = 3,600
FV = 3,350
250
Journal Entries for the year ended 30-6-2015
Office Buildings:

Page 20 of 36

37
30-6-15 Depreciation 718.75 (5,750 ÷ 8)
Accumulated depreciation 718.75
30-6-15 Revaluation Surplus 93.75 (750 ÷ 8)
Retained Earnings 93.75
Factory Buildings:
30-6-15 Depreciation 369 (3,320 ÷ 9)
Accumulated depreciation 369
Warehouse:
30-6-15 Depreciation 419 (3,350 ÷ 8)
Accumulated depreciation 419

Ans. 4
Buildings
1-1-2014 Cash 450
c/d 450
1-1-2015 b/d 450 1.1.2015 Acc. Depreciation 22.5
1-1-2015 R.S 28.5
c/d 456
1-1-2016 b/d 456 1.1.2016 Acc. Depreciation 24
1.1.2016 R.S 27
1.1.2016 R.L 27
c/d 378

Accumulated Depreciation
31-12-2014 Depreciation (450÷20) 22.5
c/d 22.5
1.1.2015 Building 22.5 1.1.2015 b/d 22.5
31.12.2015 Depreciation (456÷19) 24
c/d 24
1.1.2016 Building 24 1.1.2016 b/d 24
31.12.2016 Depreciation (378÷18) 21
c/d 21

R.S
31.12.2015 Retained earnings (28.5÷19) 1.5
1.1.2015 Building 28.5
c/d 27
1.1.2016 Building 27 1.1.2016 b/d 27
c/d -
(W-1) 1-1-2015
WDV = 450 – 22.5 = 427.5
FV = 456
R. Surplus = 28.5
1-1-2016
WDV = 456 – 24 = 432
FV = 378
R. Loss = 54
Page 21 of 36

38
R. Surplus 27
R. Loss (balance) 27
Building 54

(b) Equipment

1-1-2014 Cash 50
31-12-2014 c/d 50
b/d 50
1-8-2015 Cash 35
31-12-2015 c/d 85
b/d 85 30-6-2016 Disposal 25
c/d 60

Accumulated Depreciation

31-12-2014 Depreciation (50×10%) 5


c/d 5
01.01.2015 b/d 5
31.12.2015 Depreciation 5.96
(50-5)×10% + 35 × 10% ×
5/12
c/d 10.96
01.01.2016 b/d 10.96
30.06.2016 Disposal 5.7625 31.12.2016 Depreciation 6.3915
[25×10%] = 2.5 [85-10.96-20.25]×10%
[25-2.5]×10% = 2.25 + 20.25×10%×6/12
[25-2.5-2.25]×10%× 6/12 = [5.379 + 1.0125]
1.0125
c/d 11.58

Answer 5
Property, plant and equipment

2016
Building Equipment Total
-------- Rs. In million ---------
Gross carrying amount
Opening 456.00 85.00 541.00
Elimination (24.00) (24.00)
Revaluation surplus [28.5 – (28.5 ÷ 19)] (27.00) (27.00)
Revaluation loss (27.00) (27.00)
Disposal (25.00) (25.00)
Closing 378.00 60.00 438

Page 22 of 36

39
Accumulated depreciation
Opening 24.00 10.96 34.96
Elimination (24.00) (24.00)
Disposal [2.5 + 2.25 + 1.01] (5.76) (5.76)
Depreciation
(378÷18) 21.00 21.00
[(74.04-20.25)×10% + (20.25×10%×6÷12)] 6.39 6.39
Closing 21.00 11.59 32.59
Carrying amount [378 – 21] 357.00 48.41 405.41
[60-11.59]

Property, plant and equipment


2015
Building Equipment Total
-------- Rs. In million ---------
Gross carrying amount
Opening 450.00 50.00 500.00
Elimination (22.50) (22.50)
Revaluation 28.50 28.50
Addition 35.00 35.00
Closing Cost 456.00 85.00 541.00
Accumulated depreciation
Opening 22.50 5.00 27.50
Elimination (22.50) (22.50)
Disposal
Depreciation
(456÷19) 24.00 24.00
[(45×10%) + (35×10%×5÷12)] 5.96 5.96
Closing 24.00 10.96 34.96
Closing amount (456 – 24] 432.00 74.04 506.04
[85-10.96]

Disclosures related to property, plant and equipment:


Buildings Equipment
Measurement base Revaluation model Cost model
Useful life/depreciation rate 20 years 10%
Depreciation method Straight line Reducing balance
Disclosures regarding revaluation of buildings:
1.The last revaluation was performed on 1 Jan 2016.
2.The revaluation was performed by Accurate Valuers (private) Limited, an independent
firm of valuers.
3.Carrying value had the cost model been used.
2016 2015
Buildings 382.50 405.00
[450 – (22.5 x 2)] [450 – (22.5 x 3)]
Page 23 of 36

40
4. Change in revaluation surplus:
2016 2015
Opening balance 27 -
Revaluation (27) 28.5
Transfer of surplus - (1.5)
Closing balance - 27
There is restriction on distribution of revaluation surplus as dividend to shareholders.

Detail of Disposal of Equipment:

Gross carrying amount 25


Accumulated Depreciation (5.7625)
Carrying Amount 19.2375
Sale proceeds Not Available
Mode of Disposal Not Available
Particulars of buyer Not Available

Page 24 of 36

41
ICAP study text
IAS 16: Property, plant and equipment
DISCLOSURE REQUIREMENTS OF IAS 16

IAS 16 Property, plant and equipment requires the following disclosures in the notes to the financial statements, for each major
class of property, plant and equipment.

The measurement bases used (cost or revaluation model)

The depreciation methods used

The useful lives or depreciation rates used

Gross carrying amount (means cost or revalued amount) and the accumulated depreciation at the beginning and at the end
of the period

A reconciliation between the opening and closing values for gross carrying amounts and accumulated depreciation,
showing:
i. additions during the year
ii. disposals during the year
iii. depreciation charge for the year

iv. impairment losses

v. the effect of revaluations.

An entity must also disclose:



the existence and amounts of restrictions on title, and property, plant and equipment pledged as security
for liabilities;

the amount of expenditures recognised in the carrying amount of an item of property, plant and equipment in
the course of its construction;

the amount of contractual commitments for the acquisition of property, plant and equipment; and

if it is not disclosed separately in the statement of comprehensive income, the amount of compensation from
third parties for items of property, plant and equipment that were impaired, lost or given up that is included
in profit or loss.

Disclosures for assets stated at revalued amounts


When items of property, plant and equipment are stated at revalued amounts the following must be disclosed:
I.
the effective date of the revaluation;
II.
whether an independent valuer was involved;

III.
for each revalued class of property, plant and equipment, the carrying amount that would have been recognised had the
assets been carried under the cost model; and
IV.
the revaluation surplus, indicating the change for the period and any restrictions on the distribution of the balance to
shareholders.

Additional disclosures encouraged by IAS 16


IAS 16 encourages disclosure of the following information as users of financial statements might find it to be useful.
I.
the carrying amount of temporarily idle property, plant and equipment;
II.
the gross carrying amount of any fully depreciated property, plant and equipment that is still in use;
III.
the carrying amount of property, plant and equipment retired from active use and held for disposal; and

IV. when the cost model is used, the fair value of property, plant and equipment when this is materially different from the
carrying amount.

Page 25 of 36

42
Further Practice:
Example 01:

Question: Ali Limited (AL) uses the revaluation model for subsequent measurement of its property, plant and equipment
and has a policy of revaluing its assets on an annual basis using the net replacement value method.
The following information pertains to AL’s building:
i. The building was purchased on 01 January 2010 for Rs. 2 hundred million with expected usefullife of
ten years.
ii. AL depreciates buildings on the straight line basis over their useful life.
iii. The results of revaluations carried out during the last three years by Standard Valuation Service,an
independent firm of values, are as follows:

Revaluation date Fair value Rs. in million


1 January 2011 280
1 January 2012 170
1 January 2013 180
Required:
The journal entries relating to the above transactions including revaluations for the year ended December 31, 2010,
2011, 2012 and 2013.

Answer:

Debit Credit
Date - Property, plant and equipment
---- Rs. in million ----
January Building Account 200
01,2010 Payable 200
December Depreciation (200÷10) 20
31,2010 Accumulated depreciation 20
January Accumulated depreciation 20
01,2011 Building 20
January Building (280-(200-20)) 100
01,2011 Revaluation surplus (OCI) 100
January Depreciation (200÷9) 31
31,2011 Accumulated depreciation 31
January Revaluation surplus (100/9) 11
31,2011 Retained earnings 11
January Accumulated depreciation 31
01,2012 Building 31
January Revaluation surplus (SOCI) (280-31)-170 79
01,2012 Building 79
December Depreciation (170÷8) 21.25
31,2012 Accumulated depreciation 21.25
December Revaluation surplus 1.25
31,2012 Retained earnings 1.25
(100-11-79)=10/8 =1.25
January Accumulated depreciation 21.25
01,2013 Building 21.25
January Building 31.25
01,2013 Revaluation surplus (180-(170-21.25) 31.25

Page 26 of 36

43
December Depreciation (180÷07) 25.7
31,2013 Accumulated depreciation 25.7
December Revaluation surplus 5.7
31,2013 Retained earnings 5.7
(10-1.25+31.25)/7

Example 02 :
Question: Adjustments Limited has carried out a review of its non-current assets.
(a) A grinder was purchased on 1 January 2012 for Rs. 100,000. The plant had an estimated usefullife of ten years and a residual
value of nil. Depreciation is charged on the straight line basis. On1 January 2015, when the asset’s net book value is Rs. 70,000,
the directors decide that it would be more appropriate to depreciate this asset using the sum of digits’ approach. The
remaining useful life is unchanged.
(b) The company purchased a fifty year leased property some years ago for Rs. 1,000,000. This was being depreciated over its
life on a straight line basis. On 1 January 2015, when the net book value is Rs. 480,000 and twenty-four years of the lease are
remaining, the asset is revalued to Rs. 1,500,000. This revised value is being incorporated into the accounts.
Required:
Explain the effects of these changes on the depreciation for the year to 31 December 2015.
Answer:
(a) The grinder was purchased in 2012 and was originally being depreciated on a straight line basis. Ithas now been decided to
depreciate this on the sum of digits basis.
IAS 16 requires that depreciation methods be reviewed periodically and if there is a significant changein the expected pattern of
economic benefits, the method should be changed. Depreciation adjustments should be made in current and future periods. This
change might be appropriate if, for instance, usage of the machine is greater in the early years of an asset’s life when it is still new
and consequently it is appropriate to have a higher depreciation charge.
If the change is implemented, the net book value of the asset should be depreciated over the remaining useful life commencing
with the period in which the change is made.
The depreciation charge for the remaining life of the asset will therefore be as follows.

Year Digits Depreciation


Rs.
2015 7 7/28 x Rs.70,000 17,500
2016 6 6/28 x Rs.70,000 15,000
2017 5 12,500
2018 4 10,000
2019 3 7,500
2020 2 5,000
2021 1 2,500
— ———–
7 (7 + 1)/2 — Rs. 70,000
28 ———–

Disclosure will need to be made in the accounts of the details of the change, including the effect on the charge in the year.
The reassessment of the depreciation method is not a change in accounting policy and neither rectification of a fundamental error
so the effects of the change will not affect the previously reported financial statements
(b) Leasehold Property:
IAS 16 requires that the subsequent charge for depreciation should be based on the revalued amount. The annual depreciation will
therefore be Rs. 62,500, i.e. Rs.1,500,000 divided by the 24 years of remaining life.
There will then be a difference between the revalued depreciation charge and the historical depreciation charge.
The resulting excess depreciation may be dealt with by a movement in reserves, i.e. by transferring from the revaluation reserve to
retained earnings a figure equal to the depreciation charged on the revaluation surplus each year.

Page 27 of 36

44
Example 03:
Question: Shahwez Limited (SL) revalued its property on 1 April 2011 to Rs.20 million (Rs.8 millionfor the land). The property originally
cost Rs.10 million (Rs.2 million for the land) 10 years ago. Theoriginal useful life of 40 years is unchanged. SL’s policy is to make a
transfer to realised profits in respect of excess depreciation.

Required:

How will the property be reflected in the financial statements accounted for in the year ended 31 March 2012?

Answer:

Statement of comprehensive income extract for the year ended 31 March 2012

Depreciation expense
400
Other comprehensive income:

Revaluation gain 12,000


Statement of financial position extract as at 31 March 2012

Rs. In 000

Non-current assets Property 19,600


(20,000 – 400)

Equity Revaluation reserve 11,800

(12,000 – 200)

Statement of changes in equity extracts

Revaluation reserve Retained earnings


Revaluation surplus 12,000
Transfer of surplus (200) 200
Workings:
Gain on revaluation: Rs. In 000
Carrying value at revaluation date
(10,000 – ((10,000 – 2,000)/40 years x 10 years)) 8,000
Fair value 20,000
Gain on revaluation 12,000
Double entry:
Dr Property 10,000
(20,000 – 10,000)
Dr Accumulated depreciation 2,000
((10,000 – 2,000)/40 years x 10 years)
Cr Revaluation reserve 12,000
Depreciation charge for year to 31 March 2012:
Dr depreciation expense 400
((20,000 – 8,000)/30 years)
Cr Accumulated depreciation 400
Transfer of surplus:

Page 28 of 36

45
Historical cost depreciation charge 200
((10,000 – 2,000)/40 years)
Revaluation depreciation charge 400
Excess depreciation to be transferred 200
Dr Revaluation reserve 200
Cr Retained earnings 200

Page 29 of 36

46
Test Revaluation
Q. 1 following information pertained to a building acquired by Sk limited on 01.07.2012 for Rs. 360 million:
(i) The building is being depreciated on straight-line basis over 10 years.

(ii) SKL uses revaluation model for subsequent measurement of buildings. It accounts for revaluation on net replacement
value method. The details of revaluations as carried out by independent values are as follows:

Fair value
Revaluation date
(Rs. in million)
31 December 2013 323
31 December 2015 208
31 December 2017 167

(iii) There is no change in useful life of the building.

(iv) SKL transfers the maximum possible amount from the revaluation surplus to retained earnings on an annual basis.
(v) SKL’s financial year ends on 31 December.

Required:
Prepare entries to record revaluation surplus / loss on each of the above revaluation date. (12)

Q.2 Following information pertains to a building acquired by SK Limited (SKL) on 1 July 2012 for Rs. 360 million: The building is being
depreciated on straight-line basis over 10 years.

(i) SKL uses revaluation model for subsequent measurement of buildings. It accounts for revaluation on net replacement value
method. The details of revaluations as carried out by independent valuer are as follows:

Fair value
Revaluation date
(Rs. in million)
31 December 2013 323
31 December 2015 208
31 December 2017 167

(ii) There is no change in useful life of the building.


(iii) SKL transfers the maximum possible amount from the revaluation surplus to retained earnings on an annual basis.
(iv) SKL’s financial year ends on 30 June.

Required:
Prepare entries to record revaluation surplus / loss on each of the above revaluation date. (12)

Page 30 of 36

47
Answer: 1

“ Rs in Million”
Date Particulars Debit Credit
For the year ended 31-12-2012
1-7-2012 Building 360
Cash/Payable 360
31-12-2012 Depreciation 18
Accumulated depreciation (360/10) x 6/12 18
For the year ended 31-12-2013
31-12-2013 Depreciation 36
Accumulated depreciation 36
31-12-2013 Accumulated depreciation 54
Building 54
31-12-2013 Building 17
Revaluation Surplus (W-1) 17
For the year ended 31-12-2014
30-6-2014 Depreciation 38
Accumulated depreciation (323/8.5) 38
30-6-2014 Revaluation Surplus 2
Retained earnings (17/8.5) 2
For the year ended 31-12-2015
31-12-2015 Depreciation 38
Accumulated depreciation 38
31-12-2015 Revaluation Surplus 2
Retained earnings (17/8.5) 2
31-12-2015 Accumulated depreciation 76
Building 76
31-12-2015 Revaluation Surplus 13
Revaluation Loss (Bal) 26
Building (W-2) 39
For the year ended 31-12-2016
31-12-2016 Depreciation 32
Accumulated depreciation (208+6.5) 32
For the year ended 31-12-2017
31-12-2017 Depreciation 32
Accumulated depreciation (208+6.5) 32
31-12-2017 Accumulated depreciation 64
Building 64
31-12-2017 Building 23
Reversal of loss 18
Reversal Surplus (W-1) 5
(W-1) 31-12-2013
WDV = 360 – 54 = 306
FV = 323
R. Surplus = 17
(W-2) 31-12-2015
WDV = 323 – 76 = 247
FV = 208
R. Loss = 39
Page 31 of 36

48
“Don’t take salah as a burden. Allah gifted us salah as a relief from burden.”

(W-3) 31-12-2017
WDV = 208 – 64 = 144
FV = 167

R. Surplus = 23

Reversal of loss 26 R. Surplus 5


Extra depreciation to be charged (4 x 2) (8)
Net reversal of loss 18

If Ledgers are prepared.

Building
1-7-2012 Cash/Payable 360
c/d 360
1-1-2013 b/d 360
31-12-2013 Revaluation Surplus 17 31-12-2013 Accumulated depreciation 54
c/d 323
1-1-2014 b/d 323
c/d 323
1-1-2015 b/d 323
31-12-2015 Accumulated depreciation 76
Revaluation surplus 13
Revaluation loss 26
c/d 208
1-1-2016 b/d 208
c/d 208
1-1-2017 b/d 208
31-12-2017 Reversal of loss 18 31-12-2017 Accumulated depreciation 64
31-12-2017 Revaluation surplus 5 c/d 167

Accumulated Depreciation
31-12-2012 Depreciation (360/10 x 6/12) 18
c/d 18
1-1-2013 b/d 18
31-12-2013 Building 54 31-12-2013 Depreciation(360/10) 36
c/d -
1-1-2014 b/d -
31-12-2014 Depreciation(323/8.5) 38
c/d 38
1-1-2015 b/d 38
31-12-2015 Building 76 31-12-2015 Depreciation 38
c/d -
Page 32 of 36

49
b/d -
31-12-2016 Depreciation (208/6.5) 32
30-6-2017 c/d 32

1-1-2017 b/d 32
Building 64 31-12-2017 Depreciation 32
c/d -

Revaluation Surplus
b/d -
c/d -
1-1-2013 b/d -
31-12-2013 Building 17
30-6-2014 c/d 17
1-1-2014 b/d 17
31-12-2014 Retained Earnings(17/8.5) 2
c/d 15
1-1-2015 b/d 15
31-12-2015 Retained Earnings 2
31-12-2015 Building 13
c/d -
1-1-2016 b/d -
c/d -
1-1-2017 b/d -
c/d 5 31-12-2017 Building 5

(W-1) 31-12-2013
WDV = 360 – 18.36 = 306
FV = 323
R. Surplus = 17

(W-2) 31-12-2015
WDV = 323 – 76 = 247
FV = 208
R. Loss = 39
(W-3) 31-12-2017
WDV = 208 – 64 = 144
FV = 167
R. Surplus = 23

Reversal of loss 26 R. Surplus 5


Extra depreciation to be charged (4 x 2) (8)
Net reversal of loss 18

Page 33 of 36

50
Answer: 2

“ Rs in Million”
Date Particulars Debit Credit
For the year ended 30-6-2013
1-7-2012 Building 360
Bank 360
30-6-2013 Depreciation 36
Accumulated depreciation (360/10) 36
For the year ended 30-6-2014
31-12-2013 Depreciation 18
Accumulated depreciation (360/10) x 6/12 18
31-12-2013 Accumulated depreciation – Building (36+18) 54
Building 54
31-12-2013 Building 17
Revaluation Surplus (W-1) 17
30-6-2014 Depreciation 19
Accumulated depreciation (323/8.5) x 6/12 19
30-6-2014 Revaluation Surplus 1
Retain Earning (17/8.5 x 6/12 1
For the year ended 30-6-2015
30-6-2015 Depreciation 38
Accumulated depreciation (323/8.5) 38
30-6-2015 Revaluation Surplus 2
Retained earnings (17/8.5) 2
For the year ended 30-6-2016
31-12-2015 Depreciation 19
Accumulated depreciation (323/8.5) x 6/12 19
31-12-2015 Revaluation Surplus 1
Retained earnings (17/8.5) x 6/12 1
31-12-2015 Accumulated depreciation – Building 76
Building 76
31-12-2015 Revaluation Surplus (17 – 4) 13
Revaluation Loss 26
Building (W-2) 39
For the year ended 30-6-2016
30-6-2016 Depreciation 16
Accumulated depreciation (208/6.5) x 6/12 16
No transfer of surplus
For the year ended 30-6-2017 32
30-6-2017 Depreciation
Accumulated depreciation (208/6.5) 32
For the year ended 30-6-2018
31-12-2017 Depreciation 16
Accumulated depreciation (208/6.5) x 6/12 16
31-12-2017 Accumulated depreciation (16+32+16) 64
Building 64
31-12-2017 Building 23
Reversal of revaluation loss (W-3) 18
Revaluation Surplus 5

Page 34 of 36

51
30-6-2018 Depreciation 18.53
Accumulated depreciation (167/4.5) x 6/12 18.56
30-6-2018 Revaluation Surplus 0.56
Retain Earning (5/4.5) x 6/12 0.56

(W-1) 31-12-2013
WDV = 360 – 36 – 18 = 306
FV = 323
R. Surplus = 17

(W-2) 31-12-2015
WDV = 323 – 19 – 38 – 19 = 247
FV = 208
R. Loss = 39

(W-3) 31-12-2017
WDV = 208 – 16 – 32 – 16 = 144
FV = 167
R. Surplus = 23

Reversal of loss 26 R. Surplus 5


Extra depreciation to be charged (2+4+2) (8)
Net reversal of loss 18

[Ledgers not required. For extra information only.]

Building Account
1-7-2012 Cash 360
30-6-2013 c/d 360
1-7-2013 b/d 360
31-12-2013 Revaluation Surplus 17 31-12-2013 Accumulated depreciation 54
30-6-2014 c/d 323
1-7-2014 b/d 323
30-6-2015 c/d 12,700
1-7-2015 b/d 323
31-12-2015 Accumulated depreciation 76
31-12-2015 Revaluation surplus 14
31-12-2015 Revaluation loss 25
30-6-2016 c/d 208
1-7-2016 b/d 208
30-6-2017 c/d 208
1-7-2017 b/d 208
31-12-2017 Reversal of loss 18 31-12-2017 Accumulated depreciation 64
31-12-2017 Revaluation surplus 5 30-6-2018 c/d 167

Page 35 of 36

52
Accumulated Depreciation Account
1-7-2012 b/d -
30-6-2013 Depreciation 36
30-6-2013 c/d 36
1-7-2013 b/d 36
31-12-2013 Building 54 31-12-2013 Depreciation 18
31-12-2013 Depreciation 19
30-6-2014 c/d 19
1-7-2014 b/d 19
30-6-2015 Depreciation 38
30-6-2015 c/d 57
1-7-2015 b/d 57
31-12-2015 Building 76 31-12-2015 Depreciation 19
31-12-2015 Depreciation 16
30-6-2016 c/d 16
1-7-2016 b/d 16
30-6-2017 Depreciation 32
30-6-2017 c/d 48
1-7-2017 b/d 48
31-12-2017 Building 64 31-12-2017 Depreciation 16
30-6-2018 Depreciation 19
30-6-2018 c/d 19

Revaluation Surplus
1-7-2012 b/d -
30-6-2013 c/d -
1-7-2013 b/d -
30-6-2014 Retained Earnings 1 31-12-2013 Building 17
30-6-2014 c/d 16
1-7-2014 b/d 16
30-6-2015 Retained Earnings 2
30-6-2015 c/d 14
31-12-2015 Retained Earnings 1 1-7-2015 b/d 14

31-12-2015 Building 13
30-6-2016 c/d -
1-7-2016 b/d -
30-6-2017 c/d -
1-7-2017 b/d -
30-6-2018 Building 0.56 31-12-2017 Building 5
30-6-2018 c/d 4.44

Page 36 of 36

53
IAS 16:
When land is acquired, certain costs are necessary and should be part of the cost of land. These costs include the cost
of the land, title and legal fees, site preparation costs like grading and draining and survey costs etc. All of these
costs may be considered necessary to get the land ready for its intended use.
The cost of a self‑ constructed asset is determined using the same principles as for an acquired asset. Borrowing
costs including interest may also be capitalised in accordance with IAS 23.

AT A GLANCE
SPOTLIGHT

Example:
On 1 January 2021, M2 Limited purchased a building for Rs. 100 million with nil residual value
and 10 years useful life.
On 31 December 2021, the building was revalued to Rs. 108 million and a surplus of Rs. 18 million
was recognised. M2 Limited does not transfer any revaluation surplus to retained earnings on
annual basis.
On 31 December 2022, due to slump in the property market, the building was again revalued but
this time the worth was only Rs. 55 million
Required: Pass the journal entries for the year ended 31 December 2022.
⯈ Answer:
STICKY NOTES

Journal entries

Debit Credit
Date Particulars
Rs. m Rs. m
31 Dec 2022 Depreciation [108m / 9 years] 12
Accumulated depreciation 12
31 Dec 2022 Accumulated depreciation 12
PPE 12
Loss on revaluation (OCI) 18
Loss on revaluation (PL) [41 – 18] 23
PPE [55 – (108 – 12)] 41

54
Example:
On 1 January 2021, M3 Limited purchased a building for Rs. 100 million with nil residual value
and 10 years useful life.
On 31 December 2021, the building was revalued to Rs. 108 million and a surplus of Rs. 18 million
was recognised. M3 Limited transfers maximum possible revaluation surplus to retained
earnings on annual basis.
On 31 December 2022, due to slump in the property market, the building was again revalued but
this time the worth was only Rs. 55 million

SPOTLIGHT
Required: Pass the journal entries for the year ended 31 December 2022.
⯈ Answer:
Journal entries

Debit Credit
Date Particulars
Rs. m Rs. m
31 Dec 2022 Depreciation [108m / 9 years] 12
Accumulated depreciation 12
Revaluation surplus [12 – 10] or [18 / 9 years] 2
Retained earnings 2
31 Dec 2022 Accumulated depreciation 12
PPE 12
Loss on revaluation (OCI) [18 – 2] 16
Loss on revaluation (PL) [41 – 16] 25
PPE [55 – (108 – 12)] 41

Example:
Following information pertains to Rose Enterprises for the year ended 31 December 2017:
(i) Acquisition of land and construction of a factory building:
Rs. in '000
Cost of freehold land purchased with old building structure 25,000
Cost of demolition of the old building structure 1,500
Proceeds from sale of scrap of the old building 250
Fee paid to ABC Architects for site plan and drawings 800
AT A GLANCE

Advance paid to Quality Construction (QC) for construction of the 6,000


building
Further payment to QC 35,000
(ii) Acquisition and installation of new plant:
Rs. in '000
25% cost of the plant paid in advance 4,000
Transportation and import charges 1,250
Cost of installation 400

55
(iii) Other information:
• Cost of freehold land includes property tax for 2017-18 and transfer fee of Rs.
SPOTLIGHT

120,000 and Rs. 850,000 respectively.


• Factory building was available for use from 1 July 2017. The final invoice of Rs.
19,000,000 is still unpaid.
• Transportation and import charges of the plant include annual fire insurance
premium and insurance in-transit of Rs. 350,000 and Rs. 60,000 respectively.
• The plant started operations on 1 August 2017. Remaining amount was paid on
31 August 2017.
• Old plant was sold on 1 September 2017 at its written down value plus 20%. The
plant was purchased on 1 April 2015 at a cost of Rs. 8,500,000
• Building and plant are depreciated at the rate of 5% and 10% respectively on
STICKY NOTES

reducing balance method.


Required:
(a) Pass journal entry to record disposal of the old plant.
(b) Determine written down value of the fixed assets as at 31 December 2017.
⯈ Answer:
Part (a)

Debit Credit
Journal entry for disposal of old plant:
Rs.000 Rs. 000
Bank/Cash/Receivable (8,500–1,896) × 120% 7,925
Accumulated depreciation (W1) 1,896
Fixed assets (Plant) 8,500
Gain on disposal (Balancing figure) 1,321

W-1: Accumulated depreciation: Rs. ‘000


2015: Apr to Dec (8,500 x 10% x 9 / 12) 638
2016: (8,500 – 638) × 10% x 12 / 12 786
2017: Jan to Aug (8,500 – 638 – 786) × 10% × 8 / 12 472
1,896

56
Part (b)

Freehold Building Plant


land

AT A GLANCE
--- Rs. in '000 ---

Purchase price 25,000

Demolition of old building Rs. 1,500 – 250 1,250

Architect fee paid to ABC consultant 800

Construction cost – Advance 6,000

Construction cost – Further payment 35,000

Plant cost – advance 25% 4,000

Plant cost – remaining 75% 12,000

Transportation and import charges 1,250

Installation charges 400

Property tax year 2017-18 (should be PL) (120)

Transfer fee (correctly included already) -

SPOTLIGHT
Construction costs – unpaid amount 19,000

Annual fire insurance (should be expense) (350)

Insurance in transit (correct already) -

Cost to be capitalised 26,130 60,800 17,300

Depreciation for 2017

Land -

Building (60,800 × 5% × 6 / 12) (1,520)

STICKY NOTES
Plant (17,300 × 10% × 5 / 12) (721)

WDV of PPE as at 31 December 2017 26,130 59,280 16,579

57
58
Statement of changes in equity:
Components of financial statements
• Statement of financial position (balance sheet)
• Statement of comprehensive income (statement of profit or loss)
• Statement of changes in equity
• Statement of cash flow
• Notes to the financial statements.

Introduction of business in the form of company:


Company: An association of persons which is registered under the Companies Act 2017.
Shareholder: Person who owns shares in the company is called as shareholder.
Share certificate: Evidence of ownership in the company.

Shareholders elect directors to manage the affairs of the company on their behalf.
Capital of the company is divided into shares; therefore called as share capital.
Types of share capital:
Authorized Capital: Maximum number of shares that a company can issue (No accounting entry is made
for authorized share capital)
Issued Capital/Paid-up Capital: Actual number of shares in issue at any point of time.
Every share has a registered price called as face value, nominal value or par value. In Pakistan, this
registered price is normally Rs. 10 per share.
(I) Issue of shares at nominal value/face value/par value

100,000 shares of Rs. 10 each issued at Rs 10 each.


Bank 1,000,000
Share capital 1,000,000

(II) Issue of shares at a premium


100,000 shares of Rs. 10 each issued at Rs 27 each.
Bank 2,700,000
Share capital 1,000,000
Share premium 1,700,000
(III) Issue of shares at a discount
100,000 shares of Rs. 10 each issued at Rs 7 each.
Bank 700,000
Discount on shares 300,000
Share capital 1,000,000
Discount on issue of shares is adjusted from share premium and if not available then a separate column
of share discount as a negative figure in statement of changes in equity.
Right shares: Shares issued to existing shareholders in proportion to existing number of shares against
consideration (normally cash). Its entry will be same as discussed above.

Page 1 of 10

59
Illustration: Statement of financial position
XYZ Entity
Statement of financial position
As at 31 December 20XX

Rs. M Rs. M
Assets
Non-current assets
Property, plant and equipment 205.1
Intangible assets 10.7
Investments 6.8 222.6

Current assets
Inventories 17.8
Trade and other receivables 15.3
Cash and cash equivalents 0.7 33.8
Total assets 256.4

Rs. m Rs. m
Equity and liabilities
Equity:
Share capital 50.0
Share premium 31.9
Retained earnings (accumulated profits) 60.6 142.5

Non-current liabilities
Long-term borrowings 34.5 34.5
Current liabilities
Trade and other payables 67.1
Short-term borrowings (bank overdraft) 3.2
Current portion of long-term borrowing 5.0
Current tax payable 4.1 79.4
Total equity and liabilities 256.4

Example: statement of comprehensive income

XYZ Entity
Statement of comprehensive income
For the year ended 31 December 20XX
000
Revenue 678
Cost of sales (250)
Gross profit 428
Other income 12
Distribution costs (98)
Administrative expenses (61)
Other expenses (18)
Finance costs (24)

Page 2 of 10

60
Profit before tax 271
Taxation (50)
––––––
Profit after tax 221

Taxation: tax on profits of the current period is called as current tax. It is expense for the business. Its
accounting entries are:
Current tax xxx
Current tax payable xxx
(when accrued at the period end)
Current tax payable xxx
Cash/ bank xxx
(when paid)

Dividend: It is the distribution of profits to shareholders.


Dividend* xxx
Dividend payable xxx
(when declared)
Dividend payable xxx
Cash xxx
(when paid)
*Dividend is recorded when it is declared.

Dividend
Interim dividend Final dividend
Dividend on the basis of profits of less than one Dividend on the basis of profits of full year
year
Dividend is recorded when it is declared. If dividend is declared after the reporting period but before the
date of authorization of financial statements, then the dividend declaration is disclosed together with
the amount per share in the notes to the financial statements of the period to which it relates.
Preparation of financial statements is the responsibility of the management of the company.
Authorization of financial statements means directors have signed the financial statements after their
finalization.

Issue of Shares against Cash


Cash XX
*Share Capital XX

Difference can either be share premium or discount on issue of shares.


*Share capital and share premium are presented in equity.

Page 3 of 10

61
Dividend to Shareholders:
It is distribution of profits to shareholders.

a)
*Dividend XX
Dividend Payable XX
(When the dividend is declared)
*Dividend is not an expense instead it is deducted from retained earnings just like drawings (which are
deducted from capital).
b)
Dividend Payable XX
Cash/Bank XX
(When the dividend is paid)

Dividend

Shares
Cash

Interim Final
Interim Final

Bonus shares/ Stock dividend / Capitalization Issue


It is a situation in which company issues new shares to existing shareholders in proportion to existing
shares without any consideration.
Dividend xxxx
Share capital xxxx
Entry will be made at the amount of nominal value of shares. Therefore there will be no premium or
discount in this entry..
Summary of discussion about dividend
It is a distribution of profits to owners (shareholder). If dividend Is given as a percentage then multiply
the rate with the amount of share capital (nominal value of share capital) which is at the date of
declaration of dividend to calculate the amount of dividend. A company may pay dividend either in cash
or in form of shares.
Dividend
Cash Dividend (Either Final or Interim) Bonus Shares (Either Final or Interim)
When Declared Dividend XXX
Dividend(Retained Earnings) XXX Share Capital A/c XXX
Dividend Payable A/c XXX

When Paid
Dividend Payable A/c XXX
Cash/ Bank A/c XXX

Page 4 of 10

62
• Bonus shares are distributed from retained earnings and if no balance of retained earnings then
from share premium.
• Dividend is recognized on the date of declaration.
• If the dividend is declared after reporting date but before the authorization of the financial
statements it is disclosed in the financial statement to which it relates.
• If the date of declaration is not given, then:
1. For interim dividend assume that dividend is declared during the accounting period.
2. For final dividend assume declared after the reporting date.
• If nature of dividend (means whether final or interim) is not given then assume final dividend.
• If type of dividend (means cash or bonus) is not available then assume cash dividend.

Q.1 SCCL, a company listed on Pakistan stock exchange is in process of finalization of its accounts for the
year ended 31-12-2012. The following information is available

i. Shareholders equity as at 31-12-2011 and 2010 consist of


Rs in million

2011 2010

Share capital (Rs 10 each) 10,340 7,833

Unappropriated Profits 6,945 4,508

ii. Profit after tax for the years ended 31-12-2010, 2011 and 2012 was Rs 4,240 million, Rs 4,944
million and Rs 5,090 million respectively.

iii. Cash dividends and bonus declared/paid during the last three years were as follows:
Cash Dividend Bonus Shares

*Interim Final *Interim Final

For the year ended 31-12-2010 10% - - 20%

For the year ended 31-12-2011 - 15% 10% 10%

For the year ended 31-12-2012 - 10% 5% 5%

*Interim dividend/bonus was declared at time of announcement of half-yearly financial results.

iv. Right shares were issued on 30-11-2012 in the ratio of 4 right shares for every 5 shares held by
the shareholders of the company. The right issue was made at Rs 18/share.

Required: Prepare statement of changes in equity for the year ended 31-12-2012.
Note: If more than one dividends are declared on the same date then for both dividends base share
capital is same.

In the question of statement of changes in equity even if the question is silent prepare on year
comparative figures if there is information.

Page 5 of 10

63
Reserves: Reserves are profits retained in the business (not available for distribution to its shareholders
as dividend) to meet any future business requirements. There are two types of reserves:

1. Specific reserve (reserve created to meet any defined future business requirement e.g. any
future capital expenditure or against any court case).
2. General reserve (reserve created to meet any undefined future business requirement).
These reserves are also called as revenue reserves.
Accounting entry to create reserves:
Retained earnings xxx
Reserves xxx
(whether general or specific)

Transfer of incremental depreciation means amount of revaluation surplus transferred to retained


earnings.

Question-2: For the purpose of preparation of statement of changes in equity for the year ended 31
December 2017, Daffodil Limited (DL) has extracted the following information:
2017 2016 2015
Draft Audited Audited
Rs. in million
Net profit 650 318 214
Transfer to general reserves 112 - 141
Transfer of incremental - 49
depreciation 55
Final cash dividend - - 7.5%
Additional information:
i) Details of share issues:
o 25% right shares were issued on 1 May 2016 at Rs. 18 per share. The market price per
share immediately before the entitlement date was also Rs. 18 per share.
o A bonus issue of 10% was made on 1 April 2017 as final dividend for 2016.
o 50 million right shares were issued on 1 July 2017 at Rs. 15 per share. The market price
per share immediately before the entitlement date was Rs. 25 per share.
o A bonus issue of 15% was made on 1 September 2017 as interim dividend.
ii) Share capital and reserves as at 31 December:

2015 2014
------ Rs. in million ------
Ordinary share capital (Rs. 10 each) 1,600 1,600
General reserves 1,850 1,709
Retained earnings 1,430 1,302
Revaluation surplus 100 -

Required: Prepare DL’s statement of changes in equity for the year ended 31 December 2017 including
comparatives. (Ignore taxation)

Page 6 of 10

64
Q.3 Following information pertains to Astrazenca Limited (AL):

(i) Shareholders' equity as on 1 January 2020:

Rs. in million
Share capital (Rs. 100 each) 250
Share premium 138
Retained earnings 142
Revaluation surplus: Land 25
Buildings 20

(ii) Profit and transfer of incremental depreciation as per the draft financial
statements for the year ended 31 December 2020 amounted to Rs. 45 million
and Rs. 5 million respectively.

(iii) Dividends for the last two years:

For the year ended *Interim cash dividend Final bonus dividend
31 December 2019 10% 20%
31 December 2020 12% 15%

*Declared with half yearly accounts

(iv) AL uses revaluation model for subsequent measurement of its land and
buildings only. The revalued amounts of land and buildings have
been assessed at 31 December 2020 but not incorporated in draft financial
statements. The relevant details are as under:

Land Buildings
--- Rs. in million ---
Balances as on 31 December 2020 before revaluation:
Cost 75 240
Accumulated depreciation - 60
Revalued amounts assessed at 31 December 2020 65 158

Required:
Prepare AL’s statement of changes in equity for the year ended 31 December 2020. (08)

(Column for total and comparative figures are not required)

Page 7 of 10

65
Answers:
A.1
SCCL
Statement of Changes in Equity
For the Year ended 31-12-2012
Rs in millions
Share Share Unappropriated Total
Capital Premium Profit
Balance as on 1-1-2011 7,833 - 4,508 12,341

Final Bonus Shares-2010 (20%) 1,567 - (1,567) -


(7,833x20%)
Interim Bonus Shares-2011(10%) 940 - (940) -
(7,833+1,567)x10%
Profit for the year - - 4,944 4,944
Balance as on 31-12-2011 10,340 - 6,945 17,285

Final Bonus Shares-2011 (10%) 1034 (1034) -


(10,340x10%)
Final Cash Dividend-2011 (15%) (1,551) (1,551)
(10,340x15%)
Interim bonus Shares-2012(5%) 569 (569) -
(10,340+1034) x 5%
*Issue of Right Shares 9,554 7,643 - 17,197
Profit for the year (W) 5,090 5,090
Balance as on 31-12-2012 21,497 7,643 8,881 38,021

*Right share=(10,340+1034+569)=11,943÷10 x 4/5 = 955.4 m shares


Share capital = 955.4 x 10 = 9,554
Share Premium = 955.4 x 8 =7,643

Answer-2:
DAFFODIL LIMITED
Statement of changes in equity
For the year ended 31 December 2017
-------------------- Rs. in million --------------------
Share Share General Retained Revaluation Total
Capital Premium reserves Earnings Surplus
Balance as at 31
December 2015 1,600.00 1,850.00 1,430.0 100.00 4,980.0
Final cash dividend @
7.5% - 2015 (120.00) (120.00)
(1,600×7.5%)

Right issue @ 25% 400.00 320.00 720.00


1,600/10=160 x (40 x 10) (40 x 8)

Page 8 of 10

66
25%=40

Net profit – 2016 318 318


Transfer of surplus 49 (49) -
Balance as at 31
December 2016 - 2,000.00 320.00 1,850.00 1,677 51 5,898

Final bonus dividend


@ 10% - 200.00 (200.00) -
2016 (2,000×10%)

Right issue 500.00 250.00 750.00


(50×10) (50×5)

Interim bonus
dividend @ 15% - 405.00 (405) -
2017 (2,700×15%)

Net profit – 2017 650 650

Transfer to general
reserves 112.00 (112.00) -

Balance as at 31
December 2017 3,105.00 570 1,962.00 1,610 51 7,298

A.3 Astrazenca Limited


Statement of changes in equity
For the year ended 31 December 2020
Share Share Retained Revaluation
capital premium earnings surplus
--------------- Rs. in million ---------------
Balance as at 1 January 2020 (As given) 250 138 142 45
(25+20)
(250×20%
Final bonus dividend @ 20% for 2019 ) 50 (50)
Interim cash dividend @ 12% for (250+50)
2020 ×12 (36)
%)

Profit for the year 45–7 (W-1) 38


Revaluation (10+15)(W-1) (25)
Transfer of incremental depreciation 5 (5)
Balance as at 31 December 2020 300 138 99 15

Page 9 of 10

67
W-1: Revaluation of building Building Land
Rs. in million
Revalued amount 158 65
Carrying value (240–60) 180 75
Revaluation loss (22) (10)
Available surplus (20–5) 15 25
Revaluation loss taken to profit or loss 7 -

Page 10 of 10

68
SOCE:
EQUITY
Components of equity
Equity is defined as the residual interest in the assets of the entity after deducting all its liabilities. Equity is
presented in statement of financial position and consists of:
a) Share capital
• Ordinary share capital
• Preference share capital (irredeemable)
b) Capital (non-distributable) reserves
AT A GLANCE

• Share premium
• Revaluation surplus
c) Revenue (distributable) reserves
• Retained earnings
• General reserves
• Other specific reserves created out of retained earnings (Dividend equalization reserves)
(discussion next)

Dividend
A dividend is the distribution of a company's earnings (realised profits) to its shareholders and is determined by
SPOTLIGHT

the company's board of directors and approved by either board of directors or shareholders.

Share capital
The term “share capital” refers to the amount of money the owners of a company have invested in the business
as represented by ordinary and irredeemable preference shares.

Ordinary share capital


Owning ordinary shares entitles an investor to own a part of ownership in the company. The shareholders are
given voting rights, rights to attend the annual general meetings, dividends, and bonus shares from the company.
Most companies issue only this type of share capital.
STICKY NOTES

Preference share capital


The preference shareholders are given preference over ordinary shareholders while paying dividends and
repaying the amount of capital at the time of liquidation of the company.
Dividend payments for preference shareholders are often fixed and due to this preference shares are often seen
as a less risky investment, although payments are likely to be lesser than ordinary dividend when company is
performing well.
Preference shares are of two types: redeemable and irredeemable. Only irredeemable preference shares are
treated as equity share capital since redeemable preference shares are often classified as liability as entity has
obligation to settle the amount after a certain time.

Capital reserves
Capital reserves are reserves that are not regarded free for distribution by way of dividend. Two most common
capital reserves are share premium and revaluation surplus.

69
Share premium
The excess of issue price (i.e. the total amount a company received for shares) over par value of a company’s
shares is called “share premium”. For example, if a share having par value of Rs. 10 is issued for Rs, 12 then Rs. 2
is the share premium.
The Companies Act, 2017 (Section 81) prescribes that the share premium account may be used:
• To write-off the preliminary expenses;
• To write-off the expenses (commission, discount) of issue of shares;
• In providing for the premium payable on the redemption of any redeemable preference shares
• For issue of bonus shares

Revaluation surplus

AT A GLANCE
The result of an upward revaluation of a non-current asset is a 'revaluation surplus'. The amount accumulated in
revaluation surplus is non-distributable, as it represents unrealised profits on the revalued assets. The
revaluation surplus may diminish if an asset which had previously been revalued upwards is devalued later.
Revaluation surplus is transferred to retain earnings in case of disposal of asset and/or incremental depreciation.

Revenue reserves
Revenue reserve means reserve that is normally regarded as available for distribution including:
• retained earnings (also called un appropriated or accumulated profits);
• general reserves; and
• other specific reserves created out of profit (Dividend equalization reserves).

SPOTLIGHT
Retained earnings
Retained earnings comprise the earnings (profits and gains less expenses and losses) that the company retains
within the business, i.e. part of profits that has not been paid out as dividends or transferred to any other reserve.
A debit balance on the retained earnings account indicates that the company has accumulated losses.

General reserves
General reserve is the amount kept aside from the company’s profit during its normal operation to meet future
needs. i.e., contingencies, strengthening the company’s financial position, increasing working capital, paying
dividends to the shareholders, offsetting specific future losses, etc. Usually, a transfer to general reserves from
retained earnings is made with approval of board of directors.

STICKY NOTES
Dividend equalization reserve
Dividend equalization reserve is a specific reserve created out of retained earnings to ensure dividends remain
stable irrespective of changes in earnings.

70
SPOTLIGHT

Example:
Adeel Limited (AL) had following equity balances as on 1st January 2023:

Rs. m
Ordinary share capital (Rs. 100 each) 600
Retained earnings 15
STICKY NOTES

On 16 April 2023, AL made a 20% right issue at par value. Issue costs of Rs. 1 million were also
incurred.
Required: Journal entries.
⯈ Answer:
Journal entries

Debit Credit
Date Particulars
Rs. m Rs. m

16 Apr 2023 Bank [600 x 20%] 120

Share capital 120

16 Apr 2023 Retained earnings 1

Bank / accrual 1

Example:
Aqeel Limited (AL) had following equity balances as on 1st January 2023:

Rs. m
Ordinary share capital (Rs. 100 each) 600
Share premium 120
Retained earnings 85

On 16 August 2023, AL made a right issue of 1 for 4 shares already held at Rs. 150 per share.Issue
costs of Rs. 5 million were also incurred.
Required: Journal entries.

Answer:
Journal entries

Debit Credit
Date Particulars
Rs. m Rs. m
16 Aug 2023 Bank 225
Share capital [600 x ¼] 150
Share premium [150 / 100 x 50] 75
16 Aug 2023 Share premium 5
Bank / accrual 5

71
SPOTLIGHT
⯈ Example:
Arma Limited (AL) had following equity balances as on 1st January 2023:

Rs. m
Ordinary share capital (Rs. 100 each) 600
Share premium 60
Retained earnings (30)

On 11 December 2023, AL issued 300,000 shares at Rs. 90 each after obtaining necessary

STICKY NOTES
approvals from regulatory authorities. Issue costs of Rs. 2 million were also incurred.
Required: Journal entries.
⯈ Answer:
Journal entries

Debit Credit
Date Particulars
Rs. m Rs. m
11 Dec 2023 Bank [300,000 x Rs. 90] 27
Share premium [300,000 x 10] 3
Share capital 30
11 Dec 2023 Share premium 2
Bank / accrual 2

Example:
Z Limited (ZL) had following equity balances as on 1st January 2023:

Rs. m
Ordinary share capital (Rs. 100 each) 600
Irredeemable preference shares (Rs. 1000 each) 400
Share premium 300
Retained earnings 285

The preference shareholders are entitled to 12% cumulative dividend in arrears. However, no
dividend was declared or paid during the year.
Required: Journal entries for recognising dividend for the year ended 31 December 2023.

Answer: Journal entries

Debit Credit
Date Particulars
Rs. m Rs. m
31 Dec 2023 Dividend [Retained earnings] [400 x 12%] 48
Dividend payable 48

72
Example:
The following information relate to Multan Limited (ML):
1. ML disposed of a land on 3rd January 2023. This land had a carrying amount of Rs. 85
million (using revaluation model) and was purchased at a cost of Rs. 36 million few years ago.
2. ML revalued its motor vehicles on 1st January 2023 and a revaluation surplus of Rs. 50
million was recognised at this date. The motor vehicles had remaining useful life of 5
years on this date and are being depreciated using straight line method.
3. ML has determined its profit for the year ended 31 December 2023 at Rs. 75 million afterall
the necessary adjustments. The board of directors have approved a transfer 20% of profit
to general reserves.

AT A GLANCE
Required: Journal entries involving transfers between reserves for the year ended 31.12.2023.
Journal entries for disposal orgain (loss) on revaluation are not required.
Answer:
Journal entries

Debit Credit
Date Particulars
Rs. m Rs. m

3 Jan 2023 Revaluation surplus [85 – 36] 49

Retained earnings 49

SPOTLIGHT
31 Dec 2023 Revaluation surplus [50/5 years] 10

Retained earnings 10

31 Dec 2023 Retained earnings [75 x 20%] 15

General reserves 15

STICKY NOTES

73
Without Allah We are nothing. We surrender to the will of Allah

IAS 23 BORROWING COST


BASIC DISCUSSION

LONG TERM LOANS AND RELATED ISSUES:

A. A loan is obtained from MCB on 1-1-2012 against a sanctioned limit of 120 M. It carries markup @
12% p.a. This loan is repayable in semi-annual installments of Rs 5 M starting from 1-7-2012; along
with interest.

Required:
i.) Calculate the amount of interest expense for the year ended 30-6-2013
ii.) Prepare relevant extracts from statements of financial position as on 30-6-2013

Note
i) Sanctioned limit means maximum amount of loan agreed with the bank.
ii) Instalment of loan comprises of principal amount only unless there is any indication.

B. A loan is obtained from Citibank Limited on 1-1-2013 of Rs 5,000,000. It carries interest @ Rs 0.23
per thousand per day (means 8.395% p.a)

It is repayable in monthly instalments of Rs 100,000 starting from 1-4-2013, along with interest.

Required:
i.) Calculate the amount of interest expense for the year ended 30-6-2013
ii.) Prepare relevant extracts from statements of financial position as on 30-6-2013

C. A loan is obtained on 1-8-2012 for Rs 25 M. It carries markup @ 15% p.a. It is repayable in quarterly
instalments of Rs 5 M starting from 1-11-2012; along with interest.

Required:
i.) Calculate the amount of interest expense for the year ended 30-6-2013
ii.) Prepare relevant extracts from statements of financial position as on 30-6-2013

Page 1 of 40

74
Example A
If we want to calculate interest expense; then identify the following figures:

Outstanding Balances of loan x Rate x Time period

(It means balance which remained constant for a given period of time)

i) Calculation of interest expense:


Figures in millions

115 M x 12% x 6/12 (July to Dec) = 6.9


110M x 12% x 6/12 (Jan to June) = 6.6
13.5

Workings
1-1-2012 Loan Obtained Rs 120

30-6-2012 Closing Balance 120


1-7-2012 Repayment of Loan (5)
115
1-1-2013 Repayment of Loan (5)
110

30-6-2013 Closing Balance 110


1-7-2013 Repayment of Loan 5
1-1-2014 Repayment of Loan 5
Current Portion 10
Non-current Portion 100
ii)

Extracts from statement of Financial Positions


As on 30-6-2013
2013
Equity & Liabilities
Non-current liabilities
Long term Loan 100

Current Liabilities
Current Portion of Loan 10
Accrued Interest 6.6

Note:
In a question in which we are preparing an extract of statement of financial position or statement
of comprehensive income then there is no need of any total.

Page 2 of 40

75
May Allah guide us and forgive us for hurting other ones knowingly or unknowingly

Example B

i) Calculation of interest Expense


5,000,000 x 8.395% x 3/12 = 104,938
4,900,000 x 8.395% x 1/12 = 34,280
4,800,000 x 8.935% x 1/12 = 33,580
4,700,000 x 8.935% x 1/12 = 32,880
205,678

Workings
1-1-2013 Loan Obtained 5,000,000
1-4-2013 Loan repaid (100,000)
4,900,000

1-5-2013 Loan repaid (100,000)


4,800,000

1-6-2013 Loan repaid (100,000)


4,700,000
30-6-2013 Closing Balance 4,700,000

Current portion of loan 1,200,000


(100,000 x 12)
Non-current Portion 3,500,000

ii)
Extracts from statement of Financial Position as on 30-6-2013
2013
Non-current Liabilities
Long term loan 3,500,000

Current Liabilities
Current Portion of Loan 1,200,000
Accrued Interest 32,880

Example C
i) Calculation of Interest expense for the period
Figures in millions

25M x 15% x 3/12 Aug to Oct = 0.9375


20M x 15% x 3/12 Nov to Jan = 0.75
15M x 15% x 3/12 Feb to April = 0.5625
10M x 15% x 2/12 May & June = 0.25
2.5

Page 3 of 40

76
Workings
1-8-2012 Loan Obtained Rs 25
1-11-2012 Repayment made (5)
20

1-2-2013 Repayment made (5)


15

1-5-2013 Repayment made (5)


10

30-6-2013 Closing Balance 10

Current portion (5M x 2) 10

Non-current Portion Nil

ii) Extracts from statement of Financial Position as on 30-6-2013


2013
Non-current Liabilities
Long term loan --

Current Liabilities
Current Portion of Loan 10
Accrued Interest 0.25

Page 4 of 40

77
Quran takes us out of darkness and puts us into the light so don’t forget to read Quran.

Borrowing Cost (IAS-23)

Borrowing Costs
Interest and other related costs that an entity incurs in connection with borrowing of funds.
Other related costs includes bank charges deducted by bank at the time of disbursement of loan.
 Different names used for bank charges in exam questions.
• Arrangement fee
• Loan processing charges
• Commitment fee

 Different types of Loans/Borrowings:


• Long term Loans
• Short term Loans
• Short term running finance/Bank overdraft.

 Some important accounting entries


Issue of Shares against Cash
Cash XX
*Share Capital XX

*Share capital is presented in equity; it is not a borrowing.

Dividend to Shareholders:
It is distribution of profits to shareholders.

a)

*Dividend XX
Dividend Payable XX
(When the dividend is declared)

*Dividend is not an expense instead it is deducted from retained earnings just like drawings (which are
deducted from capital).
b)

Dividend Payable XX
Cash/Bank XX
(When the dividend is paid)
 As the share capital is not borrowing therefore dividend to shareholders is not a borrowing cost.
Treatment of borrowing cost: [Para 8]
Borrowing cost is recognized as an expense, when incurred;
Interest Expense XX
Interest expense payable XX
Other common words used for interest is finance cost or finance charge.
Unless; borrowing cost related to a qualifying asset. In that case borrowing cost is Capitalized (added) to
the cost of a qualifying asset.

Page 5 of 40

78
Qualifying Asset:
An asset which takes substantial time period to get ready for its intended use or sale.
 Asset for use
• Tangible Assets (IAS-16)
• Intangible Asset (IAS-38)

 Assets for sale


• Inventories (IAS-2)
Examples of qualifying assets:
Depending upon the circumstances, any of the following may be a qualifying asset:
a) Construction of factory building/Head office building
b) Purchase and installation of plant & machinery especially if imported.
c) Power generation facilities.
→ Assets that are ready for their intended use or sale when acquired are not qualifying assets e.g
furniture, computers, vehicles etc)
→ Similarly inventories that are manufactured over a short period of time are also not a qualifying
asset.
d) Investment property.
General Rule of Capitalization:
All amounts incurred to bring an asset into working condition as intended by management are Capitalized
(added to cost of an asset).
Conclusion:
Borrowing cost is recognized as an expense when incurred unless it relates to a qualifying asset. In that
case, it is capitalized.

Types of Borrowings with respect to Capitalization of borrowing cost:

Specific Borrowings General Borrowings

Specific Borrowings
Borrowings which are taken for the sole purpose of construction, acquisition or production of qualifying
asset.
Amount of borrowing cost to be Capitalized:
In case of specific borrowings, the amount to be capitalized is calculated as follows:

Amount of interest incurred on outstanding balance of loan Xxxx


Less: investment income (if any) on temporary investment of surplus funds (xxx)
Amount of interest to be capitalized Xxx

Page 6 of 40

79
Allah’s plans are better than our dreams

Commencement of Capitalization
As per IAS-23, entity shall commence capitalization of borrowing cost only; when
(i) Entity incurs borrowing cost; and
(ii) Entity incurs expenditures on qualifying asset and activities necessary to prepare the asset have
started.
For example
a) A loan obtained on 1-8-2014
Construction started on 1-9-2014
 Commencement date should be 1-9-2014

b) Construction started on 1-7-2014 with owned funds.


On 15-12-2014 entity obtained a loan for further expenditures on qualifying asset.
 Commencement date should be 15-12-2014
Cessation of Capitalization
Entity shall cease capitalizing borrowing costs when the asset is substantially complete i.e. when the
asset is available for its intended use or sale.

Important points to remember


a) If a non current asset is under process of construction or installation then it is called as Capital Work
in Progress (CWIP). If that CWIP is being prepared by borrowings then it is also called as qualifying
asset.
b) While calculating investment income to be deducted from interest incurred, we will consider only that
time period for which interest incurred is capitalized.

Q1. Money Limited began the construction of a new building on the 1 February 2015. Construction
costs incurred in 2015 were paid for as follows:
Rupees
On 1 February 500,000
On 1 July 600,000
On 1 November 800,000

The construction of the building ended on the 1 December 2015 when the building was complete and
ready for its intended use. This building is to be depreciated over 10 years to a nil residual value using the
straight-line method.

The construction was financed by a loan of Rs. 1,900,000 from Citi Bank Limited. The loan was raised on
1 January 2015 specifically to facilitate the construction of the building. The interest rate is 25% per
annum. There were no capital repayments during the year. Surplus funds were invested at 20% per
annum.

The building is a qualifying asset for the purposes of IAS-23.

Required:
a) Calculate the amount of borrowing costs that are eligible for capitalization during the year ended 31
December 2015.
b) Calculate the depreciation for the year ended 31 December 2015.
c) Calculate the carrying amount of the buildings as at 31 December 2015.
d) Calculate the amount of interest income to be recognized in statement of profit or loss.

Page 7 of 40

80
Q2. Loans raised specifically to fund the construction of a building (a qualifying asset} :
Loan % Raised on Rs
A 10% 1 January 2005 500,000
B 15% 1 June 2005 400,000

Rs 100,000 of the loan B capital was repaid on 31 July 2005. No other loan capital was repaid.

The only interest income earned during the year was interest income earned on the investment of surplus
funds from the specific loans in a 6% interest account.

Construction costs paid for as follows:


Date Rs
1st March 2005 300,000
30th April 2005 100,000
31st July 2005 220,000

Commencement date: 1 March 2005


Cessation date: 31 August 2005.

Required:
a) Calculate the amount of borrowing costs that must be capitalized in terms of IAS-23.
b) Calculate carrying amount of asset as on 31-12-2005, assuming that useful life is 5 years.

General Borrowings:

If a loan is for more than one purpose including a qualifying asset; then loan is called as General Purpose
Borrowing. These funds may be utilized for buying inventories, paying off expenses and multitude of other
purposes in addition to acquisition, construction or production of a qualifying asset.
E.g A loan from MCB for Rs 20M @ 12%.

Suppose:
(i) 5M used for factory expenses.
(ii) 2M used for vehicles; and
(iii) 10M used for construction of H.O building (a qualifying asset).

 In such a case all interest incurred on the loan is not related to this qualifying asset.
 In case of General Borrowings, amount of borrowing cost to be capitalized is calculated as follows;

Expenditure on qualifying asset x capitalization rate x time period

Capitalization Rate:
(i) If there is only one general loan then its rate is called as Capitalization Rate.
(ii) If there is more than one general loan having different interest rates, then we calculate an average
rate of interest called as capitalization rate as follows:

borrowing cost incurred


Capitalization rate = × 100
weighted Borrowing outstanding

Q3. MCQ (Private) Limited has the following general loans outstanding as at December 31 2015.
Rs.
Loan -1 @ 6% 300,000
Loan -2 @8% 200,000
Loan -3 @9% 150,000

Page 8 of 40

81
Send blessings on the Messenger of Allah, (Peace Be Upon him) and seek forgiveness from Allah.

All the three loans were brought forward from previous year. Neither loan is taken during the year nor is
repaid.

The company spent following amounts on construction of an asset.


January 31,2015 70,000
April,01,2015 80,000
December,01,2015 10,000
The asset is not yet available for use as on 31-12-2005

Required:
a) Calculate Capitalization rate
b) Calculate Borrowing cost eligible for capitalization.
c) Calculate the carrying amount of asset as at 31-12-2005
d) Calculate the amount of borrowing cost to be charged to statement of profit or loss as an expense
during the year ended 31-12-2005

Page 9 of 40

82
Q4. Sublime Sports Limited is currently manufacturing its power plants. Up to December 31, 2013, the
company has incurred costs totaling Rs. 500,000 on construction of one of its plants.

The following general loans are outstanding:


Rs.
Loan from MCB @ 9% 500,000
Loan from HBL @ 10% 625,000
Loan from UBL @11% 375,000

Loan from HBL was taken on July 1, 2013 while other loans were brought forward from previous year.
Expenditures on plant were incurred as follows:
May 31, 2013 Rs. 300,000
July 31,2013 Rs. 200,000
Plant is not yet available for use as on Dec 31, 2013

Required:
You are required to calculate:
a) Capitalization rate of the company;
b) Total borrowing cost to be capitalized for the year 2013.
c) Calculate the carrying amount as on 31 Dec, 2013.
d) Calculate the borrowing cost to be charged to statement of profit or loss.

Q5. Soccer Limited began the construction of a new stadium on the 1 January 2015. Details of the
progress payments made during 2015 are as follows:
Rupees
On 1 January 300,000
On 1 April 200,000
On 1 July 250,000
On 1 September 150,000
On 1 October 200,000

The stadium was still under construction at 31 December 2015.

The construction was financed by general borrowings within the company. General loans outstanding at
any one time during 2015 averaged Rs. 20,000,000. The interest expense incurred on these loans during
2015 was Rs 2,600,000.

The stadium is a qualifying asset as defined by IAS-23.

Required:
a) Calculate the amount of borrowing costs that are eligible for capitalization to the stadium during the
year ended 31 December 2015.
b) Calculate the depreciation for the year ended 31 December 2015.
c) Calculate the carrying amount of the stadium as at 31 December 2015.
 Progress Payments means payments made to contractor by customers as the work is being
completed.

Note: In case of general borrowings, interest income is not to be considered while calculating the
borrowing cost to be capitalized.

Note: the amount of borrowing cost capitalized cannot exceed the amount of borrowing cost it incurred
during the period.

Page 10 of 40

83
May Allah accept our dua, forgive us and make us wise in doing good deeds and our daily works.

Important Journal Entries

Company Investors

Company issued Shares:

Bank XX Investment in Shares XX


Share Capital XX Bank XX

Company issued Debentures (It is simply a


loan)
Bank XX Investment in Debentures XX
Debentures XX Bank XX

(Normally it is a non-current loan) (Normally it is a Non-current investment


in assets)

Q6. An entity constructs a factory building which is a qualifying asset. Funds are provided from
entity's general borrowings. Capitalization of borrowing cost commences on 1-7-2009 and
continues throughout the year until the expected completion on 31-12-2010.

The detail of expenditures on the qualifying asset is as follows:


1 July 2009 2,050,000
15th September 2009 1,000,000
15th April 2010 500,000
30 June 2010 400,000

The entity's general borrowings in the period from 1st July 2009 to 30th June 2010 consist of the
followings:
1) Rs. 5 million Debentures issued on 1-1-2009 with a fixed interest rate of 10%. The full amount of
debentures is payable in 2015.
2) Bank loan of 2 million taken on 1-7-2009 with a floating interest rate that is adjusted semi- annually by
changes in KIBOR. At 1-7-2009 interest rate was set at 8.75% which increased to 9.00% at 1 January
2010. The loan is paid off in half-yearly installments of Rs. 200,000.
3) Bank overdraft with a floating interest rate that is adjusted by changes in KIBOR. Interest rate at 1-7-
2009 is 14% which was adjusted to 14.25% at 1 Jan 2010. The overdraft at 1 July 2009 was 300,000
which increased to 700,000 at 1 Jan 2010, which remained constant till the year end.

Required:
Calculate the carrying amount of the factory building as on 30-06-2010 in accordance with relevant IFRS.

KIBOR: Karachi Inter Bank offered rate (It is an interest rate market).

Suspension of Capitalization

Sometimes after commencement of capitalization and before cessation of capitalization, there may be a
temporary stoppage of active development. In that case according to IAS-23:

Page 11 of 40

84
An entity shall suspend capitalization of borrowing costs during extended periods of time in which it
suspends active development of a qualifying asset unless temporary delay is a necessary part of the
process of getting the asset ready for its intended use or sale.

Example: Delays in Construction (Suspension Period)


A hotel was under construction in 2005. Borrowing cost of Rs. 300,000 is incurred on a loan during 2005.
The loan was specifically raised in January 1, 2005 for the sole purpose of construction of hotel.

Required:
Discuss how much of borrowing cost may be capitalized in the following two independent scenarios
assuming that:
a) Workers went on a strike for a period of two months during which no progress was made.
b) The builder of the hotel had to wait for five days for cement in the foundation to dry.

Solution:
a) During these two months, the interest incurred may not be capitalized to the asset as it is an
unnecessary interruption to the construction process.
b) The borrowing costs must still be capitalized as it is merely a temporary delay and is a normal part of
the construction process.

Q7. Hockey Limited borrowed 2,000,000 (at an interest rate of 14%) from the Bank on 1 January
2015. These funds have been borrowed in order to build a hockey stadium.

Progress payments made in 2015 are as follows:

Rupees
On 1 January 600,000
On 1 July 1,200,000
On 1 September 200,000

The surplus funds were invested in a fixed deposit earning interest at 10% per annum.

Construction began on 1 January 2015 and was still incomplete on 31 December 2015. Between 1 June
and 20 June, construction suspended while concrete cured (a necessary part of the construction
process).

The stadium is a qualifying asset as defined by IAS-23.

Required:
a) Calculate the amount of borrowing costs that may be capitalized to the hockey stadium cost account
in the year ended 31 December 2015.
b) Calculate the amount of borrowing costs that may be capitalized to the hockey stadium cost account
in the year ended 31 December 2015 assuming that construction could not begin due to building
plans not meeting municipal standards. The plans have been resubmitted and it is expected that the
municipality will give the go-ahead to begin construction in early 2016.

Expenditure on Qualifying Asset:


If an entity has funds available from its owned resources, specific borrowings and general borrowings at
any given date then assume that:
• First; own funds are utilized
• Second; funds from specific borrowing ;and
• Then funds from general borrowings

Page 12 of 40

85
O Muslims, Fear Allah and don’t deal with riba (usury) or interest.

Repayment of Principal and Interest of Specific Borrowing:


If an entity has funds available from:
a) Owned resources
b) Specific borrowings; and
c) General Borrowings

At the date of repayment of principal and interest then assume that funds were utilized in above order.

Q8. On 1-3-2010, granite corporation (GC) started the construction of a new plant to meet the growing
demand for its products. The new plant was completed on 31 may 2011.

GC financed the cost of the project from the following sources:


i. On 1-3-2010, a 7 year loan of 70 million was obtained specifically for the construction of the
plant. The loan carried markup @ 13% p.a. payable semi-annually. An arrangement fee @
1% of the loan amount was paid to the bank. Two installments each comprising of repayment
of principal of 5 million along with interest were paid on 31-08-2010 and 28-2-2011.
ii. GC also has a running finance facility of RS 100 million carrying markup @14% p.a. Any
additional amount required for the project was provided through this general purpose facility.
iii. Surplus fund were invested in saving accounts @ 8% P.a.

Payments made to contractor were as follows:

Payment date Rs in million


1-3-2010 25
31-1-2011 65
30-09-2011 10

The construction work was suspended from 1-02-2011 to 28-02-2011. The suspension was caused due
to delay in shipment of essential component for the installation of plant.

Required:
Calculate the amount of borrowing cost that may be capitalized during the year ended 30-06-2010 and
30-06-2011.

Q9. Amjad Ltd started construction of a factory building on 1st April 2011. Construction took seven
months to complete. A contractor was hired for this work and total contract price was agreed to
be Rs 20 million.

Progress payment were made as follows:


1st April 10%
31st July 45%
30th September 15%
31st October 30%

Construction was financed partly by own savings and partly by loans. First payment to contractor was
made out of own funds. Afterwards all payments were made from borrowed funds.
A loan of Rs 8 million was obtained on 31st May 2011 specifically for this construction. This loan carried a
markup of 18%.Loan is to be repaid by the end of 2012.Unutilized portion of loan was kept in a deposit
account yielding an interest rate 4%.

Page 13 of 40

86
Any shortage of funds was fulfilled from existing pool of loans:
Bank Loan Amount Interest
Rs
Aay 50,000,000 10%
Bee 25,000,000 14.5%
Cee 10,000,000 15%

All these loans remained outstanding throughout the year.

Factory building has useful life of 10 years. After completion of construction, due to a delay in purchase of
furniture, building was brought into use from 1st January 2012.

Required:
a) Calculate the amount of borrowing cost eligible for capitalization.
b) Calculate the carrying amount of building as on 31-12-2011.
c) Calculate the amount of borrowing cost to be recognized in statement of profit or lossin the year
ended 31-12-2011.

Note: while calculating the capitalization rate, only consider the outstanding balance of loans during the
period. It means commencement date, cease date and suspension period is not considered while
calculating the rate.

Page 14 of 40

87
Don’t be Muslim by just name but be Muslim by practicing Islam.

Some important Terminologies used in exam questions:


Progress Billing: Bills given by contractor to customer as the work is performed and complete.
Retention Money: An amount deducted by the customer from progress billings as a security against
satisfactory completion of work. It is treated as a liability unless refunded/adjusted.
Right Shares: Shares issued to existing shareholders against consideration (which is normally cash).
First time issue of share is not called as right shares. Its accounting entry is exactly similar to entry of
share capital.

Any funds from shareholders are treated as owned funds in a question of borrowing cost.

Q10. On July 1, 2009, Qureshi Steel Limited (QSL) signed an agreement with Pak Construction Limited
for construction of a factory building at a cost of Rs. 100 million. It was agreed that the factory
would be ready for use from January 1, 2011. The terms of payments were agreed as under:
1) 10% advance payment would be made on signing of the agreement. The advance paid would
be adjusted at 10% of the quarterly progress bills.
2) 5% retention money would also be deducted from the progress bills. Retention money will be
refunded one year after completion of the factory building.
3) Progress bills will be raised on last day of each quarter and settled on 15th of the next month.

The under mentioned progress bills were received and settled by QSL as per the agreement:
Invoice date Amount (Rs.)
September 30, 2009 30 million
December 31, 2009 20 million
March 31, 2010 10 million
June 30, 2010 15 million

On April 30, 2010 an invoice of Rs. 1.5 million was raised by the contractor for damages sustained at the
site, on account of rains. After negotiations, QSL finally agreed to make additional payment of Rs. 1.0
million to compensate the contractor. The amount was paid on May 15, 2010. It is expected that 75% of
the payment would be recovered from the insurance company.

The cost of the project has been financed through the following sources:
1) Issue of right shares amounting to Rs. 15 million, on September 1, 2009. The company has been
following a policy of paying dividend of 20% for the past many years.
2) Bank loan of Rs. 25 million obtained on December 1, 2009. The loan carries a markup of 13% per
annum. The principal is repayable in 5 half yearly equal installments of Rs.5 million each along with
the interest, commencing from May 31, 2010. Loan processing charges of Rs.0.5 million were
deducted by the bank at the time of disbursement of loan. Surplus funds, when available, were
invested in short term deposits at 8% per annum.
3) Cash withdrawals from the existing running finance facility provided by a bank. Average running
finance balance for the year was Rs. 60 million. Markup charged by the bank for the year was Rs. 9
million.

Required:
a) Compute cost of capital work in progress for the factory building as of June 30, 2010 in accordance
with the requirements of relevant IFRSs.
(Borrowing costs calculations should be based on number of months)
b) Prepare extracts from statement of Financial Position as on 30-6-2010

Page 15 of 40

88
Q11. On January 1, 2015, Imran Limited started the construction of its new factory. The construction
period is approximately 15 months and the cost is estimated at Rs. 80 million. The work has been
divided into 5 phases and payment to contractor shall be made on completion of each phase.

In the year the company had the following sources of finance available.
(i) Rights i s s u e o f shares amounting to Rs. 15 million on January 1, 2015. The company
usually pays a dividend of 10% each year.
(ii) Bank loan of Rs. 32 million carrying a mark-up of 13% was raised on March 1, 2015. (This
loan was outstanding for 306 days in the year).
(iii) On August 1, 2015, Rs. 10 million were borrowed from the bank. Interest thereon, is payable
at the rate of 11%. (This loan was outstanding for 153 days in the year).
Investment income on temporary investment of the borrowings amounted to Rs. 0.5 million.
The details of bills submitted by the contractor, during the year are as follows:
Particulars Date of payment Rupees

On completion of 1st phase March 1, 2015 20,000,000


On completion of 2nd phase April 1,2015 18,000,000
On completion of 3rd phase October 1, 2015 16,000,000
On completion of 4th phase Payment not yet made 17,000,000

On June 1, 2015, the Building Control Authority issued instructions for stoppage of work on account of
certain discrepancies in the completion plan. The company filed a petition in the Court and the matter was
decided in the company’s favour on July 31, 2015. Work recommenced after a delay of 61 days.
The following periods may be relevant:
Period Days
March 1 to December 31 306
April 1 to December 31 275
August 1 to December 31 153
October 1 to December 31 92

Required
a) Assuming that the loans were taken specifically for the project, calculate the amount of borrowing
costs that s h o u l d be capitalised in the p e r i o d e n d i n g December 31, 2015 in accordance with
the requirements of IAS 23 Borrowing Costs.
b) Assuming that the loans constituted general finance, calculate the amount of borrowing costs that
s h o u l d be capitalised in the p e r i o d e n d i n g December 31, 2015 in accordance with the
requirements of IAS 23 Borrowing Costs.
(Borrowing cost calculations should be based on number of days)

Disclosures
a) Amount of borrowing Cost capitalized during the period
b) Capitalization rate used in case of general borrowings is to determine the amount of borrowing cost
eligible for capitalization.

Page 16 of 40

89
Invite people to Islam even without words, by just your behavior of kindness and good deeds.

Self-Testing Questions
Q1. On September 1, 2008, Spin Industries Limited (SIL) started construction of its new office building
and completed it on May 31, 2009. The payments made to the contractor were as follows:

Date of Payment Rupees


September 1,2008 10,000,000
December 1,2008 15,000,000
February 1,2009 12,000,000
June 1,2009 9,000,000

In addition to the above payments, SIL paid a fee of Rs. 8 million on September 1, 2008 for obtaining a
permit allowing the construction of the building.

The project was financed through the following sources:


i. On August 1, 2008 a medium term loan of Rs. 25 million was obtained specifically for the construction
of the building. The loan carried mark up of 12% per annum payable semi-annually. A commitment
fee @ 0.5% of the amount of loan was charged by the bank.
Surplus funds were invested in savings account @ 8% per annum. On February 1, 2009 SIL paid the
six monthly Interest plus Rs. 5 million towards the principal.
ii. Existing running finance facilities of SIL
a. Running finance facility of Rs. 28 million from Bank A carrying mark up of 13% payable annually.
The average outstanding balance during the period of construction was Rs. 25 million.
b. Running finance facility of Rs. 25 million from Bank B. The mark up accrued during the period of
construction was Rs. 3 million and the average running finance balance during that period was
Rs. 20 million.

Required:
Calculate the amount of borrowing costs to be capitalized on June 30, 2009 in accordance with the
requirements of International Accounting Standards. (Borrowing cost calculations should be based on
number of months).

Page 17 of 40

90
Q2. On, 01 January 2012, Marvelous Engineering Limited (MEL) started construction of its new
factory. The construction work was completed on 30 November 2012. The payment made to the
contractor as follows:
Date of payment Rs. in million
01-Jan-12 100
01-Apr-12 310
15-Dec-12 90

The construction work was financed through the following sources:


Date Description Rs. in million
01-Jan-12 12% Redeemable preference shares 150
01-Apr-12 14% TFCs for four years 300
01-Jul-12 Issue of right shares (estimated return is 22%) 50

The following additional information is also available:


i. The preference shares would be redeemed on 31 December 2016.
ii. Surplus funds were invested in a savings scheme @ 9% per annum.
iii. Due to delay in supply of construction material, the construction work was suspended from, 01 June
2012 to 30 June 2012.

Required:
Calculate the amount of borrowing costs that may be capitalized during the year ended 31 December
2012 in accordance with the requirements of International Financial Reporting Standards. (Assume that
calculations of borrowing costs are based on number of months)

Note: if nothing is mentioned then loans are assumed as specific.

Page 18 of 40

91
Follow the teaching of Quran and Sunnah and be friend of righteous people.

Q.3. A socially responsible multinational corporation (MNC) decided to construct a tunnel that will link
two sides of the village that were separated by a natural disaster years ago. Realizing its role as a
good corporate citizen, the MNC has been in this village for a couple of years exploring oil and
gas in the nearby offshore area. The tunnel would take two years to build. It began construction
on January 1, 2014. The following payments were made during 2014:
Rs 000
January 31 200
March 31 450
June 30 100
October 31 200
November 30 250
The first payment on January 31 was funded from the entity's general borrowings. However, the entity
succeeded in raising a medium-term loan for an amount of Rs 800,000 on March 31, 2014, with simple
interest of 9% per year. These funds were specifically used for this construction. Excess funds were
temporarily invested at 6 percent per year. The general borrowings was again used for an Rs 200,000
payment on Nov 30, which could not be funded from the medium-term loan. The construction project was
temporarily halted for six weeks in May, June, when substantial technical and administrative work was
carried out which was necessary stoppage for the completion of construction work.. The following
amounts of debt were outstanding at the year end, December 31, 2014:
Rs
Medium-term loan (see description above) 800,000
Bank overdraft (The weighted average amount outstanding during the year was Rs 1,200,000
750,000, and interest charged thereon by the bank amounted to Rs 33,800.)
A 10%, 7-year loan taken on October 1, 2007, repayable in full in December 2015 9,000,000
Required:
Calculate the amount of borrowing cost to be capitalized to the cost price of the Tunnel in 2014? (10
marks)
Question No.4
NRA Limited started construction of building on 01 January 2010. Building was completed in two years’
time with cost of Rs. 30 million. Company financed this construction mainly from existing running finance
facility which carries mark up at 14% per annum. Running finance limit is 100 million.
NRA Limited obtained a loan of Rs. 10 million at 12% per annum for this building on 1 December 2010
but due to some breach of contract with bank it had to return the total amount of loan with interest on 01
June 2011. Surplus funds were invested in a saving account @ 14%.
Construction activities remained suspended for the month of August 2011 due to shortage of material.
Following is the detail of payments made to contractor for the construction of building:

Date Amount
01-March-2010 3,200,000
30-June-2010 4,400,000
16-Sep-2010 2,300,000
01Dec-2010 6,200,000
31-jan-2011 5,500,000
01-May-2011 4,400,000
01-Oct-2011 2,500,000

Page 19 of 40

92
01-Dec-2011 1,500,000

Required:
Calculate the cost of building at 31 Dec., 2010 and 31 Dec., 2011 after capitalizing borrowing cost where
applicable. (Marks 15)

Page 20 of 40

93
Solutions
A1.

Money Limited

a) Borrowing costs to be capitalized


Interest incurred during construction:
1,900,000 x 25% x 10 / 12 395,833
Interest earned
(1 900,000 – 500,000) x 20% x 5/12 116,667
(1 900,000 – 500,000 – 600,000) x 20% x 4/12 53,333
170,000
Borrowing cost to be capitalized 225,833

b) Depreciation in 20X5

Construction costs 500,000 + 600,000 + 800,000 1,900,000


Borrowing costs capitalized Part (a) 225,833
2,125,833
Depreciation:
= 2,125,833 / 10 years x 1 / 12 months = 17,715

c) Carrying amount at 31 December 20X5


Cost 2,125,833
Accumulated depreciation Part (b) (17,715)
Carrying amount 2,108,118

d) Interest income to be recognized in income statement

1,900,000 x 20% x 1/12 31,667 (Related to Jan 2005)


=

 Ledgers not required in question. For Additional Information


Loan A/c Bank A/c
1-Jan Bank 1,900,000 1-Jan 1,900,000 1-Feb 500,000
Loan CWIP
1-July 600,000
CWIP
C/D 1,900,000 1-Nov 800,000
CWIP
1,900,000 1,900,000 1,900,000 1,900,000

A2.

a) Borrowing Cost to be Capitalized


Loan A
1-3 to 31-8 500,000 x 10% x 6/12 = 25,000

Loan B
1-June to 31-7 400,000 x 15% x 2/12 = 10,000
1-August to 31-8 300,000 x 15% x 1/12 = 3,750
38,750

Page 21 of 40

94
Less: Investment Income
Loan A Funds 1-1 500,000
Payment made 1-3 (300,000)
200,000 x6%x2/12 = 2,000
Payment made 30-4 (100,000)
100,000 x6%x1/12 = 500
Loan B Funds 1-6 400,000
500,000 x6%x2/12 = 5,000
Payment made 31-7 (220,000)
Repayment of Loan 31-7 (100,000)
180,000 X6%x1/12 = 900
(8,400)

Amount of borrowing cost to be capitalized 30,350

b) Carrying Amount of Building as on 31-12-2005:


Construction Cost = 620,000
(300,000 + 100,000 + 220,000)
Borrowing Cost Capitalized = 30,350
Total Cost of Building = 650,350

Depreciation
650,350 x 4/12 = 43,357
5
Carrying Amount as on 31 Dec,2005 is 650,350 – 43,357=606,993

A3. a) Calculation of Capitalization Rate


Rupees
Loan 1 300,000 X 6% 18,000
Loan 2 200,000 X 8% 16,000
Loan 3 150,000 x 9% 13,500
650,000 47,500

47,500 x 100 = 7.31%


650,000
b) Borrowing cost to be capitalized;
Loan Capitalization rate Months Rupees
January 31, 20X5 70,000 7.31% 11/12 4,691
April,01,20X5 80,000 7.31% 9/12 4,386
December,01, 20X5 10,000 7.31% 1/12 61
9,138

c) Carrying amount: Amount at which asset or liability is presented in statement of financial position.
Construction Cost = 160,000
Borrowing Cost = 9,138
= 169,138
Less Acc Dep = ( -- )
Carrying Amount = 169,138

Page 22 of 40

95
d) Borrowing cost to be expensed out during the year:
Total Borrowing cost = 47,500
(18,000 + 16,000 +
13,500)
Less Capitalized = (9,138)
To be expensed out = 38,362
A4.

a) Calculation of Capitalization Rate


Rupees
Loan MCB 500,000 X 9% 45,000
Loan HBL 615,000 X 10% 6/12 31,250
Loan UBL 375,000 x 11% 41,250
1,490,000 A 117,500

Weighted average Rupees


Loan MCB 500,000
Loan HBL (625000X 6/12) 312,500
Loan UBL 375,000
B 1,187,500

Capitalization rate B/A 9.89%

b) Borrowing cost to be capitalised


Loan Capitalization rate Months Rupees
May 31, 20X3 300,000 9.89% 7/12 17,308
July 31, 20X3 200,000 9.89% 5/12 8,242
25,550

c) Carrying Amount
Total Construction cost = 500,000
(300,000 + 200,000)
Borrowing cost capitalized = 25,550
Total Cost 525,550
Less Acc Dep = (--)
Carrying Amount = 525,550
d) Borrowing cost recognized as an expense:
Total borrowing cost incurred = 117,500
Less: Capitalized borrowing cost = (25,550)
Borrowing cots recognized as an expense = 91,950
A5.

a) Borrowing costs to be capitalized


Capitalization rate (weighted average rate of interest):
2 600,000 / 20,000,000
=13 %
300,000 X 13% X 12/12 39,000
200,000 X 13% X 9/12 19,500
250,000 X 13% X 6/12 16,250
150,000 X 13% X 4/12 6,500
250,000 X 13% X 3/12 6,500
87,750

Page 23 of 40

96
I thank Allah for giving me birth as Muslim and blessing with Islam.

b) Depreciation
Since the asset is not available for use (Still under construction) so there is no depreciation for the
year.

c) Carrying amount at 31 December 2005


Total Cost incurred = 1,100,000
Borrowing Cost capitalized = 87,750

Total Cost = 1,187,750


Less: Acc Dep = ( -- )
Carrying Amount = 1,187,750

A6.

Capitalization Rate:
Weighted
Borrowing Interest Period
borrowing
Debenture at 10% 5,000,000 500,000 12/12 5.000,000
Bank loan:
8.75% for six months 2,000,000 87,500 6/12 1,000,000
9% for six months 1,800,000 81,000 6/12 900,000
Bank overdraft
14% for six months 300,000 21,000 6/12 150.000
14.25% for six months 7006-,000 49,875 6/12 350,000

739,375 7,400,000

Capital rate = 739,375 x 100


7,400,000
= 9.992% p.a

Borrowing Cost to be Capitalized


1-7-2009 2,050,000 x 9.992% x 12/12 = 204,836
15-9-2009 1,000,000 x 9.992% x 9.5/12= 79,103
15-4-2010 500,000 x 9.992% x 2.5/12= 10,408
30-6-2010 400,000 x 9.992% x 0/12= -
3,950,000 294,347

Carrying Amount :
Total Cost (3950,000 + 294, 355) 4,244,347
Less *Acc (--)
Dep
Carrying Amount 4,244,347
*As asset is not yet available for use, therefore no depreciation.

Page 24 of 40

97
A7.

a) Borrowing costs to be capitalized


Interest incurred:
2,000,000 x 14% x 1 year = 280,000

Interest earned:
(1 400,000 x 10% x 6 / 1 2 ) + (200,000 x 10% x 2/12) = 73,333

You may find it easier to understand the calculation of the interest earned by looking at the following
tabular calculation:
Total Available for Interest on
From Used working
borrowings investment investment

1-Jan 2,000,000 600,000 1,400,000 70,000 1,400,000 x 10% x 6/12


1-Jul 1,200,000 200,000 3,333 200,000 x 10% x 2/12
1-Sep 200,000 - -
73,333

Interest to be capitalised
Interest incurred during construction 280,000
Less investment income during construction (73,333)
206,667

Please note: capitalization of borrowing costs does not cease during a temporary delay in construction.
b) Since construction has not yet started (the activities have not yet begun), none of the interest incurred
is eligible for capitalization during the year ended 31 Dec, 2005 (i.e. the interest incurred should be
expensed).

A8.
Borrowing cost to be capitalized
2011 2010
Commitment fee - 700,000
Specific Borrowing (W 2) 6,987,500 3,033,333
Investment income (W 3) (1,749,000) (1,181,333)
General Borrowing(W 4) 1,381,625 -
6,620,125 2,552,000
Workings:
(1) Borrowing cost to be capitalized
Detail of expenditure

Used from Used from


Date Payments amount
specific general
1-3-2010 1st payment 25,000,000 25,000,000 -
5,000,000+4,550,000(70m x 13%
31-8-2010 1st repayment x6/12) 9,550,000 -
31-01-2011 2nd payment 65,000,000 34,750,000 30,250,000
5,000,000+4,225,000(65m x 13%
28-02-2011 2nd repayment x6/12) - 9,225,000

69,300,000

Page 25 of 40

98
(2) Specific borrowing
Interest incurred :

up to 31 June 2010 70,000,000 13% 4/12 3,033,333


For upto 30 June 2010 3,033,333

July and august 70,000,000 13% 2/12 1,516,667


September to January 65,000,000 13% 5/12 3,520,833
March April May 65,000,000 13% 3/12 1,950,000
For Upto 30 June 2011 6,987,500

(3) Investment income:

up to 31 June 2010 44,300,000 8% 4/12 1,181,333


For upto 30 June 2010 1,181,333

44,300,000 8% 2/12 590,667


34,750,000 8% 5/12 1,158,333
For Upto 30 June 2011 1,749,010

(4) General Borrowing:

31-1-2011 30,250,000 14% 3/12 * 1,058,750


28-2-2011 9,225,000 14% 3/12 322,875
1,381,625
* one month suspended
A9.
a) Amount of borrowing cost to be Capitalized
Specific borrowings (W-1) 546,667
General borrowings (W-2) 59,550
606,217
1) Specific borrowing
Interest Incurred
8,000,000 x 18% x 5/12 = 600,000
Investment Income
8,000,000 x 4% x 2/12 = (53,333)
546,667

2) General Borrowings
1,000,000 x 11.91% x 3/12 = 29,775
3,000,000 x 11.91% x 1/12 = 29,775
6000,000 x 11.91% x 0/12 = -
59,550

Page 26 of 40

99
Workings

Capitalization Rate = 5,000,000 + 3,625,000 + 1,500,000 x 100 = 1,0125,000 x 100 =


11.91%
50,000,000 + 25,000,000 + 10,000,000
Details of Expenditures
Particulars Amount Owned Specific General
1-4-2011 10% Payment 2,000,000 2,000,000 - -
31-7-2011 45% Payment 9,000,000 - 8,000,000 1,000,000
30-9-2011 15% Payment 3,000,000 - - 3,000,000
31-10-2011 30% Payments 6,000,000 - - 6,000,000

b) Carrying Amount
Construction Cost 20,000,000
Borrowing Cost 606,217
Total Cost 20,606,217
Less: Acc. depreciation (342437)
(20,606,217 ÷ 10 x 2/12)
Carrying Amount 20,262,780

c) Interest Expense to be charged in income statement


Specific
8,000,000 x 18% x 2/12 240,000
Nov & Dec
General
Aay 50,000,000 x 10% = 5,000,000
Bee 25,000,000 x 14.5% = 3,625,000
Cee 10,000,000 x 15% = 1,500,000
10,125,000
Interest Capitalized (59,550)
To be expensed 10,065,450

Total Borrowing cost to be expensed out (240,000+10,065,450) 10,305,450

A10.

Capital work in progress - Factory building Rs. in ‘000'


Progress invoices received from the contractor 75,000.00
(30,000+20,000+10,000+15,000)
(Rain damages paid would be chargeable to profit and loss account /
insurance claim)
Borrowing costs to be capitalised:
Loan processing charges 500.00)
Interest on bank loan W-1 1,841.67)
Interest on running finance W-2 2,730.00)
Interest income from surplus loan amount W-4 (395.00)
Capital work in progress - June 30, 2010 79,676.67)

Page 27 of 40

100
Islam is the most beautiful and perfect religion. Islam teaches us peace and the truth

W-1: Interest on bank loan (Specific):


Rupees in ‘000
Interest amount Outstanding loan
Months Interest at 13%
From To amount
01-12-2009 31-05-2010 6 25,000 1625.00
01-06-2010 30-06 -2010 1 20,000 216.67
1,841.67

W-2: Interest on running finance


Rupees in ‘000
Payments from Months Interest at
Payment Payments
Invoice Runnin outstandin 15% per
s Description net of Right Bank
amount g g up to 30- annum
date deductions Issue Loan
finance 6-10 (W-3)
01-07-09 Advanced 10,000 10,000 10,000 12.00 1,500
payment
15-10 - 1st progress 30,000 25.500 15,000 10,500 8.50 1,116
09 bill
15-01 - 2nti progress 20,000 17,000 17,000 - - -
10 bill
15-04 - 3rd 10,000 8.500 7,500 1,000 2.50 31
10 progress bill
31-05 - Loan 1,625 1,625 1.00 20
10 interest
31-05 - Loan 5,000 5,000 1.00 63
10 instalment
15,000 *24,500 29,125 2,730

*Loan amount of Rs. 25,000,000 less processing charges of Rs. 500,000


15%
W-3: Average rate of interest for running finance facility (9,000/60,000)

W-4: Interest income from surplus loan amounts:


Rupees in
‘000'
Interest income Surplus loan Interest
From Months
To amounts income at 8%
01-12-09 15-01-10 1.5 24,500 (245)
16-01-10 15-04-10 3.0 7,500 (150)
(395)

W-5 Calculation of Net Payments: (used in W-2)


1st Bill 2nd Bill 3rd Bill 4th Bill
Amount of Bill 30,000 20,000 10,000 15,000
Less 10% Advance (3,000) (2,000) (1,000) (1,500)
5% Retention Money (1500) (1,000) (500) (750)
Net Payment 25,500 17,000 8,500 12,750

Page 28 of 40

101
Accounting Entries for the year ended 30-6-2010
(Not required in question just for additional information)

1-7-09 Advance to Contractor 10,000


Bank 10,000

30-9-09 CWIP 30,000


Advance to contractor 3,000
Retention money 1,500
payable
Payable to contractor 25,500

15-10-09 Payable 25,500


Bank 25,500

31-12-09 CWIP 20,000


Advance to contractor 2,000
Retention money 1,000
payable
Payable to contractor 17,000

15-1-10 Payable 17,000


Bank 17,000

31-3-10 CWIP 10,000


Advance 1,000
Retention money 5,00
payable
Payable to contractor 8,500

15-4-10 Payable 8,500


Bank 8,500

30-6-10 CWIP 15,000


Advance 1,500
Retention money 750
Payable
Payable to contractor 12,750

b)
Qureshi Steel Limited
Statement of Financial Position
As on 30-6-2010
2010
(000)
Non-Current Assets
CWIP-Factory Building 79,677

Current Assets
Advance to contractor 2,500
(10,000-3,000-2,000-1,000-1,500)
Insurance claim Receivable 750

Equity & Liabilities:


Equity
Share Capital 15,000

Page 29 of 40

102
Non-current Liabilities
Bank Loan 10,000
Retention Money Payable (1500+1000+500+750) 3,750

Current Liabilities
Current Portion of loan (5M x 2) 10,000
Financial Charges payable (20M x 13% x 1/12) 217
Payable to contractor(related to June Quarter 12,750

Running Finance (Assumed as Closing as well) 60,000

Page 30 of 40

103
A11.
a) Assuming borrowings as Specific Borrowing:
Interest Incurred:
32,000,000 x 13% x (306-61)/365 = 2,792,329
10,000,000 x 11% x 153/365 = 461,096
Less: Investment Income: (Given) = (500,000)
Borrowing Cost to be capitalized = 2,753,425

b) Assuming borrowings as General Borrowings:


5,000,000 x 12.73% x (306-61)/365 = 427,240
18,000,000 x 12.73% x (275-61)/365 = 1,343,451
16,000,000 x 12.73% x 92/365 = 513,385
Borrowing cost to be Capitalized 2,284,076

W-1) Calculation of Capitalization Rate:


= 3,948,658 X 100 = 12.73%
31,019,178

 Interest incurred on loans:


32,000,000 x 13% x 306/365 = 3,487,562
10,000,000 x 11% x 153/365 = 461,096
3,948,658

 Weighted Avg outstanding balance:


32,000,000 x 306/365 = 26,827,397
10,000,000 x 153/365 = 4,191,781
31,019,178

W-2) Details of Payments:


Funds used From
Date Particulars Amount Right Shares Loans
1-3-2015 First Payments 20,000,000 15,000,000 5,000,000
1-4-2015 Second Payments 18,000,000 - 18,000,000
1-10-2015 Third Payments 16,000,000 - 16,000,000

Page 31 of 40

104
Be righteous and prevent your nafs (soul) from evil desires.

Solutions of Self-TEST Questions


A1.

Spin Limited
Amount of borrowing cost to be Capitalized as on June 30,2009
Commitment Fee 125,000
Specific Borrowings(W-1) 1,912,500
General Borrowings (W-2) 1,419,808
3,457,308

Workings:
1) Specific Borrowings:
Interest Incurred:
25,000,000 x 12% x 5/12 1,250,000
20,000,000 x 12% x 4/12 800,000
2050,000
Less Investment Income:
6,875,000 x 8% x 3/12 137,500
1,912,500

2) General Borrowings
8,125,000 x 13.88% x 6/12 563,875
12,,000,000 x 13.88% x 4/12 555,200
6,500,000 x 13.88% x 4/12 300,733
1,419,808

3) Detail Of Payments

Particulars Amount Specific General Months for General Borrowing


1-9-2008 Permit 8,000,000 8,000,000
1-9-2008 1st Payment 10,000,000 10,000,000
1-12-2008 2nd Payment 15,000,000 6,875,000 8,125,000 6 months
1-2-2009 3rd Payment 12,000,000 - 12,000,000 4 months
1-2-2009 Repayment 6,500,000 - `6,500,000 4 months

4)Capitalization Rate
6,250,000 x 100 = 13.88%
4,5000,000
2.1 25,000,000 x 13% = 3,250,000
20,000,000 3,000,000
45,000,000 6,250,000

Page 32 of 40

105
A2. Amount of borrowing cost to be capitalized:
Rs. in million
Interest on redeemable preference shares [150 x 12% x (11- 1)-12] 15.00
Interest on TFCs [300 x 14% x (8-1)-12] 24.50

39.50
Less: Interest income from surplus funds (W-1) (3.23)
Amount of borrowing cost to be capitalized: 36.27

Surplus funds available Surplus Amount Interest income @ 9%


From To Months RS. in million
01-01-2012 31-03-2012 3 50 1.13
01-04-2012 30-11-2012 7* 40 2.1
3.23
*One month suspension

Details of Expenditures
Date Particulars Payments Specific Right Shares
Preference TFCs
Shares
1-1-2012 1st Payment 100,000 100,000
1-4-2012 2nd Payment 310,000 50,000 260,000
15-12-2012 3rd Payment 90,000 40,000 50,000
150,000 300,000 50,000

A3. Amount of borrowing cost to be capitalized:

Rs
Specific Loan

Rs 800,000 x 9% x 9/12 54,000


Less: Interest earned on unutilized amount of loan:
April 1 to June 30 [(800,000 – 450,000) x 3/12 x 6%] (5,250)
July 1 to October 31 [(800,000 – 550,000) x 4/12 x 6%] (5,000)
November 1 to November 30 [(800,000 – 750,000) x 1/12 x (250)
6%]
A 43,500
General Pool of Funds

Capitalization rate is 9.58% (W-1)


Paid on January 31 (200,000 x 11/12 x 9.58%) 17,563
Paid on November 30 (200,000 x 1/12 x 9.58%) 1,597
B 19,160

Total Amount to be Capitalized 62,660

Note: Although the activities has been interrupted for administrative work during May and June 2014,
capitalization of borrowing cost is not suspended for this period according to IAS-23, because it is a
necessary stoppage for the completion of construction work.

Page 33 of 40

106
W-1 Rs
Capitalization rate for pool of debt

Total interest paid on these borrowings:


Bank Overdraft 33,800
7-year note (9,000,000 x 10%) 900,000
933,800
Borrowings outstanding
Bank overdraft 750,000
7-year note 9,000,000
9,750,000
Capitalization rate = 933,800/9,750,000 9.58% (rounded)

W-2 Detail of Payments:


Particulars Specific General
31-1 1st Payment 200,000
31-3 2nd Payment 450,000
30-6 3rd Payment 100,000
31-10 4th Payment 200,000
30-11 5th Payment 50,000 200,000

Page 34 of 40

107
Answer No. 4
Cost of Capital work In Process: Building
As on 31-12-2010
Payments (3.2 + 4.4 + 2.3 + 6.2) 16,100,000
Add borrowing cost to be capitalised on Specific borrowings 55,667
Add borrowing cost to be capitalised on General borrowing 775,250
16,930,917

As on 31-12-2011
Opening balance 16,930,917
Add Payments (5.5 + 4.4 + 2.5 + 1.5) 13,900,000
Add borrowing cost to be capitalised on Specific borrowing 455,667
Add borrowing cost to be capitalised on General borrowing 2,675,166
33,961,750

(W-1) borrowing cost to be capitalised on Specific Borrowings:


2010
Interest incurred 10,000,000 × 12% × 1/12 = 100,000
Interest income 3,800,000 × 14% × 1/12 = (44,333)
55,667

2011
Interest incurred 10,000,000 × 12% × 5/12 = 500,000
Interest income 3,800,000 × 14% × 1/12 = (44,333)
455,667

(W-2) Borrowing cost to be capitalised in case of General Borrowings:


2010 3,200,000 × 14% × 10/12 = 373,333
4,400,000 × 14% × 6/12 = 308,000
2,300,000 × 14% × 3.5/12 = 93,917
9,900,000 775,250
2011
On opening 9,900,000 × 14% × 11/12 = 1,270,500
balance
1,700,000 × 14% × 10/12 = 198,333
4,400,000 × 14% × 7/12 = 359,333
10,600,000 × 14% × 6/12 = 742,000
2,500,000 × 14% × 3/12 = 87,500
1,500,000 × 14% × 1/12 = 17,500
2,675,166

Page 35 of 40

108
(W-3) Detail of Payments:

Funds Used From


Date Particulars Amount
Specific General
1-3-2010 1st Payment 3,200,000 -- 3,200,000
30-6-2010 2nd Payment 4,400,000 -- 4,400,000
16-9-2010 3rd Payment 2,300,000 -- 2,300,000
1-12-2010 4th Payment 6,200,000 6,200,000 --
31-1-2011 5th Payment 5,500,000 3,800,000 1,700,000
1-5-2011 6th Payment 4,400,000 -- 4,400,000
1-6-2011 Loan repayment + Interest 10,600,000 -- 10,600,000
(10 + 0.6)
1-10-2011 7th Payment 2,500,000 -- 2,500,000
1-12-2011 8th Payment 1,500,000 -- 1,500,000

Page 36 of 40

109
If your dream is Jannah then follow Quran and Sunnah.

Further practice
Example1: SHAYAN LIMITED
Question: Shayan Limited (SL) started the construction of its new factory on 1 January 2018 with a loan of
Rs. 50,000,000 borrowed at an interest rate of 8% per annum.

The loan was used on the factory as follows:

Date of Rs. in
Payment million
Jan 1, 2018 25
May 1, 2018 15
Oct 1, 2018 10

The construction of the asset was completed on 31 December 2018. However, during the accounting
period SL invested the surplus funds at an interest rate of 3%.

Required:

How much the amount of borrowing cost eligible for capitalization at 31.12.2018?

Answer:
Borrowing costs to be capitalised Rs.
Borrowing costs incurred Rs. 50m x 8% 4,000,000
Less: Temporary investment income Rs. 25m x 3% x 4/12 (250,000)
Rs. 10m x 3% x 5/12 (125,000)
3,625,000

Example 02: S LIMITED


Question: On January 1, 2018 S Limited (SL) started the construction of an asset. To meet the financing
general requirements, borrowing was made from three different banks at the start of the year as follows:

Banks Amount Interest Rate per


(Rs.) annum
A 70,000 10%
B 60,000 8%
C 50,000 12%

The funds were used on the assets as follows:


Date of Rs.
Payment
Jan 1, 2018 30,000
May 1, 2018 20,000
Oct 1, 2018 15,000

The construction of asset was completed on 31 December 2018.

Required: Calculate the general weighted average borrowing rate and eligible borrowing cost.

Page 37 of 40

110
Answer:

Borrowing costs eligible for capitalization Rs.


30,000 x 9.89% x 12/12 2,967
20,000 x 9.89% x 8/12 1,318
15,000 x 9.89% x 3/12 371
Borrowing costs to be capitalised 4,656

Capitalisation rate = (70,000 x 10%) + (60,000 x 8%) + (50,000 x X 100 = 9.89%


12%)
70,000 + 60,000 + 50,000

Example 03: LOONEY


Question: Looney has recently finished building a new item of plant for its own use. The item is a press for
use in the manufacture of industrial diamonds. Looney commenced construction of the asset on 1st April
2013 and completed it on 1st April 2015.
1st January 2013, Looney took out a loan to finance the construction of the asset. Interest is charged on
the loan at the rate of 5% per annum. The annual interest must be paid in four equal installments at the
end of each quarter. Looney capitalises interest on manufactured assets in accordance with the rules in
IAS 23 Borrowing costs.
The costs (excluding finance costs) of manufacturing the asset were Rs. 28 million.

Required:
State the IAS 23 rules on the capitalisation of borrowing costs, calculate the cost of the asset on initial
recognition and explain the amount of borrowing cost capitalised.
Answer:
IAS 23 should be applied in accounting for borrowing costs. Borrowing costs are recognised as an
expense in the period in which they are incurred unless they are capitalised in accordance with IAS 23
which says that borrowing costs that are directly attributable to the acquisition, construction or production
of a qualifying asset can be capitalised as part of the cost of that asset.
• A qualifying asset is an asset that necessarily takes a substantial period of time to get ready for its
intended use or sale.
• Borrowing costs that are directly attributable to acquisition, construction or production are taken to
mean those borrowing costs that would have been avoided if the expenditure on the qualifying asset
had not been made.

When an enterprise borrows specifically for the purpose of funding an asset, the identification of the
borrowing costs presents no problem as the amount capitalised is the actual borrowing costs net of any
income earned on the temporary investment of those borrowings.

If funds are borrowed, generally, the amount of borrowing costs eligible for capitalisation is determined by
applying a capitalisation rate to the expenditures on that asset calculated as the weighted average of the
borrowing costs applicable to general borrowings.
IAS 23 also contains rules on commencement of capitalisation, suspension of capitalisation and cessation
of capitalisation.

Page 38 of 40

111
It is never too late to turn to Allah. Leave shirk and biddah and turn to Allah.

Amount capitalised Rs.000


Cost of manufacture 28,000
Interest capitalised (Rs.28m × 5% × 2 years) 2,800
30,800

Example 04: GOOGLY INDUSTRIES LIMITED


Question: On 1 January 2016 Googly Industries Limited (GIL) borrowed Rs.15 million to finance the
production of two assets, both of which were expected to take a year to build. Work started during 2016.
The loan facility was drawn down and incurred on 1 January 2016, and was utilised as follows, with the
remaining funds invested temporarily.

Asset A Asset B

----------- Rs. in million ---------


1 January 20X6 2.5 5
1 July 20X6 2.5 5

The loan rate was 9% and GIL can invest surplus funds at 7%.

Required: Calculate the borrowing costs which may be capitalised for each of the assets and
consequently the cost of each asset as at 31 December 2016.

Answer:
The borrowing cost to be capitalized & cost of assets are as under:
Cost of asset Rs.
A Expenditure 5,000,000

Borrowing costs incurred 5m (2.5+2.5)x 9% x 12/12 450,000


Less: Temporary investment income 2.5m x 7% x 6/12 (87,500)
362,500
5,362,000

Cost of asset Rs.


B Expenditure 10,000,000

Borrowing costs incurred 10m (5+5) x 9% x 12/12 900,000


Less: Temporary investment income 5m x 7% x 6/12 (175,000)
725,000
10,725,000

Page 39 of 40

112
Example 05: KHAN LIMITED
Question: Khan Limited (KL) has the following general loan arrangements as at 1 January 2020:

Rs. in million

7% Debentures 55
8% Loan notes 110
12% Line of credit 85
10% Running finance arrangement 150

On the 1 January 2020, KL commenced the construction of a new factory. The construction of the factory
will cost Rs.100 million and the company funded the construction with the existing borrowings. The
factory was completed on 31 August 2020 but was not available for use until 31 January 2021 as a result
of minor modification. During the construction period, active work was interrupted and the building
construction was stopped for two months as a result of adverse weather conditions.

Required:

Calculate the borrowing cost to be capitalised and the cost of the building to be recognised upon initial
recognition.

Answer:
The borrowing cost to be capitalized & cost of assets are as under:
Rs.
Expenditure 100,000,000
Borrowing costs to be capitalised 100,000,000 x 9.46% x 6/12 4,730,000
104,730,000

January to August are 8 months – two months suspension = 6 months

Capitalisation rate = (55 x 7%) + (110 x 8%) +(85 x 12%) + (150 x 10%) X 100 = 9.46%

55+110+85+150

Page 40 of 40

113
IAS 23
Example:
Identify whether or not the following are qualifying assets.
(i) A construction company constructing a bridge for government which will take 6 years
to complete.
(ii) A very sophisticated integrated circuits being made by an entity who manufactures and
sales 10,000 to 12,000 units every month.
STICKY NOTES

(iii) A power plant under construction, it may take 10 months to complete this.
(iv) An equipment purchased by X Limited, the equipment may be used immediately after it
is delivered.
(v) Special order from a customer to manufacture a machine for him which will take 11
months at the least.
(vi) An entity is constructing office building which will take 8 months to complete.
Answer:
Items (i), (iii), (v) and (vi) are qualifying assets.
Items (ii) and (iv) are not qualifying assets.

Example:
Up Limited borrowed a loan of Rs. 10 million from Down Bank on 15% per annum for
constructing its power generation facilities.
The loan was received on February 01, 2011. Up Limited paid Rs. 3 million to contractor
immediately but remaining Rs. 7 million were paid to the contractor on March 1, 2011. The
remaining Rs. 7 million were temporarily invested in a saving account at 9% per annum.

STICY NOTES
Up Limited has year-end of 31 December. As on December 31, 2011 the construction is still in
process and the loan is also outstanding.
Required: Calculate the amount of borrowing cost to be capitalised for the year ended December
31, 2011?
⯈ Answer:

Borrowing costs to be capitalised Rs.

Actual borrowing costs [Rs. 10 m x 15% x 11/12] 1,375,000

Less: Temporary investment income [Rs. 7m x 9% x 1/12] (52,500)

1,322,500

114
Example:
On 1 January 2016 Okara Engineering issued a bond to raise Rs. 25,000,000 to fund a capital
project which will take three years to complete. Amounts not yet needed for the project are
invested on a temporary basis. During the year to 31 December 2016, Okara Engineering spent
Rs. 9,000,000 on the project for labour, materials and direct overheads etc.
The cost of servicing the bond was Rs. 1,250,000 during this period and the company was able to
earn Rs. 780,000 through the temporary reinvestment of the amount borrowed.
Required: Calculate the amount of addition to capital work in progress during the year to
December 31, 2016.
⯈ Answer:
AT A GLANCE

Rs.
Costs incurred (labour, material, overhead etc.) 9,000,000
Borrowing costs to be capitalised:
Actual borrowing costs incurred on that borrowing 1,250,000
Less: Temporary investment income (780.000)
470,000
Additions to capital work in progress 9,470,000

AT A GLANCE
Example:

SIKA Sports Limited is constructing a stadium for last some years. During the year ended 31
December 2011, it has incurred the following expenditures.

April 30, 2011 Rs. 2,500,000

July 31, 2011 Rs. 2,300,000

No specific loan was borrowed for the construction; rather general pool of funds was used. The
following loans are outstanding:

Loan from FBL @12% Outstanding since beginning of year Rs. 5,000,000

Loan from BAH @14% Outstanding since beginning of year Rs. 10,000,000

Loan from BAF @16% Outstanding since 01-09-2011 Rs. 750,000

STICKY NOTES
Required: Calculate total borrowing costs eligible for capitalisation during the year ended
December 31, 2011.
⯈ Answer:
Capitalisation rate =
4
[𝑅𝑠. 5𝑚 × 12%] + [𝑅𝑠. 10𝑚 × 14%] + [750,000 × 16% × ]
12 = 𝟏𝟑. 𝟑𝟖%
=
4
𝑅𝑠. 5 𝑚 + 𝑅𝑠. 10𝑚 + 𝑅𝑠. 750,000 ×
12
Borrowing costs to be capitalised Rs.
Rs. 2,500,000 x 13.38% x 8/12 223,000
Rs. 2,300,000 x 13.38% x 5/12 128,225
351,225

115
Commencement date of capitalisation [IAS 23: 17 to 19]
An entity shall begin capitalising borrowing costs as part of the cost of a qualifying asset on the commencement
date.
The commencement date for capitalisation is the date when the entity first meets all of the following conditions:
• it incurs expenditures (resulted in payment of cash or transfer of other assets) for the asset;
• it incurs borrowing costs; and
• it undertakes activities that are necessary to prepare the asset for its intended use or sale.
The activities necessary to prepare the asset for its intended use or sale encompass more than the physical
AT A GLANCE

construction of the asset. This includes technical and administrative work prior to the commencement of physical
construction, such as the activities associated with obtaining permits prior to the commencement of the physical
construction.
However, such activities exclude the holding of an asset when no production or development that changes the
asset’s condition is taking place. For example, borrowing costs incurred while land is under development are
capitalised during the period in which activities related to the development are being undertaken. However,
borrowing costs incurred while land acquired for building purposes is held without any associated development
activity do not qualify for capitalisation.

Suspension of capitalisation [IAS 23: 20 & 21]


An entity shall suspend capitalisation of borrowing costs during extended periods in which it suspends active
development of a qualifying asset.
SPOTLIGHT

An entity does not normally suspend capitalising borrowing costs during a period when it carries out substantial
technical and administrative work. An entity also does not suspend capitalising borrowing costs when a
temporary delay is a necessary part of the process of getting an asset ready for its intended use or sale. For
example, capitalisation continues during the extended period that high water levels delay construction of a
bridge, if such high water levels are common during the construction period in the geographical region involved.

End of capitalisation period [IAS 23: 22 to 25]


An entity shall cease capitalising borrowing costs when substantially all the activities necessary to prepare the
qualifying asset for its intended use or sale are complete. An asset is normally ready for its intended use or sale
when the physical construction of the asset is complete even though routine administrative work might still
STICKY NOTES

continue. If minor modifications, such as the decoration of a property to the purchaser’s or user’s specification,
are all that are outstanding, this indicates that substantially all the activities are complete.
When an entity completes the construction of a qualifying asset in parts and each part is capable of being used
separately while construction continues on other parts, the entity shall cease capitalising borrowing costs when
it completes substantially all the activities necessary to prepare that part for its intended use or sale.
⯈ Example:
A business park comprising several buildings, each of which can be used individually, is an
example of a qualifying asset for which each part is capable of being usable while construction
continues on other parts. Capitalisation of borrowing cost should cease for each part when that
part is substantially complete even though the work on other parts continues.
⯈ Example:
An example of a qualifying asset that needs to be completed before any part can be used is an
industrial plant involving several processes which are carried out in sequence at different parts
of the plant within the same site, such as a steel mill. In such case, capitalisation of borrowing
costs shall cease when the whole asset is substantially complete.

116
⯈ Example:
Cord Limited is engaged in the manufacturing of automobiles. Currently the company is
manufacturing its power generation plant. The project was started on January 03, 2011 with
company’s own funds. Subsequently, Cord Limited borrowed a loan from ZBL Bank to finance the
project on February 22, 2011. The first payment out of the loan was made on March 04, 2011.
Due to some law and order situation, the project remained closed from April 25, 2011 to May 9,
2011. The work was also stopped for a week from May 23, 2011 to May 30, 2011 so that
necessary plan and layout can be finalized after testing of project completed so far.
The plant was completed on July 31, 2011 except that some sign board could not be installed
until August 10, 2011. Loan was repaid on August 31, 2011. Cord Limited started using the plant
on September 1, 2011.

AT A GLANCE
Required:
(a) When Cord Limited should start capitalising borrowing costs?
(b) Should Cord Limited suspend capitalisation from April 25, 2011 to May 9, 2011?
(c) Should Cord Limited suspend capitalisation from May 23, 2011 to May 30, 2011?
(d) When Cord Limited should cease to capitalise borrowing costs?
⯈ Answer:
(a) February 22, 2011 i.e. when all three criteria for commencement date of capitalisation
have been met.
(b) Suspend capitalisation, as active development of qualifying asset was suspended during
this period.

SPOTLIGHT
(c) Continue capitalisation, substantial technical and administrative work was still being
carried out.
(d) July 31, 2011 i.e. when asset was substantially completed.

Expenditure on qualifying asset [IAS 23: 18]


Expenditures on a qualifying asset include only those expenditures that have resulted in payments of cash or
transfers of other assets.
The average carrying amount of the asset during a period, including borrowing costs previously capitalised, is
normally a reasonable approximation of the expenditures to which the capitalisation rate is applied in that
period.

STICKY NOTES
Expenditures are reduced by:
• any progress payments received; and
• grants received in connection with the asset.
⯈ Example:
Zeal Limited (ZL) is building a dam for Federal Government. The project will take 10 years to
complete. On February 01, 2011 ZL used Rs. 100 million from pool of general loans with
capitalisation rate of 12% for the expenditures incurred on the same date for payment to sub-
contractors who started work immediately.
On July 01, 2011 the Federal Government made first progress payment of Rs. 30 million.
As ZL had offered employment opportunities to locals of the area, considering this fact; Provincial
Government has given ZL a grant of Rs. 10 million on October 31, 2011.
Required: The borrowing costs to be capitalised for Zeal Limited for the year ended December
31, 2011.

117
⯈ Answer:

Cumulative Borrowing costs to be


Expenditure
Date Particulars (net) capitalised
Rs. m Rs. m
1 Feb Sub-contract costs 100 100 x 12% x 5/12 5
1 Jul Progress payments (30) 70 x 12% x 4/12 2.8
31 Oct Grant (10) 60 x 12% x 2/12 1.2
9
AT A GLANCE

Disclosure [IAS 23: 26]


An entity shall disclose:
• the amount of borrowing costs capitalised during the period; and
• the capitalisation rate used to determine the amount of borrowing costs eligible for capitalisation.
⯈ Example:
Alpha Limited borrowed Rs. 9 million @ 15% per annum to fund a qualifying asset project on 1
January 2016. The following expenditures were made on the project during the year ending 31
December 2016:

Date Rs. m
SPOTLIGHT

1 March 2016 2.5


1 October 2016 4.2
1 December 2016 2.3

Surplus funds were invested @10% whenever available.


The project activities started on 1 March 2016. Work on the project was suspended during the
whole month of August and resumed at start of September. Construction was completed on 31
December 2016.
Required: Calculate the borrowing costs to be capitalised and net borrowing cost
STICKY NOTES

to be charged to profit or loss.


Answer:

The project commenced on 1st March resulting in a construction period of 10 months up to the
year end. However, interest cannot be capitalised during the period of suspension. Therefore,
interest is capitalised only for 9 months.

Borrowing costs to be capitalised Rs. 000

Borrowing costs incurred [Rs. 9m x 15% x 9/12] 1,012.5

Less: Temporary investment income [Rs. 6.5m x 10% x 6/12] (325)

[Rs. 2.3m x 10% x 2/12] (38.3)

649.2

118
Borrowing costs to be charged as expense Rs.
Borrowing costs incurred (Jan, Feb, Aug) [Rs. 9m x 15% x 3/12] 337.5
Less: Temp. Inv. income (Jan & Feb) [Rs. 9m x 10% x 2/12] (150)
(Aug) [Rs. 6.5m x 10% x 1/12] (54.17)
133.33

119
Accounting Policies changes in Accounting Estimates and Errors
[IAS 8]

Accounting Polices: These are principles, rules and regulations adopted by management of an entity in preparing and
presenting financial statements. These rules and regulations are specified in IFRS.

Examples of IFRS where we have discussed accounting treatments are IAS-2, IAS-16, IAS-23 etc.

• As per IAS-8, accounting policies should be selected by applying the requirements of IFRS. E.g FIFO or Weighted
Average for inventory measurement.
• As per IAS-8, once a policy is selected it should be applied consistently to achieve comparability of figures over a
period of time.

Changes in Accounting Policies: An entity shall change its accounting policy only if:

a) The change is required by an IFRS (compulsory change); or


b) Management feels that change results in financial statements providing more relevant and reliable
information (voluntary change).

Prospective change: means effect of change is applied in current and future periods. No effect of change is applied in
previous periods.

Retrospective change: means effect of change is applied in current, future as well as in all previous periods.

Transitional provisions: sometimes if there is a change in an accounting policy in an accounting standard, standard
itself explains how to change the policy (means accounting treatment of change is given in the relevant standard). That
given accounting treatment in an accounting standard is called as transitional provisions.

Impracticable: means not possible to apply after making every reasonable effort to do so, e.g records related to all
relevant previous periods are not available.

How to adjust the Financial statements in case of Change in Accounting Policy:

a) If change is required by IFRS then use transitional provisions (if available) otherwise apply the change
retrospectively, unless it is impracticable to do so. If it is impracticable to apply retrospectively, then apply the
change from the date it is practicable to change.
b) If it is a Voluntary Change then apply the change retrospectively unless it is impracticable to do so. If it is
impracticable to apply retrospectively, then apply the change from the date it is practicable to change.

Page 1 of 33
120
Example
Change in accounting policy
During 2003, a revised IFRS on borrowing costs (IAS 23) was published. The company had previously been expensing
borrowing costs as a period cost, but the revised IFRS required that all borrowing costs be capitalized to the related
asset. The borrowing costs were all incurred on construction of a plant. The revised IFRS provided transitional provisions
that allowed the company to capitalize the costs from years beginning on or after 2004, or before this date, if preferred.
This entity chose to capitalize the borrowing costs from the earliest date possible. The plant is not yet available for use.
The effect of this change is as follows:

2001 2002 2003


Interest expense
Old policy 15 000 17 000 9 000
New policy 0 0 0

The following drafts were produced before adjusting for the change in accounting policy:

Draft statement of comprehensive income 2003 2002


For the year ended 31 December 2003 (extracts)
Profit for the year 455 000 380 000
Retained earnings at the beginning of the year 500 000 120 000
Retained earnings at the end of the year 955 000 500 000

Draft statement of financial position 2003 2002 2001


As at 31 December 2003 (extracts)
ASSETS

Plant 500 000 450 000 300 000


EQUITY AND LIABILITIES
Retained earnings 955 000 500 000 120 000

Required: Prepare the followings


a) Calculate the effect of change in policy
b) Statement of Comprehensive income (extracts) for the year ended 31-12-2003.
c) Statement of changes in equity (extracts) for the year ended 31-12-2003.
d) Statement of Financial Position (extracts) as on 31-12-2003
e) Notes to the financial statements for the year ended 31-12-2003.

Page 2 of 33
121
Solution
a) Effect of change in accounting policy
Decrease in Interest Expense Increase in Profit for the year
2001 15,000 15,000
2002 17,000 17,000
2003 9,000 9,000

b) Company Name
Statement of Comprehensive Income (Extracts)
For the year ended 31-12-2003
Restated
2003 2002
Profit for the year 464,000 397,000
2003 (455,000+9,000)
2002 (380,000+17,000)

c) Company Name
Statements of Changes in Equity (Extracts)
For the Year ended 31-12-2003
Retained Earnings
Balance as on 1-1-2002 120,000
Effect of change in policy (a) above 15,000
Balance as on 1-1-2002-Restated 135,000
Profit for the year 2002-Restated 397,000
Balance as on 31-12-2002-Restated 532,000
Profit after tax 2003 464,000
Balance as on 31-12-2003 996,000

Restated has not been mentioned with figures of 2003, because these figures are reported for the first time
to users of financial statements.

d) Company name
Statement of financial position (extracts)
As at 31 December 2003
Restated Restated
2003 2002 2001

ASSETS

Plant (500 000+15 000+17 000+9 000) 541 000 482 000 315 000
(450 000 + 15 000+17 000)
(300 000 + 15 000)

LIABILITIES AND EQUITY


Retained earnings (statement of changes in equity) 996 000 532 000 135 000

Disclosure of change in accounting policy:

a) Nature of change in accounting policy.


b) Reason of change in policy.
c) Effect on each *line item of statement of financial position and statement of comprehensive income.

*Line items means main headings of statement of financial position and statement of comprehensive income.

Page 3 of 33
122
e) Company name
Notes to the financial statements (extracts)
For the year ended 31 December 2003
Accounting policies
Borrowing costs
Borrowing costs are capitalized to qualifying assets. This represents a change in accounting policy.
Change in accounting policy
The company changed its accounting policy from expensing borrowing costs as they are incurred to capitalizing
borrowing costs to plant, a qualifying asset.
The change was made to comply with the revised IAS 23 Borrowing costs issued during the year. The effect of the change
in accounting policy is as follows:
2002

Effect on the statement of comprehensive income


Decrease in Finance costs 17 000
Increase in profit for the year 17 000

2002 2001
Effect on the statement of financial position

Increase in Plant 32 000 15 000


(15+17)
32 000 15 000
Increase in Retained earnings
(10,500+11,900)

Prior period Errors is an error made in financial statements of one or more prior periods but discovered in current period
after the publication of financial statements of those periods. For example in 2015 we have discovered that sale was
overstated in 2001.

Prior period errors are corrected retrospectively.

Errors that occurred in current period are simply corrected in current period (with no disclosure requirements).

Disclosures of prior period error

a) Nature of prior period error


b) Effect on each line items of statement of financial position and statement of comprehensive income

Page 4 of 33
123
Q.1 Mohani Manufacturing Limited is engaged in manufacturing of spare parts for motor car assemblers. The audited
financial statements for the year ended December 31, 2007 disclosed that the profit and retained earnings were Rs. 21
million and Rs. 89 million respectively. The draft financial statements for the year show a profit of Rs. 15 million.
However, following adjustments are required to be made:
(i) The management of the company has decided to change the method for valuation of raw materials from FIFO to
weighted average. The value of inventory under each method is as follows:

FIFO Weighted Average


Rupees in million
December 31, 2006 37.0 35.5
December 31, 2007 42.3 44.5
December 31, 2008 58.4 54.4
(ii) In 2007, the company purchased a plant for Rs. 100 million. Depreciation on plant was recorded at Rs. 25 million
instead of Rs. 10 million. This error was discovered after the publication of financial statements for the year ended
December 31, 2007. The error is considered to be material.
Required:
Produce an extract showing the movement in retained earnings, as would appear in the statement of changes in equity
for the year ended December 31, 2008.
Points to remember

 Closing stock has direct relationship with profit


 Opening stock has inverse relationship with profit
 Closing stock of previous period becomes opening stock of next period
 If there is no information of dividend then ignore dividend
 Even if question is silent but information is available then always prepare at least one year comparative figures
in statement of changes in equity as well.

Change in Accounting Estimates

Examples of accounting estimates are allowance for doubtful debts, useful life, depreciation methods or residual value of
depreciable assets etc.

Accounting estimate are always changed prospectively.


Prospective change: means effect of change is applied in current and future periods. No effect of change is applied in
previous periods.

Whenever there is change in accounting estimates of useful life or residual value or there is a change in
method of depreciation then the WDV of the asset at the time of change is depreciated over remaining useful
life. Depreciation of previous periods is not adjusted.

Disclosures:

a) Nature of change in accounting estimate.


b) Financial effect of change in estimate in current and future periods on profit.

Q.2 CM is the managing director of Cl Limited, a company specializing in solving murders. Cl Limited owns a large
amount of very costly forensic equipment.
• The forensic equipment was purchased on 1 April 2001, at a cost of Rs 600 000.
• At that time it was determined that the equipment would be depreciated on a straight line basis over a period of 6
years.
• On 1 April 2003, the total life of the forensic equipment was re-estimated to be 10 years. Cl Limited decided to
adjust their records accordingly (using the re-allocation method) (means prospectively).

Required:
Show the depreciation journal entries necessary from the information provided above, assuming:
i. Depreciation had not yet been processed for the year ended 31 March 2004;
ii. Depreciation based on the old estimate had already been processed for the year ended 31 March 2004.

Page 5 of 33
124
Q.3 Wonder Limited (WL) is engaged in the manufacturing and sale of textile machinery. Following are the draft
extracts of the statement of financial position and the statement of profit or loss for the year ended 30 June 2012:

Statement of Financial Position


2012 2011
Rs. in million
Property, plant and equipment 189 130

Retained earnings 198 108

Statement of profit or loss


2012 2011
Rs. in million
Profit for the year 90 120

Following additional information has not been taken into account in the preparation of the above financial statements:
(i) Cost of repairs amounting to Rs. 20 million was erroneously debited to the machinery account on 1 October 2010.
The estimated useful life of the machine is 10 years.
(ii) On 1 July 2011, WL reviewed the estimated useful life of its plant and revised it from 5 years to 8 years. The plant
was purchased on 1 July 2010 at a cost of Rs. 70 million.
Depreciation is provided under the straight line method.

Required:
Prepare relevant extracts (including comparative figures) for the year ended 30 June 2012 related to the following:
(a) Statement of financial position
(b) Statement of profit or loss
(c) Statement of changes in equity
(d) Correction of error note* ]
*correction of error note means disclosure.

Q.4 The following information has been extracted from the financial statements of Fine Fiber Limited (FFL) for the year
ended 30 June 2013:
2013 2012
Rs. in million
Profit for the year 93 82
Share capital (Rs. 10 each) 30 30
Retained earnings at the end of the year 186 105
Subsequent to preparation of the draft financial statements, an error has been detected in the financial statements for
the year ended 30 June 2012 whereby the depreciation on an assembly plant was mistakenly accounted for at Rs. 21.8
million instead of Rs. 12.8 million.

Other relevant information is as under:


i. The assembly plant was installed on 1 July 2010 at a cost of Rs. 80 million and is depreciated at 20% per annum
using the diminishing balance method.
ii. Final cash dividend for the year ended 30 June 2012 was approved at the Annual General Meeting held on 25
September 2012 at Rs. 4 per share (2011: Rs. 5 per share).
Required:
Prepare the following extracts (including comparative figures) from FFL’s financial statements for the year ended 30 June
2013 in accordance with the International Financial Reporting Standards:
a) Retained earnings column as would appear in the statement of changes in equity; and
b) Correction of error note.

Page 6 of 33
125
Before next question:
Depreciation by number of units produced
Depreciation is calculated by expressing the useful life of an asset in terms of its expected total output and allocating the
annual charge to depreciation based on actual output.
Formula:
Rate/Unit = Cost – Residual value
Total expected production
Example
A machine cost Rs 500,000.
It is expected to produce 5,000,000 units over its useful life.
47,850 units were produced in first year of production.
The depreciation charge for the first year of operation is
Depreciation = 500,000 x 47,850 = 4,785
5,000,000
Impairment loss: if carrying amount is more than recoverable amount, difference is called as impairment loss.
Recoverable amount is higher of:
• Fair value less cost to sell
• Value in use.
Q.5 Zain Pharma Limited (ZPL) owns patents of branded products A and B. ZPL uses cost model to account for its intangible
assets. It is policy of the company to amortize the cost on the basis of total estimated revenue earned over the life of the
patents. The following information is available:

Product 'A' Product 'B'


Date of acquisition of the patents 1 -Jan-2011 l-Jul-1998
Patents’ life 15 years 18 years
Rs. in million
Acquisition cost of the patents 37.50 36.00
Sales upto 31-Dec-2012 800.00 11,200.00
Sales for the year ended 31-Dec-2013 600.00 1,600.00
Future estimated sales 6,100.00 3,200.00
Recoverable amount of the patents as at 31-Dec-2013 35.00 5.00
However, in 2013 the management identified that upto the year ended 31 December 2012, the cost of patents had
erroneously been amortized on the basis of estimated useful life.

Required:
Prepare accounting entries relating to the patents, for the year ended 31 December 2013 in accordance with the
International Financial Reporting Standards.

Page 7 of 33
126
Summary of discussion about dividend
It is a distribution of profits to owners (shareholder). If dividend Is given as a percentage then multiply the rate with the
amount of share capital (nominal value of share capital) which is at the date of declaration of dividend to calculate the
amount of dividend. A company may pay dividend either in cash or in form of shares.
Dividend
Cash Dividend (Either Final or Interim) Bonus Shares (Either Final or Interim)
When Declared Dividend XXX
Dividend(Retained Earnings) XXX Share Capital A/c XXX
Dividend Payable A/c XXX

When Paid
Dividend Payable A/c XXX
Cash/ Bank A/c XXX

• Bonus shares are distributed from retained earnings and if no balance of retained earnings then from share
premium.
• Dividend is recognized on the date of declaration.
• If the dividend is declared after reporting date but before the authorization of the financial statements it is
disclosed in the financial statement to which it relates.
• Dividend is adjusted against retained earnings in the statement of changes in equity,
• If the date of declaration is not given, then:
1. For interim dividend assume that dividend is declared during the accounting period.
2. For final dividend assume declared after the reporting date.
• If nature of dividend (means whether final or interim) is not given then assume final dividend.
• If type of dividend (means cash or bonus) is not available then assume cash dividend.

Q.6 SCCL, a company listed on Karachi and Lahore stock exchange is in process of finalization of its accounts for the
year ended 31-12-2012. The following information is available

i. Shareholders equity as at 31-12-2011 and 2010 consist of


Rs in million
2011 2010
Share capital (Rs 10 each) 10,340 7,833
Unappropriated Profits 6,945 4,508

ii. Profit for the year for the years ended 31-12-2010, 2011 and 2012 (unaudited) was Rs 4,240 million, Rs
4,944 million and Rs 5,090 million respectively.
iii. During the year ended 31-12-2012 it was discovered that machinery purchased on 1-7-2011 for Rs 35
million was erroneously debited to repair account. SCCL depreciates machinery at the rate of 20% per
annum on reducing balance method No adjustment has been made in respect of this material error.
iv. Cash dividends and bonus declared/paid during the last three years were as follows:
Cash Dividend Bonus Shares
*Interim Final *Interim Final
For the year ended 31-12-2010 10% - - 20%
For the year ended 31-12-2011 - 15% 10% 10%
For the year ended 31-12-2012 - 10% 5% 5%

*Interim dividend/bonus was declared at time of announcement of half-yearly financial results.

v. Right shares were issued on 30-11-2012 in the ratio of 4 right shares for every 5 shares held by the
shareholders of the company. The right issue was made at Rs 18/share.

Required: Prepare statement of changes in equity for the year ended 31-12-2012.
Note: If more than one dividend is declared on the same date then for both dividends base share capital is same.

Page 8 of 33
127
Self-Practice Questions
Q.1 Question of Change in Accounting Estimate and its relevant disclosures:
On 1 January 2003 M Limited purchased a plant for Rs 500,000. The company used the reducing balance method for
calculating depreciation at a rate of 20% per annum.
During 2006 the directors decided it was necessary to change to the straight line method of calculating depreciation for
plant. It was agreed that the remaining estimated life of the plant was 3 years and the estimated residual value Rs
16,000 (previously this was nil).
M accounts for changes in estimates using the re-allocation method (means prospectively).
The draft statement of comprehensive income and statement of changes in equity had been prepared for the year
ended 31 December by the bookkeeper who had not been advised of the director’s decision.

M LIMITED
DRAFT STATEMENT OF COMPREHENSIVE INCOME
FOR THE YEAR ENDED 31 DECEMBER 2006
2006 2005
Revenue 800,000 650,000

Profit before depreciation 380,200 300.000


Depreciation – plant (51,200) (64,000)
Profit for the year 329,000 236,000

M LIMITED
DRAFT STATEMENT OF CHANGES IN EQUITY
FOR THE YEAR ENDED 31 DECEMBER 2006
Retained earnings

Balance: l January 2005 (40,000)


Profit for the period 236,000
Dividends (10,000)
Balance: 31 December 2005 186,000
Profit for the period 329,000
Dividends (15,000)
Balance: 31 December 2006 500,000

Required:
a) Prepare the statement of comprehensive income (extracts) and relevant extracts from the statement of changes in
equity of M Limited for the year ended 31 December 2006, in compliance with International Financial Reporting
Standards.
b) Notes to the financial statements related to change in accounting estimates.
Comparatives are required.

Page 9 of 33
128
Q.2
Chand Paints Limited (CPL) is engaged in the manufacturing of chemicals and paints. In April 2016 it was discovered that
certain errors had been made in the financial statements for the year ended 30 June 2015. The errors were corrected in
2016. The details are as follows:

2015 After
2016 (Draft) Correction of 2015 Audited
Errors
----------- Rs. In million ----------
Statement of comprehensive income
Sales tax, commission and discounts (7,939) (8,246) (7,916)
Cost of sales (45,508) (44,606) (44,633)
Selling and distribution expenses (2,940) (2,635) (2,441)
Administration expenses (2,356) (2,254) (2,149)
Other operating charges (495) (467) (515)
Other operating income 920 427 509
Profit for the year 4,089 3,723 4,359
Statement of financial position
Trade and other receivables 1,839 1,613 2,025
Trade and other payables 11,600 8,894 8,670

The share capital and un-appropriated profit of CPL as on 1 July 2014 was Rs. 10,400 million and Rs. 19,089 million
respectively.
The details of dividend declared are as follows:

2016 2015
Cash dividend – Interim 10% 5%
– Final 15% 10%

Required:
(a) Prepare a correction of error note to be included in the financial statements for the year ended 30 June 2016.
(Ignore earnings per share and taxation) (10)
(b) Prepare the statement of changes in equity for the year ended 30 June 2016. (08)

Q. 3
The following information has been taken from the financial statements of Asif engineering Limited (AEL) for the year
ended 31 December 2015:

2015 (draft) 2014 2013


--------------- Rs. In million ---------------
Property, plant equipment 2,430 2,402 2,105
Stores and spares 73 80 70
Retained earnings as at 31 December 353 224 101
Net profit 129 123 112

In the above financial statements, AEL has recognised consumption of spare parts as expense. AEL has now decided to
change its above policy and classify consumption of spares having useful life of more than one year as capital spares
under property, plant and equipment.

Page 10 of 33
129
Following information pertains to capital spares consumed during the past there years:

Parts issued during the year Rs. In


Year ended Useful life of the issued parts
million
31 December 2013 55 5 years
31 December 2014 39 3 years
31 December 2015 44 4 years

Depreciation on these parts is to be charged using straight line method over its useful life.

Required:
In accordance with the requirements of International Financial Reporting Standards, prepare the revised extracts
(including comparative figures) of the following:
(a) Statement of financial position as at 31 December 2015. (04)
(b) Statement of comprehensive income for the year ended 31 December 2015. (03)
(c) statement of changes in equity for the year ended 31 December 2015. (03)
(Ignore taxation)

Q. 4
The following information pertains to draft financial statements of Pak Ocean Limited (POL) for the year ended 31
December 2014.
(i)
2014 2013
Rs. In million
Profit after tax 90 47
Incremental depreciation on revaluation of property, plant and
equipment (means transfer of Revaluation surplus) 1.5 2.3

(ii) Installation of an assembly plant was completed in December 2012 at a cost of Rs. 60 million and it was ready for
use on 1 February 2013. However, depreciation for the year ended 31 December 2013 amounting to Rs. 4.5
million was worked out from the date of production i.e. 1 April 2013. The mistake was corrected by adjusting the
profit and loss account for the year ended 31 December 2014.
(iii) Shareholders’ equity as at 1 January 2013 was as follows:

Rs. In million
Share Capital (Rs. 100 Each) 200
Retained Earning 45
Revaluation surplus 100

On 30 November 2014, POL issued 25% right shares to its ordinary shareholders at Rs. 120 per share.

(iv) Cash dividend and bonuses declared/paid during the last three years:

Final *Interim
For the year ended
Cash Bonus Cash Bonus
31 December 2012 - 15% 16% -
31 December 2013 18% - 20% -
31 December 2014 - 25% - 10%

*Declared with half yearly accounts

Page 11 of 33
130
Required:
Prepare Statement of Changes in Equity for the year ended 31 December 2014 in accordance with the requirements of
the Companies Act 2017 and International Financial Reporting Standards. (15)
(Ignore taxation)

Q.5 For the purpose of preparation of statement of changes in equity for the year ended 31 December 2017, Daffodil
Limited (DL) has extracted the following information:

2017 2016 2015


Draft Audited Audited
--------- Rs. in million ---------
Net profit 650 318 214
Transfer to general reserves 112 - 141
Transfer of incremental depreciation - 49 55
Final cash dividend - - 7.5%

Additional information:
1. Details of share issues:
i. 25% right shares were issued on 1 May 2016 at Rs. 18 per share. The market price per
share immediately before the entitlement date was also Rs. 18 per share.
ii. A bonus issue of 10% was made on 1 April 2017 as final dividend for 2016.
iii. 50 million right shares were issued on 1 July 2017 at Rs. 15 per share. The market price
per share immediately before the entitlement date was Rs. 25 per share.
iv. A bonus issue of 15% was made on 1 September 2017 as interim dividend.

2. After preparing draft financial statements, it was discovered that depreciation on a plant costing Rs. 700
million has been charged @ 25% under reducing balance method, from the date of commencement of
manufacturing i.e. 1 July 2014. However, the plant was available for use on 1 February 2014.

3. Share capital and reserves as at 31 December:


2015 2014
------ Rs. in million ------
Ordinary share capital (Rs. 10 each) 1,600 1,600
General reserves 1,850 1,709
Retained earnings 1,430 1,302

4. Balance on revaluation surplus as on 01.01.2016 was Rs.100 million.


Required:

Prepare DL’s statement of changes in equity for the year ended 31 December 2017 along
with comparative figures. (Ignore taxation) (14)

Page 12 of 33
131
Solutions
A.1
Mohani Manufacturing Limited
Statement of Changes in Equity (Extracts)
For the year ended 31-12-2008

Retained Earning
Balance as on 1-1-2007 (89-21) 68
Effect of Change in Policy (1.5)
Balance as on 1-1-2007 Restated 66.5
Profit – 2007 as restated 39.7
Balance as on 31-12-2007-Restated 106.2
Profit – 2008 8.8
Balance as on 31-12-2008 115

Workings:

W-1 Effect of correction of error on profit:


2008 2007 2006
Opening Stock (2.2) 1.5 -
Closing Stock (4) 2.2 (1.5)

(6.2) 3.7 (1.5)

W-2
Statement of Comprehensive Income (Extracts)
For the year ended 31-12-2008
2008 2007
Profit as Given 15 21
Effect of error in depreciation 0 15
Effect of policy change (W-1) (6.2) 3.7
Profit as restated 8.8 39.7

A.2 a)

i) Depreciation journal: Debit Credit

Depreciation 50 000
Accumulated depreciation: equipment 50 000
Depreciation of equipment

ii) Adjustment to depreciation journal:

Accumulated depreciation: equipment 50 000

Depreciation 50 000
Adjustment to depreciation of equipment

Working
Date of purchase 1-4-2001
Cost of purchase Rs 600,000
Depreciation upto 31-3-2003 is 600,000/6 x 2 = Rs 200,000
WDV= 600,000 – 200,000/8 = 50,000

Page 13 of 33
132
A.3 Solution

Wonder Limited
a) Extracts of Statement of financial position
For the year ended 30 June 2012
2012 2011
(Restated)
Rs. in million
Property, plant and equipment 178.50 111.50
Retained earnings-SOCE 187.5 89.50
PPE: Year2012:[189 – 18 . 5+2+6] PPE: Year 2011:(130 -18.5)

b) Wonder Limited
Extracts of Statement of profit or loss
For the year ended 30 June 2012
2012 2011
(Restated)
Rs. in million
Profit for the year 98.00 101.50
Profit for the year: Year 2012: [90+2+6]
Profit for the year: Year 2011: [120-18.5]

c) Wonder Limited
Extracts of statement of changes in equity
For the year ended 30 June 2012
Retained earnings
Rs. in million
Balance as on 1 July 2010 (108-120) (12.00)
Profit for the year ended 30 June 2011-restated 101.50
Balance as at 30 June 2011 - restated 89.50
Profit for the year ended 30 June 2012 98.00
Balance as at 30-June 2012 187.5

d) Wonder Limited
Notes to the financial statements
For the year ended 31 December 2012
Correction of error
During the year ended 30 June 2011, the repair works was erroneously debited to machinery account. The effect of this
error is as follows:
2011
Rs. in million
Effect on the income statement
Increase in Repairs and maintenance (20.00)
Decrease in Depreciation 1.50
Decrease in profit for the year (18.50)
Effect on the statement of financial position
Decrease in Property, plant and equipment (18.50)
Decrease in retained earnings (18.50)

Page 14 of 33
133
Workings
Effect of error
1-10-2010 Repair capitalized to machinery 20M
During the year ended 30-6-2011

Decrease in Profit (20M) Repair 20M


Machinery 20M
Increase in Profit 1.5M Acc Depreciation 1.5M
(20M ÷ 10 x 9/12) Depreciation 1.5M
Net Decrease in Profit (18.5)

During the year ended 30-6-2012


Increase in Profit 2M Acc Depreciation 2M
Depreciation 2M
Net Increase in Profit 2M

Effect of changes in Estimates


During the year ended 30-6-2012
Cost 70M
Acc depreciation (70 ÷ 5) 14M
WDV as on 30-6-2011 56M
Depreciation to be charged
(56M÷7) 8M
Depreciation already charged 14M
Depreciation to be reversed 6M Acc Depreciation 6M
Depreciation 6M

A.4 Solution
Fine Fibre Limited
a) Statement of Changes in Equity for the year ended 30 June 2013 (extracts)

Retained
Earnings

Rs, in million
Balance as at 1 July 2011 (W-1) 38.00
Final cash dividend for the year ended 30 June 2011 @ Rs. 5 per share (30÷ 10x5) (15.00)
Profit after tax for the year ended 30 June 2012 - restated (W. a) 91.00
Balance as at 30 June 2012 - restated 114.00
Final cash dividend for the year ended 30 June 2012 @ Rs. 4 per share (30÷10X4) (12.00)
Profit after tax for the year ended 30 June 2013 (W. a) 91.20
Balance as at 30 June 2013 193.20

W. a : Corrected profit after tax 2013 2012


Profit after tax before correction 93.00 82.00
Depreciation excess/(short) charged [W] (1.80) 9.00
91.20 91.00

Page 15 of 33
134
(b) Fine Fiber Limited
Notes to the financial statements for the year ended 30 June 2013 (extracts)
1 Correction of error
During the year ended 30 June 2012, accounting depreciation was incorrectly accounted for at Rs. 21.8 million instead of
Rs. 12.8 million. The effect of the correction is as under:
2012
Rs. in million
Effect on the statement of comprehensive income
Decrease in depreciation expense 9.00
Increase in Profit for the year 9.00
Effect on the statement of financial position
Increase in property, plant and equipment 9.00
Increase in retained earnings (114-105) 9.00

Workings
1. Calculation of opening retained earnings as on 1-7-2011
Retained Earnings
Dividend (3x5) 15 b/d (Bal) 38
Profit 82

c/d 105
Dividend (3x4) 12 b/d 105
Profit 93
c/d 186

2. Accounting adjustment in 2012


Acc Depreciation 9
Depreciation 9

3. Accounting adjustments in 2013:


In this period, accounting depreciation would have been less recorded because of less WDV at the beginning of
this period because of wrongly charging extra depreciation in previous period.
Depreciation 1.8
Acc Depreciation 1.8
(9 x 20%)

A.5 Solution

Zain Pharma Limited


Accounting entries for patents

Debit Credit
Date Description
(Rs. in million)
1-1-2013 Accumulated amortization/Intangible assets W-4 4.80
Retained earnings W-4 4.80
(Amortization expense over charged for the period up to 31
December 2012)

31-12-2013 Amortization expense (P&L account) W-2 6.60


Acc. amortization/Intangible assets W-2
6.60
(Amortization expense for the year based on sales)

31-12-2013 Impairment (P&L account) W-3 2.20


Intangible assets/Acc Impairment loss 2.20
(Impairment of patent product B as at 31 December 2013)

Page 16 of 33
135
Workings
W-1: Patents' amortization as recorded upto 31-12-2012 Total
(based on number of years) Product-A Product-B (Rs. in
million)
Date of purchase 1-Jan-2011 1-Jul-1998
Acquisition cost of patents A 37.50 36.00
Estimated life of the patents B 15 years 18 years
No. of years upto 31-12-2012 C 2 years 14.5 years
Amortization upto 31-12-2012 D (A ÷ B x C) 5.00 29.00

W-2: Patents’ amortization as corrected (based on sales)


Sales upto 31-12-2012 E 800.00 11,200.00
Sales for the year ended 31-12-2013 F 600.00 1,600.00
Future estimated sales 6,100.00 3,200.00
Total sales G 7,500.00 16,000.00
Amortization upto 31-12-2012 H (A ÷ G x E) 4.00 25.20
Amortization for the year ended 31-12-2013 J (A÷ G x F) 3.00 3.60 6.60
K (H+J) 7.00 28.80

W-3: Impairment as at 31-12-2013


Carrying value as at 31-12-2013 A-K 30.50 7.20
Recoverable amount as at 31-12-2013 35.00 5.00
Impairment loss - (2.20) (2.20)

W-4 : Amortization erroneously over booked (1.00) (3.80) (4.80)


(H-D) (5-4) (29-25.2)
upto 31-12-2012

A.6

SCCL
Statement of Changes in Equity
For the Year ended 31-12-2012
Rs in millions
Share Share Unappropriated Total
Capital Premium Profit
Balance as on 1-1-2011 7,833 - 4,508 12,341

Final Bonus Shares-2010 (20%) 1,567 - (1,567) -


(7833x20%)
Interim Bonus Shares-2011(10%) 940 - (940) -
(7,833+1,567)x10%
Profit for the year –Restated (W) - - 4,975.50 4,975.50
Balance as on 31-12-2011 10,340 - 6,976.50 17,316.50

Final Bonus Shares-2011 (10%) 1034 (1034) -


(10,340x10%)
Final Cash Dividend-2011 (15%) (1,551) (1,551)
(10,340x15%)
Interim bonus Shares-2012(5%) 569 (569) -
(10,340+1034) x 5%
*Issue of Right Shares 9,554 7,643 - 17,197
Profit for the year (W) 5,083.70 5,083.70

Page 17 of 33
136
Balance as on 31-12-2012 21,497 7,643 8,,906.20 38,046.2

*Right share=(10,340+1034+569)=11,943÷10 x 4/5 = 955.4 m shares


Share capital = 955.4 x 10 = 9,554
Share Premium = 955.4 x 8 =7,643
Workings
For 2012 2011
Machine 35
Repair 35
Depreciation 6.3 Depreciation 3.5
Machine 6.3 Machine 3.5 35x20%x6/12
(35-3.5)x20%

W-2 Effect on Profit for the year:

2012 2011
5,090 4,944
- 35
(6.3) (3.5)
5,083.70 4,975.50

Page 18 of 33
137
Solution of Self-Practice Questions

A.1 Solution
a) M LIMITED
STATEMENT OF COMPREHENSIVE INCOME (Extracts)
FOR THE YEAR ENDED 31 DECEMBER 2006
2006 2005

Revenue Given 800 000 650 000


Profit for the year (329 000 - 28 800) 300 200 236 000

M LIMITED
STATEMENT OF CHANGES IN EQUITY (Extracts)
FOR THE YEAR ENDED 31 DECEMBER 2006

Retained
Earnings

Balance 31/12/2004 (40 000)


Profit for the year: 2005 236 000
Dividends (10 000)

Balance 31/12/2005 186 000

Profit for the year: 2006 300 200


Dividends (15 000)

Balance 31/12/2006 471 200

Workings
W1 Change in estimate - re-allocation method
There are two estimates that have changed:
• The estimated residual value has been changed (from Rs 0 to Rs 16 000); and
• The estimated pattern of consumption of the asset/ method of depreciation (from reducing balance method to
straight-line method).

Date Calculations Was Is Difference


Cost 1/1/2003 500 000
Depreciation 2003 (500 000 x 0.20) (100 000)
Carrying amount 31/12/2003 400 000
Depreciation 2004 (400 000 x 0.20) (80 000)
Carrying amount 31/12/2004 320 000
Depreciation 2005 (320 000 x 0.20) (64 000)
Carrying amount 31/12/2005 256 000 256 000
Residual value 0 (16 000)
Depreciable amount 256 000 240 000
Rate or remaining useful life 20% 3 years
Depreciation 2006 (256 000 x 0.20) (240 000 / 3) (51 200) (80 000) (28 800)
increase in
depreciation
Carrying amount 31/12/2006 204 800 176 000 (28 800)
Future depreciation Was: (204,800 – 0) 204,800 160,000 44,800
Is: (176,000 – 16,000) decrease in
depreciation

Page 19 of 33
138
Accounting Entries
i) Depreciation 28,800
Acc Depreciation 28,800

b) M LIMITED
NOTES TO THE FINANCIAL STATEMENTS FOR THE YEAR ENDED 31 DECEMBER 2006
1.Change in estimate
The company has changed the method of estimating depreciation on plant from the reducing balance to the straight-line
method and changed the estimated residual value from Rs 0 to Rs 16 000.

The (increase)/ decrease caused by the change in estimate is as follows:


Figures from working 1
• Decrease in Current profits 28 800
• Increase in Future profits 44 800

A. 2(a)
Chand Paints Limited
Notes to the Financial Statements
For the year ended 30 June 2016
The effect of retrospective restatement on statement of comprehensive income is tabulated below:

2015
Increase / (decrease) in income
Rs. In million
Increase in sales tax, commission and discounts (7,916 – 8,246) (330)
Decrease in cost of sales (44,633 – 44,606) 27
Increase in selling and distribution expenses (2,441 – 2,635) (194)
Increase in administration expenses (2,149 – 2,254) (105)
Decrease in other operating charges (515 – 467) 48
Decrease in other operating income (509 – 427) (82)
Decrease in profit for the year (3,723 – 4,359) (636)

The effect of retrospective restatement on statement of financial position for 2015 is tabulated below:

Decrease in trade debts (2,025 – 1,613) (412)


Increase in trade and other payables (8,894 – 8,670) (224)
Decrease in un-appropriated profit (636)

Page 20 of 33
139
A. 2(b)
Chand Paints Limited
Statement of changes in equity
For the year ended 30 June 2016

Retained
Share Capital Total
Earnings
----------- Rs. In million ----------
Balance as on 1 July 2014 10,400 19,089 29,489
Interim dividend for the year ended 30 June 2015 (10,400 × 5) (520) (520)
Total comprehensive income for the year 2015 – restated 3,723 3,723
Balance as at 30 June 2015 - restated 10,400 22,292 32,692
Final dividend for the year ended 30 June 2015 (10,400 × 10%) (1,040) (1,040)
Interim cash dividend for the year 2016 (10,400 × 10%) (1,040) (1,040)
Total comprehensive income for the year ended 30 June 2016 4,089 4,089
Balance as at 30 June 2016 10,400 24,301 34,701

A. 3
(a) Asif Engineering Ltd.
Extracts from statement of financial position

2014 2013
2015
(Restated) (Restated)
-------------- Rs. In million ---------------
Property, plant & equipment (W-2) 2,498 2,461 2,149
Stores and spares – Given 73 80 70
Retained earnings 421 283 145

(b) Extracts from statement of comprehensive income

2015 2014 (Restated)


------------- Rs. In million ------------
Net profit (W-1) 138 138

(c) Extracts from statement of changes in equity

Retained earnings Rs.


In million
Balance as at 1 January 2014 101
Effect of retrospective change in accounting policy (W-1) 44
Balance at 1 January 2014 – restated 145
Total comprehensive income – 2014 (W-1) 138
Balance as at 1 January 2015 – (restated) 283
Total comprehensive income – 2015 138
Balance as at 31 December 2015 421

Page 21 of 33
140
(W-1) Computation of net profit
Depreciation expense for the year
2013 2014 2015
-------------- Rs. In million ---------------
Depreciation for 2013 (55÷5) 11 11 11
Depreciation for 2014 (39÷3) - 13 13
Depreciation for 2015 (44÷4) - - 11
11 24 35
Less: Amount already charged 55 39 44
Adjustment to be made in net profit 44 15 9
Profit for the year – Given 123 129
Adjusted profit for the year 138 138

(W-2) Property, plant and equipment

2013 2014 2015


-------------- Rs. In million ---------------
As given 2,105 2,402 2,430
Add: Stores issued 2013 55 55 55
Add: Stores issued 2014 0 39 39
Add: Stores issued 2015 0 0 44
Less: Accumulated depreciation as calculated above 2014:

11 + 24; 2015: 11 + 24 + 35 (11) (35) (70)


Revised carrying value 2,149 2,461 2,498
A.4
PAK OCEAN LIMITED
Statement of changes in Equity
For the year ended 31-12-2014
“Rs In Millions”
Share Share Retained Revaluation Total
Capital Premium Earnings Surplus
Balance as on 1-1-2013 200 - 45 100 345
Bonus Share-2012 (200 X 15%) 30 (30) -
Interim Cash dividend-2013 (230 X 20%) (46) (46)
Profit after tax-Restated -(W-1) 46 46
Transfer of Surplus 2.3 (2.3) -
Balance as on 1-1-2014 (-Restated) 230 - 17.3 97.7 345
Final cash dividend-2013 (230 X 18%) (41.4) (41.4)
Interim bonus-2014(230 X 10%) 23 (23) -
Issue of right shares- (W-2) 63.25 12.65 - 75.90
Profit after tax 91 91
Transfer of surplus 1.5 (1.5)
Balance as on 31-12-2014 316.25 12.65 45.4 96.2 470.5

W-1 Prior period error


2014 2013
Profit After tax-Given 90 47
Effect of depreciation* 1 (1)
Profit after tax [adjusted] 91 46

4.5/9 X 2 = 1 (effect of depreciation of Feb & March 2013 to be reversed in 2014 and charged in 2013.

Page 22 of 33
141
W-2) [230+23] /100 = 2.53 M shares X 25 % = 0.6325 M shares
0.6325M X 100 = 63.25
0.6325 M X 20 = 12.65
Ans.5 Daffodil Limited
Statement of changes in equity
For the year ended 31 December 2017
Ordinary Share General Retained Revaluation Total
Share
capital premium reserves earnings surplus
-------------------- Rs. in million --------------------
Balance as at 31
December 2015 (As
given) 1,600.00 1,850.00 1,430.00 100 4,980.00
Effect of correction of
error (W-1) (54.69) (54.69)
Balance as at 31
December 2015 –
Restated 1,600.00 1,850.00 1,375.31 4,925.31
Final cash dividend @
7.5% - 2015
(1,600×7.5%) (120.00) (120.00)
Right issue @ 25% 400.00 320.00 720.00
(1,600×25%) (160×25%×8)
Net profit - 2016 -
Restated[318+13.67(W-
1)] 331.67 331.67
Transfer of incremental
depreciation 49.00 (49.00) 49.00
Balance as at 31
December 2016 -
Restated 2,000.00 320.00 1,850.00 1,635.98 51 5,856.98
Final bonus dividend @
10% - 2016
(2,000×10%) 200.00 (200.00) -
Right issue 500.00 250.00 750.00
(50×10) (50×5)
Interim bonus dividend
@ 15% - 2017
(2,700×15%) 405.00 (405) -
Net profit - 2017 [650 +
10.25 (W-1)] 660.25 660.25
Transfer to general
reserves 112.00 (112.00) -
Balance as at 31
December 2017 3,105.00 570 1,962.00 1,579.22 51 7,267.23

W-1: Correction of error


Correct Wrong Increase/(decrease)
depreciation @ 25% depreciation @ 25% in depreciation
-------------------------- Rs. in million --------------------------

Cost 700 700


2014 160.42 87.50 72.92
(700 × 25% × 11 ÷ 12) (700 × 25% × 6 ÷ 12)
2015 134.90 153.13 (18.23)
(700 – 160.42) × 25% (700 – 87.50) × 25%
54.69
2016 101.17* 114.84** (13.67)

Page 23 of 33
142
(134.90 × 75%) (153.13 × 75%)
2017 75.88*** 86.13**** (10.25)
(101.17 × 75%) (114.84 × 75%)

*(700 – 160.42 – 134.9) x 25%


**(700 – 87.5 – 153.13) x 25%
***(700 – 160.42 – 134.9 – 101.17) x 25%
****(700 – 87.5 – 153.13 – 114.84) x 25%
ICAP study text

Page 24 of 33
143
Test
Q.1 Following information has been extracted from the draft financial statements of Marvellous Limited (ML)
for the year ended 30 June 2017:

Statement of financial position


2017 2016
Rs. in million
Property, plant and equipment 700 612
Retained earnings 330 265

Statement of profit or loss


Profit for the year 65 85

The following matters are under consideration of the management:

• It was identified that ML has incorrectly charged Rs. 36.75 millions as maintenance expenses, incurred
on installation of plant. The plant was available for use on 01.07.2014 and has been depreciated on
straight line basis over a useful life of four year.
• In view of significant change in the expected pattern of economic benefits from an item of the
equipment, it has been decided to change the depreciation method from reducing balance to straight
line. The equipment was purchased on 1 July 2015 at a cost of Rs. 80 million having estimated useful
life of 5 years and residual value of Rs. 16 million. The depreciation at the rate of 27.5% on reducing
balance method is included in the above draft financial statements.

The following balances pertain to ML’s statement of financial position as on 30 June 2015:

Rs. in million
Property, plant and equipment 650
Retained earnings 180

Required:
Prepare extracts from the following (including comparative figures) for the year ended 30 June 2017:
a) Statement of financial position
b) Statement of profit or loss
c) Correction of error note

Page 25 of 33
144
A. 1
Marvellous Ltd.
Statement of Financial Position (Extracts)
As on 30-06-2017
Restated Restated
2017 2016 2015
Property , Plant & Equipment: 714.63 630.37 677.56
(2015: 650+36.75-9.19);(2016: 612+36.75-9.19 X 2)
(2017: 700+36.75-(9.19 X3) +5.45)
Retained earnings (W-3) 344.63 283.37 207.56

b) ML
Statement of profit or loss (extracts)
For the year ended 30-06-2017
Restated
2017 2016
Profit for the year 61.26 75.81
(2016: 85-9.19);(2017: 65-9.19+5.45)

c) Correction of error note


It was identified in current year that installation cost of Rs. 36.75 million to be capitalized was incorrectly
charged as maintenance expense on july 01,2014. The effects of corrections of this error as follows:
Effect on statement of profit or loss
2016 2015
Decrease in maintenance expense 0 36.75
Increase in Depreciation 9.19 9.19
Increase / Decrease in profit for the year 9.19 27.56

Effect on Statement of Financial Position:


2016 2015
Increase in P.P.E 18.37 27.56
(630.37-612) (677.56-650)
Increase in R.E 18.37 27.56
(283.37-265) (207.56-180)

Workings:
W-1)
1-07-2014 Plant 36.75
Maintenance expense 36.75

30-06- Dep. 9.19


2015
Plant 9.19
(36.75/4 =9.19)
For 30-06-2016:
30-06-2016 Dep. 9.19
Plant (36.75/4) 9.19
For 30-06-2017:
30-06-2017 Dep. 9.19
Plant (36.75/4) 9.19

145
Page 26 of 33
W- 2) Change in estimate
1-07-2015 Equipment cost 80
30-06-2016 Dep. (80 X 27.5%) (22)
30-06-2016 WDV 58
30-06-2016 Dep. (16)
30-06-2017 WDV 42

Dep. Charged during 2017 = 16


Dep. to be charged (58-16/4) = 10.5
Dep. To be reversed = 5.50

30-06-2017 Acc. Dep/ Equipment 5.45


Dep. 5.45
(16-10.5) = 5.5

(W-3) Statement of changes in equity:


Balance on 1-07-2015 180
Effect of error (36.75-9.19) 27.56
Balance as on 1-07-2015- Restated 207.56
Profit for 2016 –Restated 75.81
Balance as on 30-06-2016- Restated 283.37
Profit for 2017 61.26
Balance as on 30-06-2017 344.63

146
Page 27 of 33
Q.2
During the year, it was discovered that due to some calculation error in excel sheet, fair value of Coal limited’s
office building was taken incorrectly as Rs. 460 million instead of Rs. 360 million. Resultantly, the building was
recorded based on incorrect revaluation amount in CL’s financial statements for the year ended 30 June 2017.

This building was acquired on 1 July 2015 for Rs. 500 million and then revalued for the first time on 30 June
2017.

CL follows revaluation model for subsequent measurement of its building classified as property, plant and
equipment and charges depreciation over its useful life of 10 years using straight line method. CL accounts for
revaluation on net replacement value method and transfers the maximum possible amount from the
revaluation surplus to retained earnings on an annual basis.

As on 30 June 2019, the revalued amount of building has been determined at Rs. 320 million.

Required:
Prepare extracts from CL’s statement of financial position and related notes to the financial statements for the
year ended 30 June 2019 alongwith comparative figures for the above.
(Note on Property, plant and equipment is not required)

147
Page 28 of 33
A.2 Coal Limited
Statement of financial position
As on 30 June 2019
2019 2018 2017 2016
Non-current assets: Restated Restated Restated
------------ Rs. in million ------------
315 450
(402.5- 360
Building 320 100+12.5) (460-100)
(360-45)
Equity
- - -
Revaluation surplus (W-2) 20.00 (52.5-60+7.5) (60-60)
(185) (140) (50)
(-140-45) (-50-50-40)
(200) or-150- Or (-100-
Retained earnings (-185-45+30) 40+12.5-7.5) 40)

Correction of error note:


It was identified in current year that revalued amount of one of its buildings was taken as Rs. 460 million
instead of 360 million in 2017's financial statements of the company.

2017
Effect on the statement of profit or loss 2018
Rs. in million
Decrease in depreciation expense (100÷8) 12.50
Increase in revaluation loss 40
Effect on the statement of financial position
2018 2017
---- Rs. in million ----
[(460-57.50) – 315] or (- (87.50) (100.00)
Decrease in PPE 100+12.5) [402.5-315] [460-360]
Decrease in Revaluation surplus (-60+7.5) (52.50) (60.00)
Decrease in retained earnings (-40 +12.5 -7.5) (35.00) (40.00)

W-2
Prior Period Error
01-07-2015
Cost 500
Accumulated depreciation [500÷10×2] (100)
WDV (30.6.2017) 400

Page 29 of 33

148
Adjustment that would have been made on30.6.2017
WDV = 400
Fair value = 460
Revaluation surplus = 60

Office building 60
R. Surplus 60

Adjustment that should have been made on 30.6.2017:


WDV = 400
Fair value = 360
R. Loss = 40

Revaluation loss 40
Office building 40
Rectification on 30.6.2017
R. loss (R.E) 40
R. Surplus 60
Office building 100
Depreciation that would have been charged in 2018
460 ÷ 8 = 57.5
Depreciation that should have been charged in 2018
360 ÷ 8 = 45
Extra Depreciation of 12.5 to be reversed in 2018

Office building 12.5


Depreciation (R.E) 12.5

Transfer of surplus that would have been made in 2018:


60 ÷ 8 = 7.5
However no transfer of surplus should be made in 2018:
Retained Earning 7.5
Revaluation surplus 7.5

Page 30 of 33

149
Depreciation to be charged in 2019:
360 ÷ 8 = 45
Revaluation as on 30.6.2019
WDV [ 360- (45×2) ] = 270
Fair value = 320
Revaluation surplus = 50

Reversal of loss 40 Cr Revaluation Surplus 20


Extra dep’n to be charged 10 Dr
[50-45]×2
Net reversal of loss 30 Cr

Office building 50
Reversal of loss (PL) 30
Rev. surplus 20

Coal Limited
Original Statement of financial position
As on 30 June 2018
2018 2017 2016
Non-current assets: Restated Restated
------------ Rs. in million ------------
402.5
Building (460-57.5) 460 450

Equity
52.5
Revaluation surplus (W-2) (60-7.5) 60 0
(150) (100)
Retained earnings [-100-57.5+7.5] (-50-50) (50)

Page 31 of 33

150
Test question:
Q.1 Chand Limited (CL) was incorporated on 1 January 2020 with an authorized share capital ofRs.
500 million comprising of 50 million shares.
(i) Details of shares issued are as follows:
▪ On 1 March 2020, CL issued 20 million shares at Rs. 18 each.
▪ On 1 October 2020, CL issued 15% bonus shares. The market price per
share immediately before the announcement of bonus was Rs. 24 per
share.
▪ On 1 September 2021, CL issued 40% right shares at a premium
of Rs. 12.5 per share. The market price per share immediately before the
entitlement date was Rs. 33 per share.
(ii) Following information has been extracted from CL’s draft financial statements:
2021 2020
Draft Audited
--- Rs. in million ---
Net profit 66 48
Revaluation surplus arising during the year - 20
Transfer of incremental depreciation 4 -

Final cash dividend - 10%


(iii) After the preparation of draft financial statements for the year
ended 31 December 2021, it was discovered that installation cost of Rs. 12
million relating to a plant capitalized on 1 August 2020 was wrongly expensed
out. The plant is subsequently measured using cost model and is being
depreciated @ 20% per annum on reducing balance method.

Required:
Prepare CL’s statement of changes in equity for the year ended 31 December 2021
along with comparative figures. (Column for total is not required) (12)

Page 32 of 33

151
Answer:

Chand limited
Statement of changes in equity
For the year ended 31 December 2021
Rs. in millions
Share capital Share Retained Revaluation
premium earning Surplus
Balance as on 1.1.2020 - - - -
Issuance of shares 200 160 - -
(20 x 10) (20 x 8)
Bonus shares 30 (30)
(200 x 15%) = 30
Net profit-restated (W-1) (48 + 11) 59
Revaluation surplus 20
Balance as at 31-12-2020 -restated 230 130 59 20
Final dividend-2020 (230 x 10%) (23)
Right shares (W-2) 92 115
Net profit (W-1) (66 – 2.2) 63.8
Transfer Incremental depreciation 4 (4)
Balance as at 31-12-2021 322 245 103.8 16
Workings:
W-1: Correction of error:
For the year ended 31-12-2020
Plant 12
Expense 12

Depreciation 1
Accumulated Depreciation 1
(12 x 20% x 5/12) [Aug to Dec-2020]
Net effect on profit =12 - 1
=11 (Profit will increase)
For the year ended 31-12-2021:

Depreciation 2.2
Accumulated depreciation 2.2
(12 – 1 = 11 x 0.2)
Net effect on Profit (Profit will decrease by 2.2)

W-2: Right Shares: 230/10 = 23


23 x 0.4 = 9.2

X 10 x 12.5

= 92 = 115

Page 33 of 33

152
IAS 8
⯈ Example:
Whether the following are changes in accounting policies, changes in accounting estimates or
prior period errors:
i. An entity changed its accounting for land and buildings from cost model to revaluation
model.
ii. The useful life of plant was revised downwards following impairment loss.
iii. The depreciation method for furniture was changed from straight line method to
reducing balance method.
iv. The cost formula used for valuation of inventories was changed from FIFO to weighted
average.
v. It was discovered that last year company’s inventory sheets were under-casted.
vi. It was discovered that actual NRV of inventory was much lower than expected.
vii. Presentation changed from current/non-current to order of liquidity in statement of
financial position.
⯈ Answer:
i. Change in accounting policy
ii. Change in accounting estimate
iii. Change in accounting estimate
iv. Change in accounting policy
v. Prior period error
vi. Change in accounting estimate
vii. Change in accounting policy

153
Definitions: accounting treatment [IAS 8: 5]
Retrospective application is applying a new accounting policy to transactions, other events and conditions as if
that policy had always been applied.
Retrospective restatement is correcting the recognition, measurement and disclosure of amounts of elements
of financial statements as if a prior period error had never occurred.
Prospective application of a change in accounting policy and of recognising the effect of a change in an
accounting estimate, respectively, are:
a) applying the new accounting policy to transactions, other events and conditions occurring after the date
as at which the policy is changed; and
b) recognising the effect of the change in the accounting estimate in the current and future periods affected
by the change.
Applying a requirement is impracticable when the entity cannot apply it after making every reasonable effort
to do so.

Selection and application of accounting policies [IAS 8: 7 & 10 to 13]


When an IFRS specifically applies to a transaction, event or condition, the accounting policies applied to that item
shall be determined by applying the IFRS.
In absence of an IFRS that is applicable specifically, the management shall use its judgement in developing and
applying an accounting policy that results in information that is:
a) relevant to the economic decision‑making needs of users; and
b) reliable, in that the financial statements:
• represent faithfully the information;
• reflect the economic substance (not merely the legal form);
• are neutral, i.e. free from bias;
• are prudent; and
• are complete in all material respects.
In making such judgement, the management should consider following sources:
a) IFRS dealing with similar and related issues; and
b) Conceptual Framework for Financial Reporting; and
c) Recent pronouncement of other standard-setting bodies (to the extent not in conflict with afore-
mentioned sources).
Accounting policies should be applied consistently for similar transactions unless required or permitted by
IFRSs.
⯈ Example:
IAS 16 Property, plant and equipment allows the use of the cost model or the revaluation model
for measurement after recognition. This is an example of where IFRS permits categorisation of
items for which different policies may be appropriate.
However, either of the measurement model must be applied to an entire class of property, plant
and equipment consistently.
Consistency also implies that same accounting policies be applied in different accounting periods unless a change
in policy is required or permitted. For example, if an entity applies weighted average cost formula for certain
type of inventory in one period, the same policy shall be applied in subsequent periods.

154
Change in accounting estimates [IAS 8: 32 to 34, 36, 37, 39 & 40]
An accounting policy may require items in financial statements to be measured in a way that involves
measurement uncertainty i.e. monetary amounts that cannot be observed directly and must instead be estimated. The
use of reasonable estimates is an essential part of preparation of financial statements and does notundermine
their reliability.
An accounting estimate may need revision if changes occur in the circumstances on which the accounting
estimate was based or as a result of new information or more experience. A change in accounting estimate does
not relate to prior periods is not correction of prior period error.

Accounting and disclosure


Profit or loss The effect of change in an accounting estimate relating to profit or loss shall be recognised
prospectively by including it in profit or loss in:
(a) The period of change if the change affects that period only; or
(b) The period of change and future periods if the change affects both.
Assets, To the extent that a change in an accounting estimate gives rise to changes in assets and
liabilities liabilities, or relates to an item of equity, it shall be recognised by adjusting the carrying
and equity amount of the related asset, liability or equity item in the period of change.
⯈ Example:
A non-current asset was purchased for Rs. 200,000 on 1st January 2021, when its expected useful
life was ten years, and its expected residual value was nil. The asset is being depreciated using
straight-line method.
A review of the non-current assets during year 2023 revealed that due to technological change,
the useful life of the asset is only six years in total, and the asset therefore has a remaining useful
life of four years.
The original depreciation charge was Rs. 20,000 per year (Rs. 200,000/10 years) and at the
beginning of 2023, its carrying value was Rs. 160,000 [Rs. 200,000 – (Rs. 20,000 x 2 years).
The change in the estimate occurred in 2023 and should be applied prospectively, from year 2023
to onwards. The annual charge for depreciation for year 2023 (the current year) and for the
future years (2024 to 2026) will be changed from Rs. 20,000 to Rs. 40,000 (Rs. 160,000/4 years).

Comparative information [IAS 1: 10, 38, 38A & 40A]


An entity shall include comparative information for narrative and descriptive information if it is relevant to
understanding the current period’s financial statements.
An entity shall present, as a minimum, two statements of financial position, two statements of profit or loss and
other comprehensive income, two separate statements of profit or loss (if presented), two statements of cash
flows and two statements of changes in equity, and related notes.
An additional (third) statement of financial position as at the beginning of the preceding period is also required
when an entity:
a) applies an accounting policy retrospectively (IAS 8); or
b) makes a retrospective restatement of items in its financial statements (IAS 8); or
c) reclassifies items in its financial statements (IAS 1).

155
CHANGE IN ACCOUNTING POLICY
When allowed? [IAS 8: 14 & 16]
An entity shall change an accounting policy only if the change:
a) is required by an IFRS; or
b) results in the financial statements providing reliable and more relevant information (voluntary change).
However, the following are NOT changes in accounting policies:
a) the application of an accounting policy for transactions and events that differ in substance from those
previously occurring; and
b) the application of a new accounting policy for transactions and events that did not occur previously or
were immaterial.

Application of IAS 8 [IAS 8: 17 to 19]


The initial application of revaluation model under IAS 16 or IAS 38 shall be dealt in accordance with IAS 16 or
IAS 38 respectively, and not in accordance with IAS 8.
The initial application of an IFRS may result in change in accounting policy, which should be accounted for in
accordance with Transitional Provisions of that IFRS.
If the IFRS does not include any transitional provisions or the change in accounting policy is voluntary, the entity
shall apply the change retrospectively subject to certain limitations. For example, retrospective application of
accounting policy is made in accordance with IAS 8 when an entity:
a) changes measurement basis of its investment property from cost model to fair value model; or
b) changes cost formula for measurement of its inventory from weighted average to first-in, first-out (FIFO)
basis.

Retrospective application [IAS 8: 19, 22 & 23]


Change in accounting policy shall be applied retrospectively (as if the new accounting policy had always been
applied) except to the extent that it is impracticable to determine either period specific effects or cumulative
effect of change.
Retrospective application has three steps:
a) Apply new policy in current period (e.g., year 2023 is current period).
b) Apply new policy in comparative period as if the new accounting policy had always been applied (e.g.,
year 2022 is comparative period presented).
c) For periods before the comparative period, adjust the opening balances of earliest comparative period
of each affected component of equity and related asset or liability as if the new accounting policy had
always been applied (e.g., opening balances of year 2022 shall be adjusted).
⯈ Example:
A company presents comparatives for the previous year only.
During the year ended 31 December 2023 it changes an accounting policy, and this change must
be applied retrospectively.
If there were no change in accounting policy the company would present statements of financial
position as at December 2023 and December 2022 only.
However, because there is a change in policy the company must also present a statement of
financial position as at 1 January 2022 (the beginning of the earliest comparative period).

156
The change in accounting policy is applied retrospectively. This means that the change should be
applied to the balances at as at 1 January 2022 as if the new policy had always been applied.
Similarly, any other comparative amounts in previous periods should be adjusted as if the new
accounting policy had always been applied.
Limitation to retrospective application [IAS 8: 23 to 25]
When it is impracticable to determine the period specific effects, the entity shall apply the new accounting policy
retrospectively from the earliest date practicable.
When it is impracticable to determine the cumulative effect, at the beginning of the current period, of applying a
new accounting policy to all prior periods, the entity shall adjust the comparative information to apply the new
accounting policy prospectively from the earliest date practicable.
Disclosure [IAS 8: 28 & 29]
When a change in accounting policy has an effect on the current period or any prior period (or would have an
affected that period except that it is impracticable to determine the amount of the adjustment) or might have an
effect on future periods the following must be disclosed:

Change due Voluntary


Disclosure:
to IFRS change
The title of the IFRS. ✓
The nature of the change in accounting policy. ✓ ✓
When applicable, that the change in accounting policy is made in accordance ✓
with its transitional provisions and a description of those transitional
provisions and their effect on future periods.
The reason why the new accounting policy provides reliable and more relevant ✓
information.
For the current and previous period(s), to the extent practicable, the amount ✓ ✓
of the adjustment for each financial statement line item affected and EPS (if IAS
33 is applicable).
To the extent practicable, the adjustment relating to accounting periods before ✓ ✓
those presented.
If retrospective application is impracticable, the circumstances that led to ✓ ✓
existence of that condition and a description of how and from when the change
in accounting policy has been applied.

The financial statements of subsequent periods need not repeat these disclosures.

157
CORRECTION OF PRIOR PERIOD ERRORS
Prior period errors [IAS 8: 41]
Errors can arise in respect of the recognition, measurement, presentation or disclosure of elements of financial
statements. Financial statements do not comply with IFRSs if they contain either material errors or immaterial
errors made intentionally to achieve a particular presentation of an entity’s financial position, financial
performance or cash flows.
Potential current period errors discovered in that period are corrected before the financial statements are
authorised for issue. Material errors are sometimes not discovered until a subsequent period, and these prior
period errors are corrected in the comparative information presented in the financial statements for that
subsequent period.

AT A GLANCE
Retrospective restatement [IAS 8: 42 & 43]
Unless Impracticable, an entity shall correct material prior period errors retrospectively in the first set of
financial statements authorised for issue after their discovery by:
• restating the comparative amounts for the prior period(s) presented in which the error occurred; or
• if the error occurred before the earliest prior period presented, restating the opening balances of assets,
liabilities and equity for the earliest prior period presented.
⯈ Limitations on retrospective restatement [IAS 8: 44 & 45]
When it is impracticable to determine the period‑ specific effects of an error on comparative information for one
or more prior periods presented, the entity shall restate the opening balances of assets, liabilities and equity for
the earliest period for which retrospective restatement is practicable (which may be the current period).
When it is impracticable to determine the cumulative effect, at the beginning of the current period, of an error

AT A GLANCE
on all prior periods, the entity shall restate the comparative information to correct the error prospectively from
the earliest date practicable.

Disclosure [IAS 8: 49]


An entity shall disclose the following:
a) the nature of the prior period error;
b) for each prior period presented, to the extent practicable, the amount of the correction for each financial
statement line item and EPS (if IAS 33 is applicable);
c) the amount of the correction at the beginning of earliest prior period presented; and
d) if retrospective restatement is impracticable for a particular prior period, the circumstances that led to

SPOTLIGHT
the existence of that condition and a description of how and from when the error has been corrected.
Financial statements of subsequent periods need not repeat these disclosures.

158
⯈ Example:
Retained earnings column extracted from the draft statement of changes in equity of Zahidi
Limited (ZL) for the year ended 31 December 2020, is as follows:

Retained earnings
Rs. in million
Balance as at 31 December 2018 351
Final cash dividend for the year 2018 (15)
Total comprehensive income for the year 2019 68
Balance as at 31 December 2019 404

AT A GLANCE
Total comprehensive income for the year 2020 82
Balance as at 31 December 2020 486

On 1 January 2018, ZL had acquired a building at cost of Rs. 200 million and had rented it out on
the same day for three years. On 31 December 2020, the tenant vacated the building and ZL
decided to transfer its head office to such building.
The finance manager was considering the reporting implications of change in use of the building.
He came to know that the building has erroneously been reported as property, plant and
equipment since inception and was being depreciated on straight line basis over 20 years. The
fair value of the building has increased by 10% in each year since acquisition.
ZL follows cost model for property, plant and equipment and fair value model for investment

SPOTLIGHT
property.
Required: Prepare the following extracts from ZL’s financial statements for the year ended 31
December 2020 in accordance with IFRSs:
a) Correction of error note
b) Retained earnings column as would appear in the statement of changes in equity.
(Show comparative figures)
⯈ Answer:
Part (a) Correction of error note

STICKY NOTES
It was identified in current year that an investment property was erroneously reported as
property, plant and equipment since acquisition i.e. 1 January 2018. The error has been corrected
by retrospective restatement of prior year amount which has been summarized as follows:

2019 2018
Effect on statement of financial position Rs. m Rs. m
Increase in investment property 242 220
Decrease in property, plant and equipment (180) (190)
Increase in retained earnings 62 30

Effect on statement of profit or loss


Reversal of depreciation expense 10 10

Increase in fair value gain (investment property) 22 20

Increase in profit 32 30

159
Part (b) Extracts of retained earnings

Retained
earnings
Rs. m
Balance 31 December 2018 (as reported earlier) 351
Effect of correction of prior period errors 30
Balance 31 December 2018 (restated) 381
Final cash dividend (15)
Total comprehensive income (restated) [68 + 32] 100
AT A GLANCE

Balance 31 December2019 (restated) 466


Total comprehensive income [82 + 34] 116
Balance 31 December 2024 582

Working:

2020 2019 2018


Rs. m Rs. m Rs. m
Reversal of depreciation [200/20] 10 10 10
Fair value gain @10% 24 22 20
SPOTLIGHT

Periodic effect 34 32 30

Cumulative effect 96 62 30

160
“Prayer is powerful, never doubt the power of prayers.”

Ratios
Obtaining information from the financial statements: financial ratios
Financial statements are used to make decisions. They are used by shareholders, investors, lenders, as
well as by management. The financial statements contain a large number of figures, but the figures
themselves do not necessarily have much meaning to a user of the financial statements. However, the
figures can be analyzed and interpreted by calculating financial ratios.

Financial ratios can help the user of the financial statements to assess:
▪ the financial position of the entity, and
▪ its financial performance

Users of the financial statements and their information needs


The IASB Conceptual Framework identifies several groups of people who may use financial statements:
▪ investors and potential investors
▪ lenders
▪ employees
▪ suppliers
▪ customers
▪ government and government agencies
▪ the general public
▪ management

All these groups are interested in financial performance, financial position and cash flows, but some users
are mainly interested in performance and profitability, while others may be more interested in liquidity and
gearing or other matters.

For example:
▪ A private investor needs to know whether to continue to hold shares or to sell them. He or she will
tend to be most interested in profitability ratios (such as gross and net profit margin and return on capital
employed) and investor ratios (such as earnings per share, dividend cover and price earnings
ratio).(Investor ratios are not in the syllabus)
▪ A potential acquirer/investor needs information about an entity’s profitability and probably also
information about whether or not the entity is managed efficiently. The acquirer’s management is likely
to focus on profit margins, return on capital employed, asset turnover and working capital ratios.
▪ A bank that has been approached to lend money to an entity needs to know whether it will receive
interest payments when these are due and whether the money that it lends will eventually be repaid. A
bank manager will normally be most interested in cash flows and liquidity ratios (current ratio, acid test
ratio) gearing and interest cover. A potential lender will also be interested in predicting future
performance as without sales there will be no cash.
▪ Employees are interested in fair wages and bonus linked with profitability.
▪ Management does planning and forecasting as a decision maker.
An examination question might ask you to interpret an entity’s financial statements for the benefit of specific
people or groups of people. Therefore, your analysis should focus on the needs of theusers.
❑ What do they need to know?
❑ What are they interested in?
❑ What decision do they need to make?

Page 1 of 56

161
Purpose and Importance of Financial Ratio Analysis
Financial ratio analysis helps a business in a number of ways. The importance and advantages offinancial
ratios are given below:
• Ratios help in analysing the performance trends over a long period of time.
• They also help a business to compare the financial results to those of competitors.
• Ratios assist the management in decision making.
• They also point out problem and weak areas along with the strength areas.
• Ratios help to develop relationships between different financial statement items.
• Ratios have the advantage of controlling for differences in size. For example, two businesses
may be quite different in size but can be compared in terms of profitability, liquidity, etc., by the
use of ratios.

In a question of ratio analysis; it is assumed that all accounting entries are correctly recorded in
financial statements unless otherwise mentioned.

1. Liquidity Ratios:
Liquidity means cash or access to cash readily available to meet obligations.
Sr# Particular Formula

1 Working Capital Current Asset – Current Liabilities

(Answer in Rs.)

Interpretation +Ve / -Ve shows we have Surplus / deficit funds in business to meet current liability.

2 Current Ratio Current Asset____

Current Liabilities

(Answer is in times)

The ratio indicates the ability of the company to pay its current obligation out of the
Interpretation current asset, usually 2:1 is acceptable. This ratio shows the ability of the company to
meet its obligations in ordinary course of business (i.e within next 12 months).

3 ▪ Acid test ratio; or Current assets - Inventory


▪ Liquid Ratio; or
▪ Quick Ratio. Current liabilities

(Answer is in times)

Interpretation This ratio shows the ability of the company to settle its obligations immediately. A 1:1
ratio is usually considered to be acceptable.

*Liquidity means having cash or access to cash readily available to meet obligations.

Page 2 of 56

162
2. Turnover / Efficiency / Working capital efficiency Ratios:

4 Inventory Turnover Cost of Goods Sold

*Average Inventory

* Opening + closing

(Answer is in times)

Interpretation The number of times stock has been converted into sale. The higher the ratio the better
it is. Ratio should be higher due to more sales not due to lower stock.

5 Inventory Turnover Period Average Inventory x 365

Cost of Goods Sold

(Answer in days)

Interpretation In how many days stock is converted into sales. The minimum the days, the better it is.

6 Debtor turnover Net Credit Sales

Avg Trade Debtors


(Answer is in times)

Interpretation Normally higher the turnover ratio the better it is because it signifies speedy and effective
collection.

7 Debtor Turnover Period Average trade Debtor x 365

(Collection Period) Net Credit Sales


(Answer in days)

Interpretation The number of days, company takes between date of sale and receipt of cash.
Usually time period should not exceed 3 month. However, evaluation is made in terms
of credit policy of company.

8 Creditor turnover

Net Credit Purchases

Avg Trade creditors


(Answer is in times)

Interpretation The final decision about this ratio should be on the basis of answer of next ratio.
However, if answer in times is more it means we are paying quickly otherwise late.

Page 3 of 56

163
9 Creditor Turnover Period Avg trade Creditor x 365

Net Credit Purchase


(Answer in days)

Interpretation In how many days, payment is made to creditors after purchase. Days should be
compared with credit period agreed.

Note: If there is no information of stocks then assume no stocks; or constant stocks which
therefore means that purchase = cost of sales

10 Business/Operating/Cash/working Inventory days + debtor days – Creditor days


capital Cycle

(Examples on page 6)

Interpretation The time period between the payment of cash to creditors and receipt of cash from
debtors.

Note: If answer is to be taken in weeks, then instead of 365; take 52; if answer is to be taken in months,
then take 12.

3.Profitability Ratios

11 Total Asset Turnover / net Asset Net Sales_ __


Turnover
Avg. Capital Employed
(Utilization of Total Assets)
(Answer is in times)

Interpretation How much time the sales are in relation to capital employed. The maximum the ratio,
the better it is.

12 Gross profit Ratio Gross Profit x 100 =…...%

Net Sales

(Answer in %)

Interpretation The higher the ratio, better it is.

13 Operating Profit Ratio Operating Profit x 100

(Operating profit is Profit before interest & Tax) Net Sales

(Answer in %)

Page 4 of 56

164
Interpretation Maximum the ratio, the better it is.

14 Expense Ratio

Cost of sales x 100 Expenses x 100

Sales Net Sales

Admin expense x 100 (Answer in %)

Sales

Selling expense x 100

Sales

Interpretation The lower the ratio, the better it is.

15 Net Profit after tax Ratio

Profit After tax x 100

Net Sales

(Answer in %)

Interpretation The higher the ratio, the better it is.

16 Return on Assets

Profit before interest & tax x 100

Avg total Assets


(Answer in %)

Interpretation Total assets include both current assets and non current assets. However sometimes
return on non current assets is also calculated.The higher the ratio, the better it is.

17* Return on Capital Employed (ROCE) a) Profit before interest & tax
x100
Avg Capital Employed
Long term loans (also called long term debt)
(Answer is in %)
=TFC + Debentures + Redeemable Preferences
shares + Bond + loan stock
b) Capital Employed = Equity +
Long Term Loans

c) Capital employed = Total


Assets – C.L

Page 5 of 56

165
Interpretation The ratio shows the return on total funds invested in the business. The higher the
ratio, the better it is.

18* Return on shareholders capital (Equity) or PAT – *Preference Dividend


Investment [ROSC] x100

Equity= (Share capital + Reserves) Average Equity


(Answer is in %)

*If preference shares are non-


redeemable

Interpretation The higher the ratio, the better it is.

*Note: if the question is silent regarding the preference shares then simply ignore them.

If the question is silent regarding whether the preference shares are redeemable or irredeemable then
assume these are irredeemable.

4. Debt ratio or Long term solvency ratios


Ratios calculated by financial institutions before advancing loans to customers:

19 Gearing Ratio/Debt to Equity Ratio: Gearing also called as leverage, measures the
total long term debt of a company as a percentage of either:

i) Equity of the company; or


ii) Capital employed of the company
a) Long term loans x 100
Equity
(Answer in %)
OR

b) Long term loans______ x 100


Equity + long term loans
(Answer in %)
Interpretation If company is either above 100%

[in case of (a) above] or 50%

[in case of (b) above], the company will be considered as highly geared or highly
leveraged company, otherwise low geared. If question is silent then (b) is preferable

20 Interest Cover:

Profit before interest & Tax

Interest expense
(Answer in times)

Page 6 of 56

166
Interpretation How many times profit is available against interest expense. The higher the ratio the
more confident will be loan providers to receive their interest. An interest cover ratio
of less than 3.0 times is considered very low.

Example
In a manufacturing business there are three types of inventory:
• Raw material
• Work in process
• Finished goods

Therefore inventory turnover is divided into:


1. Raw material inventory holding period (it is the time period between purchase and
consumption(usage) of raw material)

Average raw material Inventory x 365

Raw material consumed (Answer in days)

2. Work in process inventory holding period (it is the time period between consumption(usage) of raw
material and completion of finished goods or in other words, production time of finished goods)

Average work in process Inventory x 365

Cost of Goods manufactured (Answer in days)

3. Finished goods inventory holding period (it is the time period between completion of finished goods
and their sale)

Average finished goods Inventory x 365

Cost of Goods Sold (Answer in days)

Example: Business Cycle

i) Goods purchased and payment made on 1-1-2015

Goods sold on credit on 31-1-2015


Cash received from customers on 15-2-2015
Inventory turnover period 31 days
Debtor turnover period 15 days
Creditor turnover period -
Business cycle 46 days

ii) Goods purchased on 1-1-2015

Payment for goods on 20-1-2015


Goods sold on credit on 31-1-2015
Cash received from customers on 15-2-2015
Inventory turnover period 31 days
Debtor turnover period 15 days
Creditor turnover period (20) days
Business cycle 26 days

Page 7 of 56

167
Ordinary vs. preference shares
Companies issue two main types of shares:
▪ Ordinary Shares
▪ Preference Shares
• Irredeemable Preference shares
• Redeemable Preference shares

Comparison of ordinary shares and irredeemable preference shares

Ordinary shares Irredeemable Preference shares


Dividend rate Variable – higher in a good year, Fixed per annum
lower in a bad year
Dividend distribution Paid only if there are spare Receives the dividend before ordinary
funds after the payment of shareholders (therefore lower risk)
preference dividend
Liquidation The last to be repaid in a Repaid before (in preference to) the
liquidation ordinary shareholders
Voting rights Receive the right to vote on No right to vote on company decisions.
major decisions. Each ordinary
share would attract one vote.

Dividend presentation In equity In equity (before ordinary dividend)

Amount of capital In equity In equity


presentation

Comparison of redeemable preference shares and irredeemable preference shares

Irredeemable preference shares Redeemable Preference shares


Dividend rate Fixed per annum Fixed per annum

Dividend distribution Paid only if there are spare funds after Receives the dividend before
the payment of a redeemable Irredeemable preference
preference dividend in preference to shareholders and ordinary
ordinary shareholders shareholders (therefore lower risk)
Liquidation The last to be repaid in a liquidation Repaid before (in preference to)
but before ordinary shareholders. Irredeemable preference
shareholders and the ordinary
shareholders
Voting rights No right to vote on company decisions no right to vote on company
decisions

Dividend presentation In equity (before ordinary dividend) In financial charges in income


statement
Amount of capital In equity In non-current liabilities
presentation

Page 8 of 56

168
“Prayers take you near to Allah.”

Comparison of ordinary shares and redeemable preference shares

Ordinary shares Redeemable Preference shares


Dividend rate Variable – higher in a good year, Fixed per annum
lower in a bad year
Dividend distribution Paid only if there are spare funds Receives the dividend before
after the payment of preference Irredeemable preference
dividend shareholders and ordinary
shareholders (therefore lower risk)
Liquidation The last to be repaid in a liquidation Repaid before (in preference to)
Irredeemable preference
shareholders and the ordinary
shareholders
Voting rights Receive the right to vote on major no right to vote on company
decisions. Each ordinary share decisions
would attract one vote.
Dividend presentation In equity In financial charges in income
statement
Amount of capital In equity In non-current liabilities
presentation

Example
The following information pertains to Shale Distributors Limited (SDL):
Rs. in million
Sales 300
Purchases 140
Cost of goods sold 150
Trade receivables 50
Trade payables 21
Inventories 30
All the purchases and sales are on credit.

Required:
Calculate the cash operating cycle of SDL and explain briefly its significance.
(Assume a 360-day year)

Solution

Cash operating = Inventory turnover ratio period + Trade receivable turnover period – Trade
Cycle payable

= 72(W-1) + 60(W-2) – 54(W-3)

= 78 days

Page 9 of 56

169
Workings Notes

W-1

Inventory turnover period = Inventories_____ x 360

Cost of Goods Sold

= 30 x 360

150

= 72 days

W-2

Trade receivables turnover period = Trade receivables x 360

Sales

= 50 x 360

300

= 60 Days

W-3

Trade payable turnover period = Trade Payable x 360

Purchases

= 21_ x 360

140

= 54 days

Significance

It shows the time between payment of materials & recovery from debtors. The organization requires
financing for this period.

Page 10 of 56

170
“No one is worth to worship except Allah so pray to Allah and none.”

Example:

You are given the following information about Company R:


At 31 December 31-12-2015
Rs.000
Total assets 5,800
Share capital 1,200
Retained earnings 2,400
3,600
Long-term liabilities (Bank loans) 1,500
Current liabilities 700
5,800

For the year to 31 December 2015

Rs.000
Profit before interest and taxation 700
Interest (230)
Profit before tax 470
Taxation (140)
Profit after taxation 330
Required
Use this data to calculate:
a) The gearing ratio at 31 December 2015.
b) The interest cover in the year ended 31-12-2015

Answer
a)
i) Gearing = 1,500 x 100 =41.7%
3,600

Alternatively:

ii) Gearing = 1,500 x 100 = 29.4%


(3,600 + 1,500)

(The company is fairly low-geared.)

b)
Interest cover = 700 = 3.04 times
230

Page 11 of 56

171
Self-Test Questions
Q.1 Indicate the effect of the following transactions on working capital by indicating:
• No effect with justification
• Increase with justification
• Decrease with justification
1) Purchase of raw material of Rs. 50,000
2) Sale of finished goods of Rs. 65,400
3) Depreciation for the year Rs. 70,000
4) Operating expenses paid of Rs. 90,000
5) Purchased equipment of Rs. 120,000 on account
6) Paid cash Rs. 150,000 on account of payable.
7) Received Rs. 20,000 on account of receivables
8) Declared & paid a cash dividend of Rs. 40,000
9) Issued Rs. 100,000 share capital for cash.
10) Borrowed Rs. 200,000 on short term loan.
11) Dividend declared
12) Dividend declared in last year paid during the year.

Q.2 Given below is the information of a limited company


Balance Sheet
Assess Rs.
Fixed Assets at W.D.V. 1,150,000
Investments 300,000
Current Assets
Stock 310,000
Sundry Debtors 350,000
Advances 100,000
Cash and bank Balance 40,000
800,000
Total Assets
2,250,000
Capital and Liabilities
Share Capital 700,000
Unappropriated Profit (Retained Earnings) 250,000
950,000

Long Term Loans 725,000

Current Liabilities
Accounts Payable 125,000
Sundry Creditors 250,000
Accrued and other liabilities 200,000
575,000

2,250,000

Page 12 of 56

172
Page 13 of 56

173
Page 14 of 56

174
Page 15 of 56

175
Page 16 of 56

176
Page 17 of 56

177
Page 18 of 56

178
Page 19 of 56

179
Page 20 of 56

180
Page 21 of 56

181
Page 22 of 56

182
A.6

Measures
S. No. Ratios
(i) (ii) (iii) (iv) (v)
(a) Gross profit % Increase Decrease Decrease No effect No effect
(b) Net profit % Increase Decrease Decrease Increase Increase
(c) Current ratio Increase No effect Decrease Increase Increase
(d) Stock turnover (times) Decrease Increase No effect Increase No effect
(e) Return on non-current assets Increase No effect Decrease Increase Increase
(f) Quick ratio Increase Increase Decrease Increase Increase
Workings:

Suppose Existing Figures are:

a) Statement of Profit or loss

Rs. In’‘000’’
Sales 1,000
Cost of sales (800)
Gross profit 200
Expenses (150)
Net profit 50
b) Statement of Financial Position
Non-current assets 500
Current assets :
Stock 225
Debtors 100
Cash 50
375
Total Assets 875
Equity & Liabilities
Equity: Share Capital 300
Retained Earnings 125
425
Non-Current Liabilities: 150
Current Liabilities:
Creditors 150
Bank Over Draft 150
300
Total Equity & Liabilities: 875

(C) Existing Ratios:

Gross Profit = 200/1000 x 100 = 20%


Net Profit = 50/1000 x 100 = 5%
Current Ratio = 375/300 = 1.25 Times
Stock Turn Over = 800/225 = 3.56 times
Return on Non-Current Assets 50/500 x 100 = 10%
Quick Ratio = 375 - 225/300 = 0.5 times

Page 23 of 56

183
Analysis:

(1)

Page 24 of 56

184
a)Gross Profit = (200+100-80)/(1000+100)X100 = 20% no effect
b)Net Profit = (50+100-80)/ (1000+100)X100 = 6.36% Increase
c)Current Ratio = (375+100-80)/300 = 1.32 times increase
d)Stock Turn Over = (800+80)/(225-80) = 6.07 times increase
e)Return on Non-Current Assets = (50+100-80)/500X100 = 14% increase
f)Quick Ratio = {( 375+100-80) -(225-80)or(100+50+100)}/300 0.83 times increase
5) Cash 100

Non-Current Asset 90

Gain (P.L) 10

a)Gross Profit = (200/1000)x100 = 20% no effect


b)Net Profit = (50+10)/1000x100 = 6% increase
c)Current Ratio = (375+100)/300 = 1.58 times increase
d)Stock Turn Over = 800/225 = 3.56 times no effect
e)Return on Non-Current Assets = (50+10)/(500-90)x100 = 14.63% increase
f)Quick Ratio = (375+100)-225/300 = 0.83 times increase

Window dressing

Page 25 of 56

185
Page 26 of 56

186
Page 27 of 56

187
Page 28 of 56

188
Page 29 of 56

189
Page 30 of 56

190
Page 31 of 56

191
Page 32 of 56

192
Page 33 of 56

193
Page 34 of 56

194
Page 35 of 56

195
Page 36 of 56

196
Page 37 of 56

197
Page 38 of 56

198
(b) Liquidity Ratio

Page 39 of 56

199
Page 40 of 56

200
Extra Practice Questions:
Q.1 Boom Limited (BL) is a manufacturer of sports goods. Following financial statements for the year
ended 31 December 2017 have been submitted to the Chief Executive Officer (CEO).

Statement of profit or loss

Rs. in ‘000
Revenues 21,000
Cost of sales (17,500)
Gross profit 3,500
Operating expenses (1,900)
Finance cost (450)
Profit before tax 1,150
Taxation (345)
Profit after tax 805

Statement of financial position

Rs. in ‘000
Property, plant and equipment 7,500
Current assets 1,500
9,000

Share capital 4,000


Reserves 1,000
Non-current liabilities 3,000
Current liabilities 1,000
9,000

Although performance of BL has improved from the last year, CEO wants to compare the results with other
companies operating in sports manufacturing industry. In this respect, following industry data has been
gathered:

Gross profit margin 23.5%


Net profit margin 7.7%
Current ratio 2.75
Gearing ratio 50:50
Return on non-current asset 32.9%
Return on capital employed 27.4%
Return on equity 31.3%

Required:
(a) Compute BL’s ratios for comparison with the industry. (04)
(b) For each ratio, give one possible reason for variation from the industry. (07)

Page 41 of 56

201
“When prayer become part of your life then success becomes lifestyle.”

Ans.1 Boom Limited

Comparison of BL's ratios with industry average and possible reasons for variation

Ratios (a) BL's ratios Industry's ratios


Gross profit 16.67% (3,500/21,000 x100)
margin 23.50%
Net profit margin 3.83%(805/21,000 x100) 7.70%
Current ratio 1.5(1,500/1,000) 2.75
Gearing ratio 37.5:62.5(3,000/(4,000+1,000+3,000) x100) 50:50
Return on non- 21.33%(1,150+450)/7,500 x100) 32.90%
current assets
Return on capital 20.00%(1,150+450)/(4,000+1,000+3,000) 27.40%
employed x100)
Return on Equity 16.10%(805/(4,000+1,000) x100) 31.30%

(b) Reasons for variation from industry


Gross profit margin:
Lower than industry
• Purchase of raw material at higher prices as compared to its competitors
• Inability to obtain economies of scale in Production as compared to its competitors
• Higher production costs due to inefficiencies
• Deliberately keeping selling prices lower

Net profit margin


Lower than industry (mainly due to lower gross profit)
BL’s gross profit margin is 6.83% lower than industry (16.67% Vs 23.5%) whereas net profit margin is
only 3.87% (7.7%-3.83%) lower which indicates that BL’s operating expenses as a percentage of
sales are approximately 2.96% (6.83%-3.87%) lower than the Industry(means problem is not in operating
expenses)
Current ratio
Lower than industry
Since gearing ratio is lower than the industry so BL might have:

• obtained running finances as compared to long-term financing by the industry


• availed extended credit terms from Suppliers (means creditors are more)
• Low inventory levels are maintained by BL
• Shorter credit terms are given to debtors

Gearing ratio
Lower than industry
• Difficulty in raising long-term finance from banks due to low profits
• Running finance or extended credit terms from suppliers are available for BL

Return on non- current assets


Lower than industry
• Lower profit margins
• Relatively newer non-current assets have higher carrying value

Page 42 of 56

202
Return on capital employed
Lower than industry
• Lower profit margins
• High shareholder’s equity

Return on Equity
Lower than industry
• Lower profit margins
• High shareholder’s equity

Page 43 of 56

203
“Perform your prayers (salat) timely and seek help from Allah through Prayer.”

Q.2 Following are the summarised financial statements of Keyboard Limited (KL):

Statement of financial position


2018 2017 2016
----------- Rs. in '000 -----------
Fixed assets 12,500 10,800 11,800

Current assets:
Inventory 4,000 4,500 3,000
Debtors 4,200 3,200 1,800
Cash - 800 2,100
8,200 8,500 6,900
20,700 19,300 18,700

Equity and reserves 10,400 9,000 8,600


Long term loan 4,400 5,000 5,600

Current liabilities:
Creditors 3,500 4,400 4,200
Bank overdraft 1,500 - -
Accrued expense 900 900 300
5,900 5,300 4,500
20,700 19,300 18,700

Statement of profit or loss


2018 2017 2016
----------- Rs. in '000 -----------
Sales 27,000 24,400 21,000
Cost of goods sold (21,300) (19,400) (17,200)
Gross profit 5,700 5,000 3,800
Operating expenses (3,400) (3,000) (2,400)
Finance cost (300) (350) (400)
Net profit 2,000 1,650 1,000

Required:
(a) Compute working capital cycle in days and liquidity ratios for 2018 and 2017. (11)
(b) Suggest three possible measures that can be taken by KL to improve working capital cycle
days.
(03)

Page 44 of 56

204
Ans.2 (a) Working capital cycle 2018 2017
------------- Number of days -------------
Average days to collect debtors W-1 50.0 37.4
Average inventory holding period W-2 72.8 70.5
Less: Average time to pay
creditors W-3 (69.3) (75.1)
53.5 32.8

Liquidity ratios: 2018 2017


Current ratio = Current assets 8,200 8,500
Current liabilities 5,900 5,300

1.39 : 1 1.60 : 1

Quick ratio = Current assets–inventor 8,200 – 4,000 8,500 – 4,500


Current liabilities 5,900 5,300

0.71 : 1 0.75 : 1

W1 Debtors = Average debtors x 365 [(4,200+3,200)÷2] x 365 [(3,200+1,800)÷2] x 365


collection Credit sales 27,000 24,400
period (in
days) 50 Days 37.4 Days

W2 Inventory = Average inventory x 365 [(4,000+4,500)÷2] x 365 [(4,500+3,000)÷2] x 365


holding Cost of sales 21,300 19,400
period (in
days) 72.8 Days 70.5 Days

W3 Creditors = Average creditors x 365 [{3,500+4,400)÷2] x 365 [(4,400+4,200)÷2 x 365


payment Credit purchases [21,300*+4,000 - 4,500] (19,400+4,500-3,000)
period (in
days) 69.3 Days 75.1 Days

W-4) stock

Page 45 of 56

205
“Be humble in you prayer and Pray to none but Allah.”

W-5)
Stock
b/d COS
3,000 19,400
20,900

c/d
4,500

(b) Measures to improve working cycle days:


▪ Give incentives to customers to pay on time.
▪ Do not transact with customers who have a history of defaulting / late payments.
▪ Automate the monitoring of accounts receivables.
▪ Try to sell the inventory quickly.
▪ Negotiate with suppliers to increase credit period.

Page 46 of 56

206
Q. 3 SK Limited (SKL) deals in a single product. Following are the summarized financial statements of SKL
for the year ended 31 December 2017:
Statement of financial position Statement of profit or loss

2017 2016
Rs. in million 2017 2016
Fixed assets 410 240 Units sold in million 39 30
Current assets 90 200
500 440 Rs. in million
Sales 371 300
Cost of goods sold (273) (210)
Capital 280 260
Gross profit 98 90
Long-term loan 170 100
Selling and administrative (55) (60)
Current liabilities 50 80
Finance cost (13) (8)
500 440
Net profit 30 22
Additional information:
i. With effect from 1 January 2017, selling price was decreased by 5% to boost sales volume.
ii. During the year 2017, suppliers demanded price increase of 4%. SKL resisted the price increase.
However, both parties agreed to reduce the credit period.
iii. SKL had been running its business in a rented building whose annual rent was Rs. 15 million. During
the year, SKL purchased this building for Rs. 200 million. Funds were arranged partially through a long-
term loan. Useful life of the building is estimated at 40 years.
iv. 75% of the selling and administration cost incurred in 2016 was fixed cost.

Required:
(a) Compute the following ratios for 2016 and 2017:

Gross profit margin Net profit margin


Return on capital
Return on assets employed
Debt equity ratio Current ratio (08)

(b) Keeping in view the above information, comment on profitability and liquidity position of SKL for 2017.
(04)

Page 47 of 56

207
“You are never too late nor old to move forward and make your life best.”

A. 3 SK Traders
Computation of ratios

2017 2016
(i) Gross profit margin 26.42% 30.00%
(98 ÷ 371) x 100 (90 ÷ 300) x 100
(ii) Net profit margin 8.09% 7.33%
(30 ÷ 371) x 100 (22 ÷ 300) x 100
(iii) Return on assets 8.60% 6.82%
(30 + 13) ÷ 500 x 100 (22 + 8) ÷ 440 x 100
(iv) Return on capital employed 9.56% 8.33%
(30 + 13)/280 + 170) x 100 (22 + 8) ÷ (260 + 100) x
100
(v) Debt equity ratio 37.78% 27.7%
170 ÷ (280 + 170) x 100% 100 ÷ (260 + 100) x 100%
(vi) Current ratio 1.80 2.50
90 ÷ 50 200 ÷ 80

(b)
(i) Profitability:
In 2017, gross profit margin of SKL has reduce from 30% to 26.42%. however, gross and
net profits amounts have been increased by Rs. 8 million mainly due to:
• Increase in sales volume as a result of 5% decrease in selling price. This resulted in
increase in gross profit by 8.89%[(98-90)÷90×100]. ( Horizontal analysis)
• Acquisition of building has resulted in savings in expenses as rent saved (Rs. 15 million)
is higher than the depreciation (Rs. 5 million) and increased in finance cost (Rs. 5 million).
• Since 75% of selling and administrative cost was fixed, expenses did not increase due to
increase in sales volume (economies of scale).

(ii) Liquidity:
The decrease in current ratio from 2.5 to 1.8 is net effect of the following:
• Cash payment for purchase of building which significantly decreased current assets.
• Prompt payment to suppliers which decreased the current liabilities.

Page 48 of 56

208
Further practice
1. WASIM
Wasim is an importer and retailer of vegetable oils. Extracts from the financial statements for this year and
last are set out below:
Income statements for the years ended 30 September
Year 7 Year 6
Rs. 000 Rs. 000
Revenue 2,160 1,806
Cost of sales (1,755) (1,444)
Gross Profit 405 362
Distribution costs (130) (108)
Administrative expenses (260) (198)
Profit before tax 15 56
Income tax expense (6) (3)
Profit for the period 9 53

Statement of financial position as on September


Year 7 Year 6
Rs. 000 Rs. 000
Assets
Non-current assets
Property, plant and equipment 78 72
Current assets
Inventories 106 61
Trade receivables 316 198
Cash - 6
422 265
Total assets 500 337
Equity and liabilities
Equity
Ordinary shares 110 85
Preference shares 23 11
Share premium 15 -
Revaluation reserve 20 20
Retained earnings 78 74
246 190
Current liabilities
Bank overdraft 49 -
Trade payables 198 142
Current tax payable 7 5
254 147

Page 49 of 56

209
Total equity and liabilities 500 337

Required:
Calculate profitability ratios, liquidity ratios and working capital ratios for both years to make comparison:
Profitability ratios means:
(a) Gross profit percentage.
(b) Net profit percentage
(c) Return on capital employed
(d) Asset turnover
Liquidity ratios means:
(e) Current ratio
(f) Quick ratio
Working capital ratios means:
(g) Average receivables collection period (average time to collect)
(h) Average payable period (average time to pay)
(i) Inventory turnover period

2. AMIR AND MO
The income statements and statements of financial position of two manufacturing companies in the same
sector are set out below:
Amir Mo
Rs. 000 Rs. 000
Revenue 150,000 700,000
Cost of sales (60,000) (210,000)
Gross profit 90,000 490,000
Interest payable (500) (12,000)
Distribution costs (13,000) (72,000)
Administrative expenses (15,000) (35,000)
Profit before tax 61,500 371,000
Income tax expense (16,605) (100,170)
Profit for the period 44,895 270,830
Assets
Non-current assets
Property - 500,000
Plant and equipment 190,000 280,000
190,000 780,000
Current assets
Inventories 12,000 26,250
Trade receivables 37,500 105,000
Cash at bank 500 22,000

Page 50 of 56

210
50,000 153,250
Total assets 240,000 933,250
Equity and liabilities
Equity
Share capital 156,000 174,750
Retained earnings 51,395 390,830
207,395 565,580
Non-current liabilities
Long-term debt 10,000 250,000
Current liabilities
Trade payables 22,605 117,670
Total equity and liabilities 240,000 933,250

Required:
Calculate profitability ratios, liquidity ratios and working capital ratios for each company to make
comparison:
Profitability ratios means:
(a) Gross profit percentage.
(b) Net profit percentage
(c) Return on capital employed
(d) Asset turnover
Liquidity ratios means:
(e) Current ratio
(f) Quick ratio
Working capital ratios means:
(g) Average receivables collection period (average time to collect)
(h) Average payable period (average time to pay)
(i) Inventory turnover period

Page 51 of 56

211
“Unity gives victory.”

Q.3 ALPHA LIMITED AND OMEGA LIMITED


Alpha Limited and Omega Limited are in the same trade, but operate in different areas. Their accounts for
the year ended 31 December, 2016 are as follows:
Profit and loss account Alpha Limited Omega Limited
Rs. ‘000’ Rs. ‘000’ Rs. ‘000’ Rs. ‘000’
Sales 1,440 1,720
Less: Cost of sales 1,120 1,342
Gross profit 320 378
Less: overheads 220 300
Profit before tax 100 78
Taxation 40 30
Dividends 20 24
60 54
Retained earnings 40 24
Statement of financial position
Share capital of Rs. 1 each 600 200
Reserves 240 104
840 304
8% Debentures - 120
840 424
Represented by:
Non-current assets at cost 660 520
Less: Accumulated Depreciation 200 160
460 360
Current assets:
Inventory 280 172
Receivables 310 300
Cash 30 32
620 504
Current Liabilities:
Taxation 40 30
Creditors 180 344
Bank overdraft - 42
Dividends 20 24
240 440
Net Current assets 380 64
840 424

Page 52 of 56

212
Required:
(a) Compute the following ratios for each of the companies:
(i) Current ratio
(ii) Acid test
(iii) Creditors ratio (payment period)
(iv) Collection Period or Receivables Ratio
(b) Carry out comparative analysis of the companies based on the computed ratios in (a) above.

A.1 WASIM
Ratios
Profitability ratios:
Year 7 Year 6
Gross profit % =
Gross profit 405 362
× 100 × 100 = 19% × 100 = 20%
Sales 2,160 1,806

Net profit % =
Net profit 9 53
× 100 ×100 = 0.4% ×100 = 2.9%
Sales 2,160 1,806
Return on capital employed =
Pr ofit before int erest and tax 15 56
× 100 = 6% × 100 = 29%
Share capital and reserves + Long − term debt capital 246 190
Asset turnover =
Sales 2,160 1,806
×100 = 8.8 times = 9.5 times
Share capital and reserves + Long − term debt capital 246 190
Liquidity ratios:
Current ratio =
Current assets 422 265
= 1.7 times = 1.8 times
Current liabilities 254 147
Quick ratio =
422 − 106
Current assets excluding inventor =1.2 265 − 61
254 = 1.4 times
Current liabilitie s 147
times
Working capital ratios:
Average time to collect =
Trade receivables 316  365 198 365
× 365 =53 days = 40 days
Sales 2,160 1,806

Page 53 of 56

213
Average time to pay =
Trade payables 198 365 142 365
× 365 = 41 days = 36 days
Cost of purchases 1,755 1,444
(Inventory turnover =
Inventory 106 365 61 365
× 365 = 22 days = 15 days
Cost of Sales 1,755 1,444

Note 1: averages are only calculated if it can be for all relevant years.
Note 2: cost of sales are considered in payment period because purchases for all relevant periods
cannot be calculated.

A.2 AMIR AND MO


Profitability ratios:
Amir Mo
Gross profit % =
90,000 490,000
Gross profit ×100 = ×100 =
× 100 150,000 700,000
Sales
60% 70%
Net profit % =
44,895 270,830
Net profit ×100 = ×100 =
× 100 150,000 700,000
Sales
30% 39%
Return on capital employed =
Pr ofit before int erest and tax
Share capital and reserves + Long − term debt capital
61,500 + 500
Amir × 100 = 28.5%
207,395 + 10,000
371,000 + 12,000
Mo × 100 = 47%
565,580 + 250,000
Asset turnover =
Sales
×100
Share capital and reserves + Long − term debt capital

150,000
Amir = 0.7 times
207,395 + 10,000
700,000
Mo = 0.85 times
565,580 + 250,000
Liquidity ratios:
Amir Mo
Current ratio =
Current assets 50,000 153,250
= 2.2 times = 1.3 times
Current liabilities 22,605 117,670

Page 54 of 56

214
Quick ratio =
50,000 − 12,000 153,250 − 126250
Current assets excluding inventor =
22,605 117,670
Current liabilitie s
=1.7 times 1.1 times
Working capital ratios:
Average time to collect =
37,500 105,000
Trade receivables ×365=91 ×365=55
× 365 150,000 700,000
Sales
days days
Average time to pay =
22,605 117,670
Trade payables ×365=137 ×365=204
× 365 60,000 210,000
Cost of purchases
days days
Inventory turnover =
12,000 26,250
Inventory ×365=73 ×365=46
× 365 60,000 210,000
Cost of Sales
days days
Note 1: averages are only calculated if it can be for all relevant years.
Note 2: cost of sales are considered in payment period because purchases for all relevant periods
cannot be calculated.
A.3 Alpha Limited and Omega Limited
(a)
(i) Current ratio Alpha Limited Omega Limited
Current assets 620,000 504,000
= 2.58:1 = 1.15:1
Current Liabilitie s 240,000 440,000
(ii) Acid test
Current assets − stock 340,000 * 332,000 *
= 1.42:1 = 0.75:1
Current Liabilities 240,000 440,000

*(620-280) *(504-172)
(iii) Creditors ratio
Average Creditors 180,000 344,000
× 365 ×365=59 days ×365=94 days
Purchases or cos t of sale 1,120,000 1,342,000
(iv) Collection period/Receivables ratio
310,000
Average debtors ×365=79 300,000
× 365 1,440,000 ×365=64 days
Sales 1,720,000
days

Page 55 of 56

215
(b) Comments on comparative analysis of the two companies based on the ratio computed
above:
(i) In terms of working capital and liquidity, Alpha Limited is in a better position to honour its
obligations as they fall due because its current ratio and acid test ratio are higher than those
of Omega Limited.
(ii) Omega Limited’s payment period is better than that of Alpha Limited’s because Omega
Limited uses supplier’s funds to finance its operation. However final analysis should be on the
basis of credit period available.
(iii) Omega Limited’s collection period is also better than that of Alpha Limited. It extends shorter
credit period to its customers than Alpha Limited. However final analysis should be on the
basis of credit period available.
(iv) Omega Limited’s credit policy is better than that of Alpha Limited. This because there is 30
days (94-64) difference between its payments period and collection periods compared with
Alpha Limited that has a longer collection period than its payment period.

Page 56 of 56

216
ANALYSIS OF FINANCIAL INFORMATION
⯈ Example:
The following is a specimen of horizontal analysis calculated for Adeel Limited:
Statement of profit or loss for the year ended 31 December 2024

Change from
2024 2023
2023 to 2024

AT A GLANCE
Rs. in millions %
Sales 4,315 3,760 14.8
Cost of Sales (3,020) (2,669) 13.2
Gross Profit 1,295 1,091 18.7
Distribution costs (800) (760) 5.3
Administrative expenses (296) (162) 82.7
Other income 15 10 50
Profit before interest and tax 214 179 19.6
Interest expense (13) (15) (13.3)
Profit before tax 201 164 22.6
Tax (35) (30) 16.7
Profit after tax 166 134 23.9

Statement of financial position as at 31 December 2024

Change from
2024 2023
2024 to 2023
Rs. in millions %
Non-current assets
Property, plant and equipment 865 880 (1.7)
Investment property 124 112 10.7
989 992 (0.3)
Current assets
Inventories 272 280 (2.9)
Trade receivables 168 173 (2.9)
Cash and bank 12 10 20.0
452 463 (2.4)
Total assets 1,441 1,455 (1.0)

217
Equity
Share capital 800 800 -
Retained earnings 360 310 16.1
1,160 1,110 4.5
Non-current liabilities
Long term loans 125 150 (16.7)
125 150 (16.7)
Current liabilities
AT A GLANCE

Trade payables 153 162 (5.6)


Accruals and other payables 3 33 (90.9)
156 195 (20.0)
Total equity and liabilities 1,441 1,455 (1.0)

⯈ Example:
The following is a specimen of vertical analysis calculated for Adeel Limited:
Statement of profit or loss for the year ended 31 December 2024

2024 2023
STICKY NOTES

Rs. m % Rs. m %
Sales 4,315 100 3,760 100
Cost of Sales (3,020) (70.0) (2,669) (71.0)
Gross Profit 1,295 30.0 1,091 29.0
Distribution costs (800) (18.5) (760) (20.2)
Administrative expenses (296) (6.9) (162) (4.3)
Other income 15 0.3 10 0.3
Profit before interest and tax 214 5.0 179 4.8
Interest expense (13) (0.3) (15) (0.4)
Profit before tax 201 4.6 164 4.4
Tax (35) (0.8) (30) (0.8)
Profit after tax 166 3.8 134 3.6

218
Statement of financial position as at 31 December 2024

2024 2023
Rs. m % Rs. m %
Non-current assets
Property, plant and equipment 865 60 880 60.5
Investment property 124 8.6 112 7.7
989 68.6 992 68.2
Current assets
Inventories 272 18.9 280 19.2
Trade receivables 168 11.7 173 11.9
Cash and bank 12 0.8 10 0.7
452 31.4 463 31.8
Total assets 1,441 100 1,455 100

Equity
Share capital 800 55.5 800 55.0
Retained earnings 360 25.0 310 21.3
1,160 80.5 1,110 76.3
Non-current liabilities
Long term loans 125 8.7 150 10.3
125 8.7 150 10.3
Current liabilities
Trade payables 153 10.6 162 11.1
Accrual and other payables 3 0.2 33 2.3
156 10.8 195 13.4
Total equity and liabilities 1,441 100 1,455 100

Example:
Ali and Bashir are chartered accountants and have been working as Managing Director (MD) and
Chief Financial Officer (CFO) in a listed company. In a recent meeting of the Board, the directors
have decided to expand the business within six months by opening 20 retail outlets. This
expansion would require financing of Rs. 300 million which may be arranged through bank loan.
The following information has been extracted from latest draft financial statements of the
company:

Rs. in ‘000
Sales 1,700
Gross profit 545
Tax expense 23
Profit after tax 40
Total assets 2,500
Non-current assets 900
Inventories 850

219
Trade receivables 600
Share capital 800
Reserves 152
Long term debt @ 9% 750

Following additional information is also available:


• 80% of the sales are on credit.
• Opening inventory was Rs. 100 million.
• 40% of current liabilities comprise of trade payables.
Required: Compute liquidity, working capital and debt ratios of the company.
⯈ Answer:
Liquidity ratios

(i) Current ratio

Current assets 2500 – 900


= = 2.01 times
Current liabilities 2,500 – 800 – 152 – 750

(ii) Quick ratio

Current assets – inventories 1,600 – 850


= = 0.94 times
Current liabilities 2,500 – 800 – 152 – 750

Working capital ratios

(iii) Inventory holding period

Average inventories x 365 [(850 +100)/2] x 365


= = 150 days
Cost of sales 1,700 – 545

(iv) Receivables collection period

Trade receivables x 365 600 x 365


= = 161 days
Sales (1,700 x 80%)

(v) Payables payment period


Trade payables x 365 [798 x 40%] x 365
= = 61 days
Purchases 1,905*

*Purchases = [1,700 – 545] – 100 + 850 = Rs. 190 5

220
Debt ratios

(vi) Gearing ratio

Long term debt 750


= = 44%
Capital employed 800 + 152 + 750

(vii) Interest cover

PBIT 40 + 23 + 67.5*
= =63 days
Interest expense 67.5*

*Interest expense = Rs. 750 loan x 9% = Rs. 67.5

⯈ Example:
Following amounts have been extracted from the financial statements of Lithops Limited:

2020 2019
----- Rs. in million -----
Sales 500 450
Cost of sales 378 300
Trade receivables 95 80
Trade payables 72 60
Inventory 93 75
Cash at bank 12 16

All sales and purchases are made on credit.


Required:
a) Calculate working capital cycle days for 2020. (Assume a 360-day year)
b) Suggest four possible measures that can be taken to reduce working capital cycle days.
⯈ Answer:
Part (a) Working capital cycle

Days
Inventory holding period (i) 80
Receivables collection period (ii) 63
Payables payment period (iii) (60)
Working capital cycle 83

Relevant ratios
(i) Inventory holding period

Average inventory x 360 [(93+75)/2] x 360


= = 80 days
Cost of sales 378

221
(ii) Receivables collection period

Average receivables x 360 [(95+80)/2] x 360


= =63 days
Sales 500

(ii) Payables payment period

Average payables x 360 [(72+60)/2] x 360


= = 59.1 days
Purchases 396*

*Purchases = Cost of sales 378 – 75 opening inventory + 93 closing inventory = Rs. 396

Part (b) Measures to improve working capital cycle days:


• Give incentives to customers to pay on time
• Do not transact with customers who have a history of defaulting/late payments
• Automate the monitoring of accounts receivables
• Resolve disputes with customer as early as possible

222
“Don’t be aggressive but try to understand feelings of others.”

Government Grant
(IAS-20)

Grant related to Grant related to Assets


income as a
compensation
for Expenses
Monetary Asset Non- monetary
(cash) is received to asset (i.e. Actual
acquire asset asset e.g. land or
Grant for past expenses Grant for future machinery) is
or for immediate Expenses received
financial support
(unrelated to future Cash for depreciable Cash for non-
Recognize in profit or
costs) Asset is received depreciable asset is
loss over the period
the expenditure is (e.g. machinery) received (e.g. Land)
recognized
Recognize as
income in the
Grant is credited to Grant is credited to
period in which
Grant Income Grant is deferred grant asset account to
grant becomes
credited credited to income account which the grant
receivable
separately as related and is recognized as relates [ i.e Grant should be
(Example-1)
other income expense grant income over indirectly recognized as an
(Direct (indirect the useful life of recognized as income on the basis
approach) Approach) the asset (direct income over the that best reflects the
(Example-2) (Example-3) approach) period of grant by manner in which the
(example - 4) way of reduced conditions are met
depreciation] (Example-6)
(indirect approach)
[Example - 5]

Recognize the asset at either:

Fair value (Example-7A) Nominal amount of cash paid


(if any) (Example-7B) Repayment of Government Grants

Grant for Expenses Grant for Assets


Grants received as a package Debited first against
(means a combination of) the balance on the
deferred income
Account (if any) and Treated as a Treated as
balance (if any) to deferred income
Grant Grant for future Grant for past Grants for immediate reduction from
profit or loss separately
for asset expenses expenses financial support cost of Asset
(Example - 9)

Debited first against Debited to asset


the balance on the A/C & recognize
All these amounts will be
separately recognized as deferred income cumulative
an income as discussed account (if any) and additional
previously (Example - 8) balance (if any) to depreciation
profit or loss immediately in
(Example - 10A) profit or loss
(Example - 10B)

223 Page 1 of 1
“Trust in the plans of Allah. Be patient, Keep smiling and never overthink.”

Government Grants and Government Assistance


(IAS 20)
Government grants are provided to encourage an entity to become involved in certain activities that it may
otherwise not have involved itself in. It is often provided to assist businesses in starting up. This obviously
benefits the business but also benefits the government through creation of jobs and thus a larger base of
taxpayers.

There are two types of grants:

1. Grants related to income (as a compensation of expenses)


2. Grants related to assets:

Grants related to income as a compensation of expenses


If the grant received does not relate to an asset it could be used as:
• Compensation for past expenses or as immediate financial support; or as
• Compensation for future expenses still to be incurred.

Grant for past expenses or immediate financial support


The grant may be receivable as either:
• Immediate financial support (unrelated to future costs); or
• For expenses or losses already incurred.

Accounting treatment:
Where the grant relates to immediate financial support or past expenses, it is recognised as income in the
period in which the grant becomes receivable.

Page 1 of 22

224
“Sins take you away from Allah while prayers take you back to Allah.”

Grant as a compensation for future expenses

If the grant is to be used to subsidise certain future expenditure, then it should be recognised in the
statement of comprehensive income over the period that the expenditure is recognised.

Approaches to present government grant in income statement:


There are two approaches that the company may use in presenting the government grant:
a. direct income approach: the grant is credited to a grant income account in other income (i.e. the grant
is recognised directly as income over the period of the grant);
b. indirect income approach: the grant is credited to the expense account to which the grant relates
(indirectly recognised as income over the period of the grant by way of the reduced expenditure).

Example 2: grant for future expenses – direct approach

The government grants a company a cash of 10 000 to contribute 10% towards 100 000 of future wage
expenditure. The grant was received on 1 January 2011.
Required:
Show the journal entries for the year ended 31 December 2011 assuming that the company policy is to
present such a grant as grant income (i.e. direct income approach):
A. The company incurs 100 000 wage expenditure in 2011;
B. The company incurs 20 000 of the related wage expenditure in the year ended 31 December 2011 and
80 000 thereof in the year ended 31 December 2012.

Solution to example 2A: grant for future expenses – direct approach

1 January 2011 Debit Credit


Bank 10 000
Deferred grant
income (presented
in liabilities) 10 000
Recognising a government grant intended to reduce future
expenses

31 December 2011
Wage expenditure 100 000
Wages payable /
Cash 100 000
Wage expenditure incurred

Deferred grant income 10 000


Grant income Presented in other income 10 000
Recognising 100% of the government grant since all related
expenses that the grant was intended to compensate have been
incurred

Note: the statement of comprehensive income will reflect a wage expense of 100 000 and grant
income of 10 000 (the net effect on profit is a net expense of 90 000).

Page 2 of 22

225
Solution to example 2B: grant for future expenses – direct approach

1 January 2011 Debit Credit


Bank 10 000
Deferred grant income 10 000
Recognising a government grant intended to reduce future
expenses

31 December 2011
Wage expenditure 20 000
Wages payable/cash 20 000
Wage expenditure
incurred

Deferred grant income 10 000 x 20% 2 000


Grant income 2 000
Recognising 20% of the government grant since 20% of the
expenses
that the grant was intended to compensate have been incurred

31 December 2012
Wage expenditure 80 000
Wages payable/cash 80 000
Wage expenditure
incurred

Deferred grant income 10 000 x 80% 8 000


Grant income Recognised directly as income 8 000
Recognising 80% of the government grant since 80% of the
expenses that the grant was intended to compensate have been
incurred

Note: the statement of comprehensive income will reflect:


2011: a wage expense of 20 000 and grant income of 2 000 (the net decrease in profits: 18 000);
2012: a wage expense of 80 000 and grant income of 8 000 (net decrease in profits: 72 000).

Example 3: grant for future expenses – indirect approach


The government grants a company a cash of 10 000 to contribute 10% towards 100,000 of future specified
wages. The grant was received on 1 January 2011. The year-end is 31 December.

Required:
Show the journal entries assuming that the company policy is to recognise government grants as a credit
to the related expense (i.e. indirect income approach):
a. The company incurs all intended expenditure in the year ended 31 December 2011;
b. The company incurs 20% of the wages in 2011 and 80% in 2012.

Page 3 of 22

226
“Prayer is just like water, It keeps the root of Imaan alive so never forget to pray salat.”

Solution to example 3A: grant for future expenses – indirect approach

1 January 2011 Debit Credit


Bank 10 000
Deferred grant income 10 000
Recognising a government grant intended to reduce future
expenses

31 December 2011
Wage expenditure 100 000
Wages payable/cash 100 000
Wage expenditure
incurred

Deferred grant income 10 000


Wage expenditure Recognised indirectly as income 10 000
Recognising 100% of the government grant since all related
expenses that the grant was intended to compensate have been
incurred

Note: the statement of comprehensive income will reflect a wage expense of 90 000 (the net effect
on profit is a decrease of 90 000).

Compare this to example 2A: the effect on profit is the same.

Solution to example 3B: grant for future expenses – indirect approach

1 January 2011 Debit Credit


Bank 10 000
Deferred grant income 10 000
Recognising a government grant intended to reduce future
expenses

31 December 2011
Wage expenditure 20 000
Wages payable/cash 20 000
Wage expenditure incurred

Deferred grant income 10 000 x 20% 2 000


Recognised indirectly as
Wage expenditure income 2 000
Recognising 20% of the government grant since 20% of the
expenses
that the grant was intended to compensate have been incurred

31 December 2012
Wage expenditure 80 000
Wages payable/cash 80 000
Wage expenditure incurred

Page 4 of 22

227
Deferred grant income 10 000 x 80% 8 000
Recognised indirectly as
Wage expenditure income 8 000

Recognising 80% of the government grant since 80% of the


expenses that the grant was intended to compensate have been
incurred

Note: the statement of comprehensive income will reflect:


20X1: a wage expense of 18 000 (the net decrease in profits: 18 000);
20X2: a wage expense of 72 000 (net decrease in profits: 72 000);
Compare this to example 2B: the effect on profit is the same.

Page 5 of 22

228
Grants related to assets
Grants related to assets could be provided as:

“Sincere prayer and faith in Allah brings light in the heart.”

• a monetary asset (i.e. cash) that must be used to acquire a non-monetary asset (e.g. land or
machinery); or as
• a non-monetary asset (i.e. the actual asset is provided)

The non-monetary asset itself could be:


• a depreciable asset (e.g. a machinery); or
• a non-depreciable asset (e.g. land)
1. cash grant is received to acquire the asset:
a. Grant related to a depreciable asset

If the asset received or to be acquired is a depreciable asset, the grant is usually recognised as income
over the same period that the asset is depreciated.

Where the grant relates to an asset, the initial grant may be recorded using either of the following
approaches:

• direct income approach: the grant is credited to a deferred grant income account and is
recognised as grant income over the useful life of the asset (i.e. the grant is recognised directly as
income over the life of the asset);

• Indirect income approach: the grant is credited to the asset account to which the grant relates
(i.e. indirectly recognised as income over the period of the grant by way of a reduced depreciation
charge).

Example 4: grant related to a depreciable asset – direct approach

The government grants a company a cash of 12 000 on 1 January 2011 to assist in the acquisition of a
nuclear plant. The nuclear plant was acquired on 1 January 2011 for 90 000, was available for use
immediately and has a useful life of 3 years (the plant has a nil residual value).

Required:
Show the journal entries in the years ended 31 December 2011, 2012 and 2013. The company has the
policy of recognizing government grants directly in income.

Page 6 of 22

229
Page 7 of 22

230
“Prayers take you near to Allah.”

Note: the statement of comprehensive income will reflect:


2011 – 2013: a depreciation expense of 30 000 and grant income of 4 000 (net decrease in profits: 26 000
per year).

Example 5: grant related to a depreciable asset – indirect approach

The government grants a company a cash of 12 000 on 1 January 2011 to assist in the acquisition of a
nuclear plant. The nuclear plant:
• was acquired on 1 January 2011 for 90 000;
• was available for use immediately; and
• has a useful life of 3 years (the plant has a nil residual value).

Required:
Show the journal entries in the years ended 31 December 2011, 2012 and 2013. The company has the
policy of recognizing government grants indirectly in income (i.e. as a reduction of the cost of the asset).

Solution to example 5: grant related to a depreciable asset – indirect approach

1 January 2011 Debit Credit


Bank 12 000
Deferred grant income 12 000
Recognising a government grant intended to assist in the
acquisition
of a nuclear plant

2 January 2011
Nuclear plant: cost (asset) 90 000
Bank 90 000
Purchase of plant

Deferred grant income 12 000


Nuclear plant: cost (asset) 12 000
Recognising the government grant as a reduction of the plant’s
cost

31 December 2011
Depreciation - plant
(expense) (90 000 – 12 000 – 0) / 3 years 26 000
Nuclear plant: accumulated depreciation (asset) 26 000
Depreciation on plant

31 December 2012
Depreciation - plant
(expense) (90 000 – 12 000 – 0) / 3 years 26 000
Nuclear plant: accumulated depreciation (asset) 26 000
Depreciation on plant (net of
30,000 – 4,000 as was in
previous example)

Page 8 of 22

231
31 December 2013
Depreciation - plant
(expense) (90 000 – 12 000 – 0) / 3 years 26 000
Nuclear plant: accumulated depreciation (asset) 26 000
Depreciation on plant

Note: the statement of comprehensive income will reflect:


2011 – 2013: a depreciation expense of 26 000 (net decrease in profits: 26 000 per year). Compare this to
example 4.

b. Grant related to non-depreciable assets:


If the grant relates to the cost of a non-depreciable asset, the grant should be recognized as an income on
a basis that best reflects the manner in which the conditions are met.

If the asset to be acquired is a non-depreciable asset, the grant may require certain obligations to be met,
in which case the grant would be recognised as the obligations were met. By way of example, a grant could
be provided by way of cash to purchase land on condition that a building is erected on it. In this case, the
grant could be recognised as income once the building is erected or the grant could be recognised as
income over the life of the building (being a depreciable asset).

Example 6: grant related to a non-depreciable asset – direct approach


The government grants a company a cash of 12 000 on 1 January 2011 to assist in the acquisition of land.
A condition of the grant is that the company builds a factory on the land. The land was acquired on 1
January 2011 for 90 000. Land is not depreciated. The factory was completed on 31 March 2011 (total
building costs of 300 000 were paid in cash on this date), was available for use immediately and has a
useful life of 3 years (the factory has a nil residual value).

Required:
Show the journal entries in the years ended 31 December 2011, 2012, 2013 and 2014. The company’s
policy is to recognise grants directly in income.

Solution to example 6: grant related to a non-depreciable asset – direct approach

1 January 2011 Debit Credit


Bank 12 000
Deferred grant income 12 000
Government grant received to assist in the acquisition of land

Land: cost 90 000


Bank 90 000
Purchase of land

31 March 2011
Factory building: cost 300 000
Bank 300 000
Building costs related to factory, paid in cash

31 December 2011
Depreciation – factory building (300 000 – 0) / 3 years x 9 / 12 75 000
Factory building: accumulated depreciation 75 000

Page 9 of 22

232
Depreciation of factory
building

Deferred grant income 12 000 / 3 years x 9 / 12 3 000


Grant income 3 000
Grant income recognised on the same basis as depreciation on
the
factory building

31 December 2012
Depreciation – factory building (300 000 – 0) / 3 years 100 000
Factory building: accumulated depreciation 100 000
Depreciation of factory
building

Deferred grant income 12 000 / 3 years 4 000


Grant income 4 000
Grant income recognised on the same basis as depreciation on
the
factory building

31 December 2013
Depreciation – factory building (300 000 – 0) / 3 years 100 000
Factory building: accumulated depreciation 100 000
Depreciation of factory
building

Deferred grant income 12 000 / 3 years 4 000


Grant income 4 000
Grant income recognised on the same basis as depreciation on
the
factory building

31 December 2014
Depreciation – factory building (300 000 – 0) / 3 years x 3 / 12 25 000
Factory building: accumulated depreciation 25 000
Depreciation of factory
building

Deferred grant income 12 000 / 3 years x 3 / 12 1 000


Grant income 1 000
Grant income recognised on the same basis as depreciation on
the factory building

2. Non monetary grants


Where the grant is in cash, the measurement thereof is discussed previously. If, however, the company is
granted an asset such as a licence to operate or land or machinery etc., the company may either measure
the grant at its fair value or at the nominal cost thereof, being the actual cash paid for the asset, if any. If
nothing is paid then if the asset is not measured at fair value the government grant is not measured at all.

Page 10 of 22

233
Example 7: grant asset – fair value or nominal amount
Pakistani government grants the company a licence to fish off the coast of Gwadar. The fair value of the
licence is 50 000 and the company is required to pay a nominal amount of 1 000 for the licence.

Required:
Show the journal entries assuming:
A. The company chooses to measure the licence at its fair value.
B. The company chooses to measure the licence at its nominal amount.

Solution to example 7A: grant asset – fair value

Debit Credit
Fishing licence (asset) Given 50 000
Deferred income 50 000 – 1 000 49 000
Bank Given 1 000
Recognising the licence granted by the government at fair value

Solution to example 7B: grant asset – nominal amount

Debit Credit
Fishing licence (asset) Given 1 000
Bank Given 1 000
Recognising the licence granted by the government at nominal
value

Grants received as a package (IAS 20.19)


If the grant is received as a package to which a number of varying sets of conditions are attached, it
may be appropriate to recognise each part of the grant on a different basis. The first step is to identify
each part of the package to which there are different conditions affecting when the grant is earned.
The grant may, for instance, relate to a combination of:
• an asset
• future expenses
• past expenses
• immediate financial support.

Accounting treatment:
In such cases, the grant package may be viewed as multiple parts. The grant relating to:
A. the asset should be recognised as income in a way that reflects the pattern of depreciation;
B. future expenses should be recognised as income in a way that reflects the pattern of future
expenses;
C. past expenses should be recognised as income in the period in which the grant becomes
receivable;
D. general and immediate financial support should be recognised as income in the period in which
the grant becomes receivable

Example 8: grant is a package deal


The government grants a company a cash of 120 000 on 1 January 2011. The grant relates to two
aspects:
• 30 000 is a cash sum as immediate financial support with no associated future costs;
• 90 000 is to assist in the future acquisition of vehicles.

Page 11 of 22

234
“A prayer with sincere heart is never wasted.”

The vehicles were acquired on 1 January 2011 for 210 000. The vehicles were available for use immediately
and have a useful life of 3 years (the vehicles all have nil residual values).

The company has decided to follow the direct Approach related to vehicle grant; i.e recognize separately
as income in statement of profit or loss.

Required:
Show the journal entries in the years ended 31 December 2011, 2012 and 2013.

Solution to example 8: grant is a package deal

1 January 2011 Debit Credit


Bank 120 000
Deferred grant income 90 000
Grant income 30 000
Recognising a government grant: package deal and
Portion of grant income recognised immediately – not attached
to any
asset or future expenses and all criteria met in a prior year: 30000

1 January 2011
Vehicles: cost 210 000
Bank 210 000
Purchase of vehicles

31 December 2011
Depreciation – vehicles (210 000 – 0) / 3 years 70 000
Vehicles: accumulated depreciation 70 000
Depreciation of vehicles

31 December 2011 Debit Credit


Deferred grant income (120 000 – 30 000) / 3 years 30 000
Grant income 30 000
Portion of grant income related to purchase of vehicles
recognised on
the same basis as vehicle depreciation

31 December 2012
Depreciation – vehicles (210 000 – 0) / 3 years 70 000
Vehicles: accumulated depreciation 70 000
Depreciation of vehicles

Deferred grant income (120 000 – 30 000) / 3 years 30 000


Grant income 30 000
Portion of grant income related to purchase of vehicles
recognised on
the same basis as vehicle depreciation

Page 12 of 22

235
31 December 2013
Depreciation – vehicles (210 000 – 0) / 3 years 70 000
Vehicles: accumulated depreciation 70 000
Depreciation of vehicles

Deferred grant income (120 000 – 30 000) / 3 years 30 000


Grant income 30 000
Portion of grant income related to purchase of vehicles recognised
on the same basis as vehicle depreciation

Repayment of government grant:


A government grant might become repayable by the entity if the grant was provided on certain conditions
and these conditions were later breached by the entity.

If the grant becomes repayable it is accounted for as a change in accounting estimate (means
prospectively)

Accounting treatment:
Where the grant has to be repaid the treatment depends on whether the grant was related to expenses
or assets.
1. If the original grant related to income for the compensation of expenses, the repayment of the
grant is debited:
• first against the balance on the deferred income account, if any; and
• balance if any to profit or loss as an expense.

Example 9: grant related to expenses – repaid

The local government granted the company 10 000 on 1 January 2011 to assist in the financing of mining
expenses. The grant was conditional upon the company mining for a period of at least two years. Initially it
was expected that company can meet this condition and therefore it was decided to recognize the grant as
income equally over two years.

The company ceased mining on 30 September 2012 due to unforeseen circumstances. The terms of the
grant required that the grant be repaid immediately and in full.

Mining expenses incurred to date were as follows:


2011: 80 000
2012: 60 000

Required:
Show the journal entries assuming:
A. The company recognises grants directly as ‘grant income’.
B. The company recognises grants indirectly as income by reducing the related expense.

Solution to example 9A: grant related to expenses – repaid

1 January 2011 Debit Credit


Bank 10 000
Deferred grant income 10 000
Recognising a government grant intended to reduce future
expenses
80 000

Page 13 of 22

236
31.12.2011

Mining expenses
Accounts payable / cash 80 000

Debit Credit
Deferred grant income 5 000
Grant income 5 000

30.09.2012 Debit Credit


Mining expenses 60 000
Accounts payable / cash 60 000

Debit Credit
Deferred grant income 3 750
Grant income 3 750
(10 000 x 50% x 9/12)

Debit Credit
1 250
Deferred grant income 8 750
Other expenses
Bank 10 000
(repayment)

Solution to example 9B: grant related to expenses – repaid


1 January 2011 Debit Credit
Bank 10 000
Deferred grant income 10 000
Recognising a government grant intended to reduce future
expenses

31.12.2011

Mining expenses 80 000


Accounts payable / cash 80 000

Debit Credit
Deferred grant income 5 000
Mining expenses 5 000

30.09.2012 Debit Credit


Mining expenses 60 000
Accounts payable / cash 60 000

Page 14 of 22

237
Debit Credit
Deferred grant income 3 750
Mining expenses 3 750
(10 000 x 50% x 9/12)

Debit Credit
1 250
Deferred grant income 8 750
Other expenses
Bank 10 000
(repayment)

2. If the original grant related to an asset, and


A. it was accounted for as a deferred income, then:
• the repayment of the grant is debited to the balance on the deferred income account, if any; and
• then to the profit or loss as an expense.

B. if it was accounted for as a reduction from the cost of the asset; then
• Repayment is debited to the asset account; and
• Recognize the cumulative additional depreciation that would have been recognized to date had the
grant not been received immediately as an expense.

Example 10: grant related to assets – repaid


The local government granted the company 10 000 on 1 January 2011 to assist in the purchase of a
manufacturing plant. The grant was conditional upon the company manufacturing for a period of at least
two unbroken years.

The company purchased the plant on 1 January 2011 for 100 000. The plant is depreciated on the straight-
line basis over its useful life of 4 years to a nil residual value.

The company ceased manufacturing on 30 September 2012 due to unforeseen circumstances. The terms
of the grant required that the grant be repaid immediately and in full. The company intended to resume
manufacturing in the next year.

Required:
Show the journal entries for the year ended 31.12.2011 and 31.12.2012. assuming that the company:
A. recognises grants as grant income (direct income).
B. recognises grants as a reduction of the cost of the related asset (indirect income).

Solution to example 10A: grant related to assets – repaid

1 January 2011 Debit Credit


Bank 10 000
Deferred grant income 10 000
Recognising a government grant
1 January 2011
Plant: cost 100 000
Accounts payable/
bank 100 000
Purchase of plant

Page 15 of 22

238
31 December 2011
Depreciation - plant (100 000 – 0) / 4 years x 12 / 12 25 000
Plant: accumulated depreciation 25 000
Depreciation of plant
Deferred grant income 10 000 / 4 years x 12 / 12 2 500
Grant income 2 500
Recognising 25% of the government grant since the grant relates
to
the acquisition of an asset that is depreciated over 4 years
30 September 2012
Depreciation - plant (100 000 – 0) / 4 years x 9 / 12 18 750
Plant: accumulated depreciation 18 750
Depreciation of plant: (manufacture ceases on 30 September 20X2)
Deferred grant income 10 000 / 4 years x 9 / 12 1 875
Grant income 1 875
Recognising 9 months of the government grant to the date of repayment of the grant

30 September 2012 Debit Credit

–2
Deferred grant income 10 000 500 – 1 875 5 625
Other expense (balancing) 4 375
Bank Given 10 000
Repayment of the full grant, first reducing the balance on the deferred income account and then
expensing the rest

31 December 2012 Debit Credit


Depreciation – plant 6 250
Plant: accumulated depreciation 6 250
Depreciation of plant: (100 000/4 x 3/12)
Solution to example 10B: grant related to assets – repaid

1 January 2011 Debit Credit


Bank 10 000
Deferred grant income 10 000
Recognising a government grant

1 January 2011
Plant: cost 100 000
Accounts payable/
bank 100 000
Purchase of plant

Deferred grant income 10 000


Plant: cost 10 000
Recognising the grant income as a decrease in the asset’s cost

Page 16 of 22

239
31 December 2011
(100 000 –10 000 – 0) / 4 years x 12
Depreciation - plant / 12 22 500
Plant: accumulated depreciation 22 500
Depreciation of plant:

30 September 2012
(100 000 – 10 000 – 0) / 4 years x 9 /
Depreciation - plant 12 16 875
Plant: accumulated depreciation 16 875
Depreciation of plant: (manufacture ceases on 30 September 20X2)

Plant: cost Original grant refunded 10 000


Bank 10 000
Depreciation – plant W1: 2 500 + 1 875 4 375
Plant: accumulated
depreciation W1: 2 500 + 1 875 4 375

Repayment of the full grant: increase cost and increase accumulated depreciation with extra cumulative
depreciation that would otherwise have been expensed if no grant had been received on 1 January
20X1

31 December 2012 Debit Credit


Depreciation – plant 6 250
Plant: accumulated depreciation 6 250
Depreciation of plant: (100 000/4 x 3/12)
Or [100,000-10,000-22,500-16,875+10,000-4,375]/2.25x3/12=6,250

Working extra depreciation to be charged:


Date Calculations Was Is Difference
Cost 1.1.2011 100,000 - 90,0000 100,000 10,000
10,000
Depreciation 31.12.2011 22,500 25,000 2,500
(90,000/4) (100,000/4)
Depreciation 31.12.2012 9 months 16,875 18,750 1,875
Carrying 31.12.2012 50,625 56,250 5,625
amount

Page 17 of 22

240
“Prayer is a weapon for believers to overcome the hard situations.”

Disclosures:
The following issues must be disclosed in the notes to the financial statements:
1. Accounting policy regarding both recognition and method of presentation, for example:
o Government grants are recognised as income over the period to which the grant applies;
and
o The grant is presented as a decrease in the expenditure to which it relates (or: the grant is
presented as a separate line item: grant income).;
2. The nature and extent of government grants recognised in the financial statements;
3. An indication of other forms of government assistance not recognised as government grants but from
which the entity has benefited directly (e.g. provision of guarntee and free marketing and technical
advice);
4. Unfulfilled conditions and other contingencies attached to recognised government grants.

Page 18 of 22

241
“Prayer can change your situation so remember Allah and offer prayers.”

Question:
During the year ended 30 June 2012, Ahmed Limited (AL) received three grants, the details of which are
set out below.
(1) On 1 September 2011, a grant of Rs. 40,000 from local government. This grant was in respect of
training costs of Rs. 70,000 which AL had incurred during the year.
(2) On 1 November AL bought a machine for Rs. 350,000. A grant of Rs. 100,000 was received from
central government in respect of this purchase. The machine, which has a residual value of Rs.
50,000, is depreciated on a straight-line basis over its useful life of five years.
(3) On 1 June 2012 a grant of Rs. 100,000 from local government. This grant was in respect of relocation
costs that AL had incurred moving part of its business from outside the local area. The grant is
repayable in full unless AL recruits ten employees locally by the end of 30.06.2012. AL is finding it
difficult to recruit as the local skill base does not match the needs of the business.
Required
Show the journal entries and how the above transactions should be reflected in the financial statements for
the year ended 30 June 2012, under both the options: i.e.:
1. Treat the relevant grants as separate income.
2. Treat the relevant grants as deduction.

Solution:

(a) Journal entries:

1. Grant 1:
Option-1:
[as separate income]

Training exp 70,000


Cash 70,000
1-9-2011
Cash 40,000
Grant Income 40,000
(treat it as a separate
other income)

Option2{As a deduction from cost of Assets]

Training exp 70,000


Cash 70,000
1-9-2011
Cash 40,000
Training exp 40,000

Page 19 of 22

242
2. Grant 2:

1-11-2011
Machine 350,000
Cash 350,000

Option 1: Option 2:
1-11-2011 Grant As a deferred income means Grant As a deduction from cost of
a liability asset:
Cash 100,000 Cash 100,000
Deferred Income Machine 100,000
100,000
30-6-2012
Depreciation 40,000 Depreciation 26,667
Acc. dep 40,000 Acc. Dep 26,667

([350,000 – 50,000] / 5)x ([350,000 – 100,000]-50,000] / 5)x


8/12 = 40,000 8/12 = 26,667

Deferred Income 13,333


Grant Income 13,333
[100,000/5] x 8/12 =
13,333

3. Grant 3 : As the grant is conditional on future recruitment of 10 employees and AL is finding it difficult to
recruit; therefore amount received should be treated as a liability (as it is likely to be repaid).

Cash 100,000
Grant payable 100,000
Note
The Rs. 100,000 grant (3) has conditions attached to it. In such a situation, IAS 20 states that grants should
not be recognized until there is reasonable assurance that the entity will comply with any conditions
attaching to the grant. Since AL is struggling to recruit. Hence the grant should not be recognized as such,
but should be held in current liabilities, pending repayment.

(b) Presentation in financial statements

Option 1 – Show grants separately from related expenditure


Statement of financial position as at 30 June 2012 (extracts)
Rs.
Non-current assets
Property, plant and equipment (Note 1) 310,000
Non-Current liabilities:
Deferred Income (Grant 2 ) (Note 2) 66,667
Current liabilities
Deferred Income (Grant 2 ) 20,000
Grant payable(Grant 3) 100,000

Notes to the financial statements for the year ended 30 June 2012 (extracts)

Rs.
1.Property, plant and equipment
Cost 350,000
Accumulated depreciation ((350,000 – 50,000) ÷ 5 x 8/12) (40,000)

Page 20 of 22

243
––––––––
Carrying amount 310,000
––––––––

2.Deferred income relating to government grants- As on 30-6-2012 86,667


(100,000 - (100,000 ÷ 5 x 8/12))
Less: Grant income of 30-6-2013 [ Current liability]
(100,000/5) (20,000)
––––––––
Balance of Non-Current Liability 66,667
––––––––
Included in statement of profit or loss for the year ended 30 June 2012
Operating expenses: Rs.
Depreciation charge (40,000)
Training costs (70,000)
Other Income:
Government grant received (Grant 1) 40,000
Government grant income (Grant 2) 13,333

Option 2 – net off grant from related expenditures


Statement of financial position as at 30 June 2012 (extracts)

Rs.
Non-current assets
Property, plant and equipment (Note 1) 223,333
Current liabilities
Grant payable (Grant 3) 100,000

Notes to the financial statements for the year ended 30 June 2012 (extracts)

1. Property, plant and equipment Rs.


Cost (350,000 – 100,000) 250,000
Accumulated depreciation ((250,000 – 50,000) ÷ 5 x 8/12) (26,667)
––––––––
Carrying amount 223,333
––––––––
Included in statement of profit or loss for the year ended 30 June 2012
Rs.
Depreciation charge 26,667
Training costs (70,000 – 40,000) 30,000

Page 21 of 22

244
Extra practice question
Q.1 Discuss how the following should be dealt with in the financial statements of relevant entities
according to IAS 20 Accounting for Government Grants and Disclosure of Government Assistance:

(a) The government makes a grant to an entity which is planning to develop teaching software
for children with learning difficulties. The purpose of the grant is to help the entity to meet its
general financing requirement in the initial phase. There are no further conditions attached
to the grant.
(01)

(b) A manufacturing entity sets up a plant in an area of high unemployment. A government grant
of Rs. 4 million is received with a condition that the grant is repayable in full if the number of
its employees fell below 100 at any time during the next four years. It is highly probable that
the entity will comply with the condition attached to the grant.
(03)

(c) Free technical advice has been provided by the government’s export promotion department
to help an exporter to market his new technology in North America.
(01)

A. (i) The grant has been provided for the purpose of giving immediate financial support to the entity with
no further conditions, so this grant should be immediately recognised in profit or loss in full in the period in
which the entity qualifies to receive it (when it is receivable) with disclosure to ensure that its effect is clearly
understood.

(ii) Since there is reasonable assurance that conditions attaching to the grant will be met, the grant is
recognised in statement of profit or loss over the four year period in which the entity incurs the costs of
employing 100 people. Amount taken to statement of profit or loss may be either be presented as other
income or shown as deduction from the related expense. The remaining amount of grant will be presented
as deferred income under liabilities in the balance sheet.

(iii) Free technical advice is government assistance that cannot reasonably have a value placed upon it and
therefore should not be recognised. However, an indication of such assistance should be disclosed in
financial statements.

Page 22 of 22

245
IAS 20

Introduction [IAS 20: 1]


Across the globe, the governments provide various types of assistance to businesses in order to achieve various
economic objectives such as to promote a specific type of business (say, electric vehicles) or to create
employment opportunities. The assistance may be mere an aid by creating ease of doing business or it may be in
the form of a financial assistance. The most common form of such assistance is a grant of cash or land to the
business entity from local or national government.
IAS 20 is applied in accounting for, and in the disclosure of, government grants and in the disclosure of other
forms of government assistance.

Key definitions [IAS 20: 3 & 6]


AT A GLANCE

Government refers to government, government agencies and similar bodies whether local, national or
international.
Example:
As per IAS 20, the following may be considered “Government” if providing financial assistance to an
entity:

1. Federal governments 5. Small and Medium Enterprises Development Authority


2. Provincial governments 6. United Nations
3. District governments 7. International Monetary Fund (IMF)
SPOTLIGHT

4. Federal/State Banks 8. World Health Organisation

Government assistance is action by government designed to provide an economic benefit specific to an entity
or range of entities qualifying under certain criteria.
However, government assistance does not include benefits provided only indirectly through action affecting
general trading conditions, such as the provision of infrastructure in development areas or the imposition of
trading constraints/quotas on competitors.
Example:
JK Limited constructed its factory few years ago in Gwadar. Since then the government has
provided infrastructure by improvement to the general transport and communication network
and the supply of improved facilities including water reticulation which is available on an
ongoing basis for the benefit of an entire local community including JK Limited factory. The
business of JK Limited has become much more profitable since provision of these facilities.
Required: State whether the actions by government in above circumstances are considered
government assistance in accordance with IAS 20.
Answer:
The above actions affecting general trading conditions are not government assistance as per IAS
20.
Government grants are assistance by government in the form of transfers of resources to an entity in return for
past or future compliance with certain conditions relating to the operating activities of the entity.
However, they exclude those forms of government assistance which cannot reasonably have a value placed upon
them and transactions with government which cannot be distinguished from the normal trading transactions of
the entity.
Government grants are sometimes called by other names such as subsidies, subventions, or premiums.

246
Example:
The government provided following to XYZ Limited:
(i) Free technical advice
(ii) Free marketing advice for export to Central Europe
(iii) Free provision of guarantees for export trade with European Countries.
(iv) Supportive government procurement policy that is responsible for significant sales by
XYZ Limited.
Required: Briefly explain whether the above benefits provided by government will be
considered as government grant.
Answer:
Items (i) to (iii) are not government grants as these are government assistance that cannot
reasonably have a value placed upon them.
Item (iv) is not government grant as it cannot be distinguished from general trading conditions.

Grants related to assets are government grants whose primary condition is that an entity qualifying for them
should purchase, construct or otherwise acquire long‑ term assets. Subsidiary conditions may also be attached
restricting the type or location of the assets or the periods during which they are to be acquired or held.

Grants related to income are government grants other than those related to assets.
Example:
Identify the following government grants as either “related to assets” or “related to income”.
(i) Grant by Federal Government on condition to import and install new power generation
plant in Pakistan.
(ii) Grant by Provincial Government on condition of construction and operation of factory
in a specific rural area.
(iii) Grant by sports ministry for conducting a Football League for next three years.
(iv) Grant by ministry of manpower for maintaining low labour turnover in last five years.
Answer:
Item (i) and (ii) are grant related to assets while item (iii) and (iv) are grant related to income.

247
RECOGNITION
Recognition criteria [IAS 20: 7 & 8]
Government grants, including non‑ monetary grants at fair value, shall not be recognised until there is reasonable
assurance that:
a) the entity will comply with the conditions attaching to them; and
b) the grants will be received.
Fair value is the price that would be received to sell an asset or paid to transfer a liability in an orderly
transaction between market participants at the measurement date.

Approaches to recognition [IAS 20: 13]


There are two broad approaches to the accounting for government grants:

Capital approach Under this approach, grant is recognised outside profit or loss (e.g. directly in equity).
IAS 20 does not allow this approach.
SPOTLIGHT

Income approach Under this approach, a grant is recognised in profit or loss over one or more period.
This approach is applied in accordance with IAS 20.

Example:
State the time of recognition of income related to following government grants:
a) The grant was received for maintaining good working conditions in the past.
b) The grant was received for maintaining certain working conditions for next three years.
c) The grant was received for installation of a plant that has useful life of 15 years and being
depreciated using 30% reducing balance method.
d) The grant was awarded to facilitate the acquisition of land subject to condition of
building a factory thereon.
e) The grant was awarded to facilitate the acquisition of land for dairy farming subject to
condition of maintaining minimum 70% local employment for next 10 years.
⯈ Answer:

AT A GLANCE
a) Recognise immediately in profit or loss
b) Recognise in profit or loss over next three years
c) Recognise in profit or loss over 15 years in proportion to depreciation expense
recognised using 30% reducing balance method.
d) Recognise in profit or loss over the useful life of building.
e) Recognise in profit or loss over next 10 years.

1.2 Forgivable loan [IAS 20: 3 & 10]


Forgivable loans are loans which the lender undertakes to waive repayment of under certain prescribed
conditions.
SPOTLIGHT

A forgivable loan from government is treated as a government grant when there is reasonable assurance that the
entity will meet the terms for forgiveness of the loan. Until then, such a loan is treated as a liability in accordance
with IFRS 9.
Example:
ABC Pharmaceutical Company received cash from government for a research and development
project of a children vaccine. As per the terms of the loan, the cash received from the government
shall be waived, if the entity is able to develop the vaccine within 3 years and sell it free of cost
for 5 years.
Required: Briefly explain the accounting treatment of the above loan?

248
Answer:

This is forgivable loan as the repayment shall be waived, under prescribed conditions i.e. ability to develop
vaccine within 3 years and sell it free of cost for 5 years.
If there is reasonable assurance to meet the conditions of waiver, this forgivable loan shall be recognised
as government grant.
However, if there is expectation that it will take more time than three years in the development or there is
expectation of selling the vaccine for a price before 5 years, the loan shall be recognised as a liability in
accordance with IFRS 9.

Example:
ABC Pharmaceutical Company received cash from government for a research and development project of
a children vaccine. As per the terms of the loan, the cash received from the government is repayable in
cash only if the entity decides to commercialize the results of the research phase of the project.
If the entity decides not to commercialize the results of the research phase, the cash received is not
repayable in cash, but instead the entity must transfer to the government the rights to the research.
Required: Explain whether the loan will be considered a forgivable loan?

Answer:
In this scenario, cash received from the government does not meet the definition of a forgivable loan in IAS 20.
This is because, in this arrangement, the government does not undertake to waive repayment of the loan,
but rather to require settlement in cash or by transfer of the rights to the research.
The cash received from government shall be recognised as liability in accordance with IFRS 9.

Benefit of loan at below-market rate of interest [IAS 20: 10A]


The benefit of a government loan at a below market rate of interest is treated as a government grant. The benefit
of below market rate of interest shall be measured as the difference between the cash receipt under the
government loan and the initial carrying amount of the loan measured and recognised in accordance with IFRS
9.
AT A GLANCE STICKY NOTES

The entity shall consider the conditions and obligations that have been, or must be, met when identifying the
costs for which the benefit of the loan is intended to compensate.
⯈ Example:
On 1 January 2023, MZ Limited received a loan under government support scheme whereby the
loan of Rs. 100,000 is provided at a mark-up rate of 5% per annum whereas market rate of
interest for similar loan is 12% annum. The loan is for immediate financial support and is
repayable on 31 December 2023.
MZ Limited has determined the initial carrying amount of loan in accordance with IFRS 9 at Rs.
93,750 (i.e. Rs. 100,000 + 5% x 100,000 = Rs. 105,000 x 1.12-1)
Required: Prepare the journal entry on 1 January 2023 on receipt of the above loan.
SPOTLIGHT

⯈ Answer:

Debit Credit
Date Particulars
Rs. Rs.
1 Jan 2023 Cash/Bank 100,000
Government grant (profit or loss) 6,250
Loan (liability) 93,750

249
Example:

SPOTLIGHT
On 31 December 2020, JKL Limited received grant of Rs. 50,000 towards the cost of training
young apprentices. The training program is expected to last for two years.
Actual total cost of training was Rs. 200,000 (70% incurred in year 2021 and 30% incurred in
year 2022 as originally planned).
Required: Prepare financial statement extracts under both methods of presentation. Year end is
31 December.
Answer:

Presentation method: Other income 2020 2021 2022


Statement of financial Position (extracts) Rs. Rs. Rs.

STICKY NOTES
Non-current liabilities: Deferred grant (30%) 15,000 - -
Current liabilities: Deferred grant (70%) 35,000 15,000 -
Statement of profit or loss (extracts)
Training costs - (140,000) (60,000)
Other income: training grant - 35,000 15,000
Presentation method: Net expense 2020 2021 2022
Statement of financial Position (extracts) Rs. Rs. Rs.
Non-current liabilities: Deferred grant 15,000 - -
Current liabilities: Deferred grant 35,000 15,000 -
Statement of profit or loss (extracts)
Training costs (net) - (105,000) (45,000)

Example:
STICKY NOTES

On 1 January, 2021 ABC Limited acquired a plant at a cost of Rs. 600 million and received a grant
of Rs. 60 million on the same date.
The plant is to be depreciated on straight line basis over its useful life of 3 years and Rs. 120
million residual value. There is reasonable assurance that conditions of the grant shall be
complied with.
Required: Prepare financial statement extracts under both methods of presentation. Year end is
31 December.
Answer:

Presentation: Separate deferred income 2021 2022 2023


Statement of financial Position (extracts) Rs. m Rs. m Rs. m
PPE (Cost) 600 600 600
Accumulated depreciation (160) (320) (480)
440 280 120

250
Presentation: Separate deferred income 2021 2022 2023
Statement of financial Position (extracts) Rs. m Rs. m Rs. m
Non-current liabilities: Deferred grant 20 - -
Current liabilities: Deferred grant 20 20 -
Statement of profit or loss (extracts)
Depreciation [(600 – 120) / 3 years] (160) (160) (160)
Other income: grant [60 / 3 years] 20 20 20

Method 2: Deduct from asset’s carrying amount 2021 2022 2023

AT A GLANCE
Statement of financial Position (extracts) Rs. m Rs. m Rs. m
PPE (Cost) [600 – 60] 540 540 540
Accumulated depreciation (140) (280) (420)
400 260 120
Statement of profit or loss (extracts)
Depreciation [(540 – 120) / 3 years] (140) (140) (140)

Example:
SPOTLIGHT

On 1st January 2020, Deep Sea Limited installed a non-current asset with a cost of Rs. 500,000
and received a grant of Rs. 100,000 in relation to that asset. The asset is being depreciated on a
straight-line basis over five years.
Grant of Rs. 90,000 was repaid on 1st January 2022 on failing to meet the few conditions of grant.
Required: Journal entries for the year 2020 to 2022 (under both methods of presentation).
⯈ Answer:
Presentation method: Setting up the grant as deferred income

Debit Credit
STICKY NOTES

Date Particulars
Rs. 000 Rs.000
1 Jan 2020 PPE 500
Cash 500
1 Jan 2020 Cash 100
Deferred grant 100
31 Dec 2020 Depreciation [500 / 5 years] 100
Accumulated depreciation 100
31 Dec 2020 Deferred grant [100 / 5 years] 20
Other income P&L 20
31 Dec 2021 Depreciation [500 / 5 years] 100
Accumulated depreciation 100

251
Debit Credit
Date Particulars
Rs. 000 Rs.000
31 Dec 2021 Deferred grant [100 / 5 years] 20
Other incomeP&L 20
1 Jan 2022 Deferred grant[90- 90 / 5 x 2 years] 54
Other expenses Profit or loss (bal) 36
Cash 90
31 Dec 2022 Depreciation [300 / 3 years] 100
PPE 100

AT A GLANCE
31 Dec 2022 Deferred grant [(60-54) / 3 years] 2
Other income P&L 2

Presentation method: Deducting the grant in the carrying amount of an asset

Debit Credit
Date Particulars
Rs. 000 Rs.000
1 Jan 2020 PPE 500
Cash 500
1 Jan 2020 Cash 100

SPOTLIGHT
PPE 100
31 Dec 2020 Depreciation [400 / 5 years] 80
Accumulated depreciation 80
31 Dec 2021 Depreciation [400 / 5 years] 80
Accumulated depreciation 80
1 Jan 2022 PPE 90
Cash 90

STICKY NOTES
Depreciation [90 / 5 x 2 years] 36
Accumulated depreciation 90
31 Dec 2022 Depreciation [(500-100+90) / 5 years] 98
Accumulated depreciation 98

252
Example:
Adeel Limited (AL) imported and installed a plant at total cost of Rs. 250 million on 1 January
2021. The plant has useful life of 3 years. The residual value of plant at the end of useful life has
been estimated at Rs. 128 million. Based on this AL has correctly determined depreciation rate
of 20% under reducing balance method that it uses for depreciating plant and machineries.
On the same date, AL also received a government grant of Rs. 60 million towards this plant. It is
reasonably certain that AL will comply with the conditions of this grant. AL has policy to present
the plant and grant separately in its financial statements.
AL year-ends on 31 December.
Required:
AT A GLANCE

Prepare journal entries in the books of AL in respect of above plant from 1 January 2021 to 31
December 2023 (Journal entry for disposal of plant is not required).
⯈ Answer:
Tutorial note: Grants related to depreciable assets are usually recognised in profit or loss over
the periods and in the proportions in which depreciation expense on those assets is recognised
(IAS 20.17).

Year Annual depreciation Rs. m Grant income Rs. m


2021 Rs. 250m x 20% 50 Rs. 60m x 50/122 24.59
2022 Rs. (250m-50m) x 80% 40 Rs. 60m x 40/122 19.67
2023 Rs. (250m-50m-40m) x 32 Rs. 60m x 32/122 15.74
SPOTLIGHT

80%
Total 122 Total 60

Journal entries

Debit Credit
Date Particulars
Rs. m Rs. m
1 Jan 2021 Property, plant & equipment 250
Bank 250
Bank 60
STICKY NOTES

Deferred government grant 60


31 Dec 2021 Depreciation 50
Accumulated depreciation 50
Deferred government grant 24.59
Other income Profit or loss 24.59
31 Dec 2022 Depreciation 40
Accumulated depreciation 40
Deferred government grant 19.67
Other income Profit or loss 19.67
31 Dec 2023 Depreciation 32
Accumulated depreciation 32
Deferred government grant 15.74
Other income Profit or loss 15.74

253
⯈ Example:
Kashif Limited (KL) imported and installed a plant at total cost of Rs. 250 million on 1 January
2021. The plant has useful life of 3 years. The residual value of plant at the end of useful life has
been estimated at Rs. 128 million. Based on this KL has correctly determined depreciation rate
of 20% under reducing balance method that it uses for depreciating plant and machineries.
On the same date, KL also received a government grant of Rs. 60 million towards this plant. It is
reasonably certain that KL will comply with the conditions of this grant. KL has policy to present
the grant as deduction from the carrying amount of the plant in its financial statements.
KL year-ends on 31 December.
Required:
Prepare journal entries in the books of KL in respect of above plant from 1 January 2021 to 31

AT A GLANCE
December 2023 (Journal entry for disposal of plant is not required).
⯈ Answer:
Tutorial note: Grants related to depreciable assets are usually recognised in profit or loss over
the periods and in the proportions in which depreciation expense on those assets is recognised
(IAS 20.17).

Reduced
Annual Depreciation
Year Rs. m Grant income Rs. m
depreciation
Rs. m
2021 Rs. 250m x 20% 50 Rs. 60m x 50/122 24.59 25.41

SPOTLIGHT
2022 Rs. 50m x 80% 40 Rs. 60m x 40/122 19.67 20.33
2023 Rs. 40m x 80% 32 Rs. 60m x 32/122 15.74 16.26
Total 122 Total 60 62

Journal entries
Debit Credit
Date Particulars
Rs. m Rs. m
1 Jan 2021 Property, plant & equipment 250
Bank 250

STICKY NOTES
Bank 60
Property, plant & equipment 60
31 Dec 2021 Depreciation (50-24.59) 25.41
Accumulated depreciation 25.41
31 Dec 2022 Depreciation (40-19.67) 20.33
Accumulated depreciation 20.33
31 Dec 2023 Depreciation (32-15.74) 16.26
Accumulated depreciation 16.26

254
Allah wants to forgive us, We need repent often and ask Allah to forgive us.

Investment Property IAS 40


Investment property:
1. it is a property (land or buildings or part of a building or both)
2. held by an owner
3. to earn rentals or for capital appreciation or both, rather than for:
• use in the production or supply of goods and services or for administrative use (Owner-occupied
property IAS 16);or
• sale in ordinary course of business ( Inventories IAS 2)

Examples of investment property

The following are examples of investment property:


• Land held for long-term capital appreciation rather than for short-term sale in the ordinary course of
business.
• Land held for a currently undetermined future use. (if an entity has not yet determined that it will use
the land as owner-occupied property or for short-term sale in the ordinary course of business, the
land is regarded as asset held for capital appreciation (means treated as an investment property))
• Property that is being constructed or developed for future use as investment property.

Followings are not investment property


The following are examples of items that are not investment property:
• property intended for sale in the ordinary course of business (IAS 2; e.g. property acquired
exclusively with a view to subsequent disposal in the near future (e.g. property dealers) or for
development and resale (housing societies));
• owner-occupied property i.e.
a. property held for future use as owner-occupied property (IAS 16)
b. property held for future development and subsequent use as owner-occupied property,
c. property occupied by employees (whether or not the employees pay rent at market rates) and
d. owner-occupied property awaiting disposal;

Partly occupied buildings


An entity might use part of a property for the production or supply of goods or services or for
administrative purposes and hold another part of the same property to earn rentals or for capital
appreciation. In other words, a part of a property might be owner occupied and a part held as an
investment property. The two parts are accounted for separately if they could be sold separately.

If this is not the case, the entire property is investment property only if an insignificant portion is held for
use in the production or supply of goods or services or administrative purpose.

Example: joint use properties


How should Stunning Ltd account for the following properties in its financial statements:
A. Stunning Ltd owns two buildings on two separate sites in Islamabad. The first building is used by
Stunning Ltd for administration purposes and the second building is leased out (means given on rent)

Page 1 of 23

255
B. Stunning Ltd owns a 20 storey building in Karachi, which it uses for administration purposes. The top
floor of the building is leased to Unpleasant Ltd (assume portions cannot be sold separately)
C. Stunning Limited owns a 20-storey building in Lahore. It leases out 19 floors and uses the top floor for
the administration of the building (assume portions cannot be sold separately)

Solution to example: joint use properties

A. There are two distinct and separate buildings: owner-occupied and leased out. Since the two
buildings are on two separate sites, it is assumed that they can be sold separately. The owner-
occupied building must be accounted for as property, plant and equipment as it is used for
administrative purposes. The leased out buildings must be accounted for as investment property.
B. There are two components of same building: owner-occupied and leased out. Since these two
components are within one building and therefore cannot be sold separately. Nineteen of the twenty
floors is owner-occupied and is therefore the significant component. Stunning Ltd must therefore
report the entire building as owner-occupied (i.e. as property, plant and equipment under IAS 16).
C. There are two components of the same building: owner-occupied and leased out. Since these two
components are within one building and therefore cannot be sold separately. Nineteen of the twenty
floors are leased out therefore investment property portion is the significant component. Stunning Ltd
must therefore report the entire building as investment property under IAS 40.

Accounting treatment of investment property


The recognition criteria for investment property are the same as for property, plant and equipment
under IAS 16. An owned investment property should be recognised as an asset only when:
• It is probable that future economic benefits associated with the property will flow to the entity; and
• The cost of the property can be measured reliably.

Measurement at initial recognition


Investment property should be measured initially at cost plus any directly attributable expenditure
(means transaction taxes e.g. legal fees, property transfer taxes and other transaction costs) incurred to
acquire the property.

The followings are not added to cost of an investment property:


• Start-up costs (unless necessary to bring the property to the condition necessary for it to be capable
of operating in the manner intended by management);
• Operating losses incurred before the investment property achieves the planned level of occupancy
(e.g initially property is not fully on rent because of which there are losses); or
• Abnormal waste incurred in constructing or developing the property.
• Selling and administrative expenses.

The accounting treatment of exchange of investment property is similar to PPE.


Measurement after initial recognition
After initial recognition an entity may choose as its accounting policy:
• The fair value model; or
• The cost model.
The chosen policy must be applied to all the investment property of the entity.
Once a policy has been chosen it cannot be changed unless the change will result in a more appropriate
presentation. IAS 40 states that a change from the fair value model to the cost model is unlikely to
result in a more appropriate presentation.

Page 2 of 23

256
Fear Allah wherever you are. Allah is one who knows what is in every heart.

Even if the entity uses cost model, fair value of the property is to be disclosed in notes to the financial
statements.

Fair value model for investment property


Definition: Fair value
Fair value is the price that would be received on selling an asset or paid to settle a liability in an orderly
transaction between market participants at the measurement date.
Under the fair value model the entity should:
• revalue all its investment property to ‘fair value’ at the end of each financial year; and
• recognise any resulting gain or loss in profit or loss for the period.
The investment property measured under fair value model would not be depreciated.
This is different to the revaluation model of IAS 16, where gains are accumulated as a revaluation
surplus within equity.

Cost model for investment property


The cost model in IAS 40 follows the provisions of IAS 16. The property is measured at cost less
accumulated depreciation (related to the non-land element) and less accumulated impairment loss if any.

Example: Accounting for investment property


On 1 January 2011 Entity P purchased a building for its investment potential. The building cost Rs. 1
million with transaction costs of Rs. 10,000.

The depreciable amount of the building component of the property at this date was Rs. 300,000.
The building element has a useful life of 50 years.
At the end of 2011 the property’s fair value had risen to Rs. 1.3 million.

Page 3 of 23

257
Answer:
Cost model
The amounts which would be included in the financial statements of Entity P at 31 December 2011,
under the cost model are as follows:

The property will be included in the statement of financial position as follows:


Rs.
Cost (1,000,000 + 10,000) 1,010,000
Accumulated depreciation (300,000 ÷ 50 years) (6,000)

Carrying amount 1,004,000


The statement of profit or loss will include depreciation of Rs. 6,000. Fair value of Rs. 1.3 millions will be
disclosed in notes to financial statements.
Fair value model
The amounts which would be included in the financial statements of Entity P at 31 December 2011,
under the fair value model are as follows:
The property will be included in the statement of financial position at its fair value of Rs.
1,300,000.
The statement of profit or loss will include a gain of Rs. 290,000 (Rs. 1,300,000 – Rs.
1,010,000) in respect of the fair value adjustment.

Example: In the light of poor market interest rates, the financial controller of Abbott Inc was instructed by
the managing director, in June 2019 to invest some of the company’s surplus cash in a plot of land
costing 1 million. This land may be used by Abbott Inc in the future to build a new factory on, or it may be
sold to realize a profit, depending on property prices in the coming years.

It is now the year end, June 30, 2019 and the financial controller is preparing Abbott’s financial
statements for the presentation to the Board. He knows that the land has fallen slightly in value to
950,000, but is unsure of how to account for it.

Required: Advise the financial controller about the correct accounting treatment.

A. land is to be classified as Investment property in the financial statements. According to IAS 40 land
held for currently undetermined future use be classified as Investment Property (means if an entity has
not determined that whether it will use the land as owner occupied property (IAS 16) or for short term
sale in the ordinary course of business (IAS 2), the land is regarded as held for capital appreciation)
As per IAS 40 the company has a choice of using the cost model or the fair value model. If cost model is
used then land will be carried as cost.

If the company uses fair value model then a fair value loss of 50,000 should be accounted for in the
financial statements for the year ended June 30, 2019 and land will be carried at 950,000 in the
statement of financial position.

Example: D Ltd. owns a factory with an opening carrying amount of 40 Million. At 1 Jan 2019, the
directors decided to sell the property but have continued to use the factory for manufacturing during the
year.

They wish to classify the building as an investment property and recognize a gain of 32 Million in the
income statement based on the market value at 31 Dec 2019 of 72 Million. It is estimated the factory has
a remaining life of 20 years on 01.01.2019. D Ltd. Apply the cost model for owner occupied properties
(IAS 16).

Page 4 of 23

258
“Verify news/message before you spread.”

Required: Discuss the director’s proposed accounting treatment of the property.

A. For a property to be classified as an investment property, following conditions in the definition must
be met.
It is a property (land or building or part of a building or both) held by the owner to earn rentals or
for capital appreciation or both.

In addition IAS 40 specifically states that owner occupied property awaiting disposal is not an Investment
property.

In light of the above, it is quite evident that D Ltd. Is currently using the property as owner occupied
therefore they will continue to recognize the said property under IAS 16 using cost model. Therefore
there is no question of recording the fair value gain.

Page 5 of 23

259
“If you want that Allah love you then obey Allah.”

FAR-1 QB
IAS-40

Question:
V Ltd owns several properties and has a year end of 31 December. Wherever possible, V Ltd carries
investment properties under the fair value model.
Property 1 was acquired on 1 January 2011. It had a cost of Rs. 1 million, comprising Rs. 500,000 for
land and Rs. 500,000 for buildings. The buildings have a useful life of 40 years. V Ltd uses this property
as its head office.
Property 2 was acquired many years ago for Rs. 1.5 million for its investment potential. On 31
December 2017 it had a fair value of Rs. 2.3 million. By 31 December 2018 its fair value had risen to Rs.
2.7 million. This property has a useful life of 40 years.
Property 3 was acquired on 30 June 2012 for Rs. 2 million for its investment potential. The directors
believe that the fair value of this property was Rs. 3 million on 31 December 2017 and Rs. 3.5 million on
31 December 2018. However, due to the specialised nature of this property, these figures cannot be
corroborated (means not reliable figures). This property has a useful life of 50 years.
Required
a) For each of the above properties briefly state how it would be treated in the financial statements of V
Ltd for the year ended 31 December 201 8, identifying any impact on profit or loss.
b) Produce an analysis of properties of V Ltd showing movement in opening and closing balances for
the year ended 31 December 201 8, showing each of the above properties separately (means a
schedule).

Solution:
a) Treatment in the financial statements for the year ended 31 December 2018

Property 1
This is used by V Ltd as its head office and therefore cannot be treated as an investment property. It will
be stated at cost minus accumulated depreciation in the statement of financial position, unless revalued
under IAS 16 the depreciation for the year will be charged in the statement of profit or loss.

Property 2
This is held for its investment potential and should be treated as an investment property. It will be carried
at fair value, V Ltd policy of choice for investment properties. It will be re-measured to fair value at each
year end and any resultant gain or loss taken to the statement of profit or loss (Rs. 400,000 gain in 2018)
(2.7M – 2.3M)

Property 3
This is held for its investment potential and should be treated as an investment property. However, since
its fair value cannot be measured reliably it will be held at cost minus accumulated depreciation in the
statement of financial position. The depreciation for the year will be an expense in the statement of profit
or loss.

This situation provides the exception to the rule whereby all investment properties must be held under
either the fair value model, or the cost model.

Page 6 of 23

260
b) Analysis of property, plant and equipment for the year ended 31 December 2018
Investment Investment
Head Office
property held atproperty held at Total
Property (W1)
fair value cost (W2)

Rs. Rs. Rs. Rs.


Cost/valuation
On 1 January 2018 1,000,000 2,300,000 2,000,000 5,300,000
Revaluation - 400,000 - 400,000
On 31 December 2018 1,000,000 2,700,000 2,000,000 5,700,000

Accumulated
depreciation
On 1 January 2018 (W-1) 87,500 - (W-2) 220,000 307,500
Charge for the year (W1) 12,500 - (W-2) 40,000 52,500
On 31 December 2018 100,000 - 260,000 360,000
Carrying amount

On 31 December 2018 900,000 2,700,000 1,740,000 5,340,000


On 31 December 2017 912,500 2,300,000 1,780,000 4,992,500

*Note: There is no concept of charging depreciation of Investment properties if carried at Fair value
Model of IAS-40

Workings
(1) Depreciation on Property 1 Rs.
Brought forward (500,000 ÷ 40 x 7) 87,500
2018 (500,000 ÷ 40) 12,500
(2) Depreciation on Property 3
Rs.
Brought forward (2,000,000 ÷ 50 x 5.5) 220,000
2018 (2,000,000 ÷ 50) 40,000

Page 7 of 23

261
And whoever relies upon Allah – then He is sufficient for him. (A promise of Allah)

Introduction of other comprehensive income

Change in use of property


The entity may, during the current reporting period, change the use of the property.
There are following possible changes:

From To
Owner-occupied property (IAS 16) Investment property (IAS 40)
Inventories (IAS 2) Investment property (IAS 40)
Investment property(IAS 40) Owner-occupied property (IAS 16)
Investment property(IAS 40) Inventories (IAS 2)

If the entity uses the cost model (for properties in IAS 40)
If the entity uses the cost model, a change in use will not change the carrying amount of the property
because (a) investment property, (b) property plant and equipment and (c) inventory are all carried at
their cost.

If the entity uses the fair value model (for properties in IAS 40)
If the entity uses the fair value model then there may be measurement implications.

a) Change from owner-occupied property to investment property


The entity must first revalue the property, plant and equipment to its fair value and the resultant increase
or decrease is recognised in accordance with the standard on property, plant and equipment (IAS 16):

An increase is:
o first credited to profit or loss (only where it reverses a previous loss); and
o then credited to revaluation surplus (in other comprehensive income) as in IAS 16;

A decrease is:
• first debited to the revaluation surplus (if the revaluation surplus account has a balance as a result of
previous revaluations); and
• the excess is then debited to profit or loss (revaluation loss).

Example 1: change from owner-occupied to investment property


Fantastic Limited had its head office located in Murree. During a heavy landslide on 30 June 2015, a
building nearby, which it had rented to Honda Limited, was destroyed. As Honda Limited was a valued
customer, Fantastic Limited decided to lease 80% of the head office to them as a ‘replacement’ (assume
portions cannot be sold separately).

The head office was purchased on the 1 January 2015 for 600 000 (total useful life: 10 years)
On the 30 June 2015, the fair value of the head office was 800 000. There was no change in fair value at
31 December 2015.

Fantastic Ltd uses:


• the fair value model to measure its Investment Property; and
• the cost model to measure its property, plant and equipment.
Required:
Provide the journal entries in the books of Fantastic Ltd for the year ended 31 December 2015.

Page 8 of 23

262
b) Change from inventories to investment property
Property that is intended for sale in the ordinary course of business is carried according to IAS 2. In case
it is classified as an investment property then the property is transferred to investment property at its
carrying amount in accordance with IAS 2 (i.e. lower of cost and NRV) and then revalued to fair value in
accordance with IAS 40 at the date of transfer, with any difference going to profit or loss.

Example 2:
Marvelous Limited has a building purchased with the intension of sale. On 30.06.2015 the management
decided to lease it out to earn rentals. On 30.06. 2015 the fair value was 1,300,000. The cost of
purchase on 01.01.2015 was 1,000,000. There was no change in fair value at 31 December 2015.

Marvelous Limited uses the fair value model to measure its Investment Property

Required:
Provide the journal entries for the year ended 31 December 2015.

c) Change from investment property to owner-occupied property or inventories

The entity must first adjust the property’s carrying amount to fair value on the date of change. The
resultant change must be taken to the profit or loss as a gain or loss caused by a fair value adjustment in
accordance with IAS 40. The fair value on date of transfer, determined in accordance with IAS 40, will be
treated as cost of the owner-occupied property or inventory for future valuation. If the investment
property is classified as owner-occupied, it will then be depreciated over the remaining useful life and
measured in terms of IAS 16: Property, plant and equipment. If the investment property is classified as
inventory, it will then be measured in terms of IAS 2: Inventories i.e. at lower of cost and NRV.

Example 3: change from investment property to owner occupied property


Super Limited owned and leased out a building in Peshawar, which was correctly classified as an
investment property on 31 December 2014 and carried at fair value model.

During an earthquake the head office of Super Limited was destroyed, with the result that Super Ltd had
to relocate its head office into the Peshawar Building. The tenants of this building were forced to move
out as of 30 June 2015.

The fair value of the building on 31 December 2014 was 200 000.

On the 30 June 2015 the buildings fair value was 260 000 and had a remaining useful life of 10 years.

Required:
Provide the journal entries in Super Limited’s records for the year ended 31 December 2015.

Answer:
Solution to example 1
1 January 2015
Head-office building: (PPE) 600,000
Bank/ liability 600,000
30 June 2015
Depreciation 30,000
Acc depreciation 30,000
Depreciation to date of change
in use (600 000 / 10 x 6 / 12
months)
30 June 2015
Acc depreciation 30,000
Head-office building: (PPE) 30,000

Page 9 of 23

263
30 June 2015
Head-office building: (PPE) 230,000
Revaluation surplus 230,000
Revaluation of head office to
fair value on date of change in
use (800 000 – (600 000 – 30
000))
30 June 2015
Investment property 800,000
Head-office building: (PPE) 800,000
31.12.2015 No further depreciation and no fair value gain or loss.
31 december 2015
Revaluation surplus 12,105
Retained earnings 12,105
[230,000/9.5x6/12]

Solution to example 2
01.01.2015
Inventory 1,000,000
Bank 1,000,000
30.06.2015
Investment property 1,000,000
Inventory 1,000,000
30.06.2015
Investment property 300,000
Fair value gain 300,000
(1,300,000 -1,000,000)

Solution to example 3
30.06.2015
Investment property 60,000
Fair value gain 60,000
(260,000 -200,000)
Head-office building: (PPE) 260,000
Investment property 260,000
31.12.2015
Depreciation 13,000
Acc depreciation 13,000
Depreciation to date of change in use
(260 000 / 10 x 6 / 12 months)

Page 10 of 23

264
Practice questions:
Question-1: (Investment property to owner-occupied property)
ABC limited company owns two buildings:
• A head office building located in Quetta; and
• Another office building located in Karachi.

The office building located in Quetta is used as ABC Limited’s head office. A minor earthquake, on 30
June 2015 destroyed this building.
• The building in Quetta was purchased on 1 January 2015 for 1,200,000 (total useful life 10 year and
residual value: nil).

The property in Karachi was leased to a tenant, Spider limited. After the earthquake, ABC limited
urgently needed new premises for its head office. Since spider limited was always late in paying their
lease rentals, ABC limited decided to immediately evict them and move their head office to this building
situated in Karachi.
• The building in Karachi was purchased on the 1 January 2015 for 500,000
• On the 30 June 2015 the fair value of the building in Karachi was 950,000
• There was no change in fair value at 31 December 2015
• The total useful life was estimated to be 10 years from date of purchase and the residual value was
estimated to be nil.

ABC limited uses:


• The cost model to measure its property, plant and equipment; and
• The fair value model for its investment properties.

Required:
Journalize the above transactions in the books of ABC limited for the year ended 31 December 2015.

Question-2: (Owner occupied property to investment property)


XYZ Limited owns an office block.
• XYZ Limited had occupied the office block from date of purchase until 30 June 2015.
• The office block had cost 1,000,000 on 1 January 2014
• Its residual value is estimated to be nil and total useful life is estimated to be 10 years respectively
(both estimates have remained unchanged).
• On 30 June 2015, XYZ limited moved out of the office block and thereafter rented it to tenants under
short-term operating leases.
• The fair value of the office block was equal to its carrying amount on 30 June 2015
• The fair value of the office block was 800,000 on 31 December 2014 and 1,500,000 on 31 December
2015.

XYZ limited measure owner-occupied property using the cost model and investment property using the
fair value model.

Required:
Show all journals relating to the office block in the books of XYZ limited for the year ended 31 December
2015

Solution-1:
Debit Credit
1 January 2015
Quetta building: cost (PPE) 1,200,000
Bank/liability 1,200,000
Purchase of Quetta building (owner-occupied)
Karachi building: cost (Investment property) 500,000
Bank/liability 500,000

Page 11 of 23

265
Purchase of Karachi building (leased to a tenant)
30 June 2015
Depreciation (1,200,000/10 x 6/12 60,000
months)
Accumulated deprecation 60,000
Depreciation of building (PPE) to date of destruction
Acc. Dep 60,000
loss (bal.) (1,200,000 - 60,000) 1,140,000
Quetta building: Acc. Dep and impairment 1,200,000
losses (PPE)
Write-off after earthquake
Karachi building: cost (investment prop) (950,000 – 500,000) 450,000
Fair value gain (P&L) 450,000
Revaluation of investment property prior to change in use
Karachi building: PPE 950,000
Karachi building (Investment property) 950,000
Transfer from investment property to property plant and equipment
31 December2015
Depreciation 950,000/9.5 x 6/12 months 50,000
Karachi building: accumulated depreciation 50,000
Depreciation to year end Karachi building (PPE)

Solution-2:
a) Office Block
Debit Credit
30 June 2015
Depreciation (1,000,000 – 0)/ 10y x 6/12 50,000
Office block: Acc. Dep (PPE) 50,000
Depreciation on the owner-occupied office block to date of change in use
Acc. dep. O/bal (1,000,000 – 0)/10 y+ 150,000
20X5 Dep 50,000 (above)
Office block: PPE 150,000
No revaluation surplus or loss
30.06.2015
Investment property 850,000
Head office building 850,000

Transfer from property plant and equipment to investment property on the


date of change in use
31 December 2015
Office block: (Investment property) (1,500,000 – 850,000) 650,000
Fair value adjustment to investment property 650,000
(income)
Investment property re-measured to fair value at year-end

Gain or loss on disposal


Gains or losses on disposals of investment properties are included in profit or loss in the period in which
the disposal occurs.

Page 12 of 23

266
Every success is based on the deeds do great deeds for great success.”

Disclosure requirements

The following disclosures are required by IAS 40 in the notes to the accounts.

Disclosure requirements applicable to both the fair value model and the cost model

whether the fair value model or the cost model is used

the methods and assumptions applied in arriving at fair values

the extent to which the fair value of investment property was based on a valuation by a qualified,
independent valuer with relevant, recent experience

amounts recognised as income or expense in the statement of profit or loss for:
1. rental income from investment property
2. operating expenses in relation to investment property

details of any restrictions on the ability to realise investment property or any restrictions on the
remittance of income or disposal proceeds

the existence of any contractual obligation to purchase, construct or develop investment property or
for repairs, maintenance or enhancements (future commitments)

Disclosure requirements applicable to the fair value model only


There must be a reconciliation, in a note to the financial statements, between opening and closing values
for investment property, showing:

additions during the year

net gains or losses from fair value adjustments

This reconciliation should show separately any amounts in respect of investment properties included at
cost because their fair values cannot be estimated reliably.

For investment properties included at cost because fair values cannot be estimated reliably, the following
should also be disclosed:

a description of the property

an explanation as to why fair values cannot be determined reliably

if possible, the range within which the property’s fair value is likely to lie.

Disclosure requirements applicable to the cost model only



the depreciation methods used

the useful lives or depreciation rates used

gross carrying amounts and accumulated depreciation at the beginning and at the end of the period

A reconciliation between opening and closing values showing:
I. additions
II. depreciation
III. impairment losses
IV. transfers

When the cost model is used, the fair value of investment property should also be disclosed. If the fair
value cannot be estimated reliably, the same additional disclosures should be made as under the fair
value model.

Page 13 of 23

267
Extra practice questions (before this question revise cash flow questions Q. 6,7,8 and Drum
limited )

Q. Following are the extracts from the financial statements of Sunday Traders Limited (STL) for the
year ended 30 June 2019:

Statement of financial position as on 30 June 2019


2019 2018 2019 2018
Assets Rs. in million Equity & liabilities Rs. in million
Property, plant and Share capital (Rs. 100
equipment 8,555 7,240 each) 4,650 3,450
Investment property 1,800 1,120 Share premium 1,600 1,240
Stock in trade 4,800 4,500 Retained earnings 1,652 (655)
Prepayments 184 268 Long term loans 6,024 6,523
Trade receivables 3,800 3,600 Trade payables 3,422 5,390
Contract liability (advance
Cash 194 480 from customer) 250 40
Accrued liabilities 310 180
Interest payable 135 110
Current maturity of long
term loans 850 700
Provision for taxation 440 230
19,333 17,208 19,333 17,208

Statement of profit or loss for the year ended 30 June 2019


Rs. in million
Sales 29,700
Cost of sales (15,750)
Gross profit 13,950
Distribution cost (6,185)
Administrative cost (2,302)
Other income 404
Profit before interest and tax 5,867
Interest expense (1,210)
Profit before tax 4,657
Tax expense (1,150)
Profit after tax 3,507

Additional information:
(i) 72% of sales were made on credit.
(ii) Depreciation expense for the year amounted to Rs. 750 million which was charged to distribution
and administrative cost in the ratio of 3:1.
(iii) Distribution cost includes:
▪ Rs. 40 million in respect of loss on disposal of equipment. The written down value at the time
of disposal was Rs. 152 million.
▪ impairment loss on vehicles amounting to Rs. 24 million.
(iv) Loan instalments (including interest) of Rs. 1,984 million were paid during the year.
(v) Other income comprises of:
▪ increase in fair value of investment property amounting to Rs. 220 million.
▪ rent received from investment property amounting to Rs. 184 million.

Page 14 of 23

268
(vi) During the year, STL issued right shares at premium.

Required:
Prepare STL’s statement of cash flows for the year ended 30 June 2019 using direct method. (19)

A. Sunday Traders Limited


Statement of Cash Flows
For the year ended 30 June 2019
Cash flows from operating activities Rs. in million
Cash receipts from customers (8,316 + 21,394) 29,710
Cash receipts from tenants 184
Cash paid to suppliers (18,018)
Cash paid for expenses (7,459)
Cash generated from operations 4,417
Interest paid (1,185)
Income taxes paid (940)
Net cash inflow from operating activities 2,292

Cash flows from investing activities


Purchase of property, plant and equipment (2,241)
Proceeds from disposal of property, plant and equipment 112
Purchase of investment property (460)
Net cash outflow from investing activities (2,589)

Cash flows from financing activities


Proceeds from issue of shares (1,200 + 360) 1,560
Dividend paid (1,200)
Repayment of loans (799)
New loans acquired 450
Net cash inflow from financing activities 11
Net decrease in cash and cash equivalents (286)
Cash and cash equivalent at the beginning of the year 480
Cash and cash equivalent at the end of the
year 194

Workings
Property Plant and Equipment
b/d 7,240 Depreciation 750
Cash 2,241 Disposal 152
Impairment 24
c/d 8,555

Investment property
b/d 1,120
F.V gain 220
Cash 460
c/d 1,800

Page 15 of 23

269
Stock
b/d 4,500 COS 15,750
Purchases 16,050

c/d 4,800

Prepayments + Accrued liabilities


b/d 268 b/d 180
Cash 7,459 Expense 7,673
(6,185+2,302-750-40-24)
c/d 310 c/d 184

Trade receivable + Advance from customer


b/d 3,600 b/d 40
Sales 21,384 Cash 21,394
(29,700×72%)
c/d 250 c/d 3800

Share Capital
b/d 3,450
Cash 1,200

c/d 4,650

Share premium
b/d 1,240
Cash 360

c/d 1,600

Retained Earnings
b/d 655
Dividend 1,200 PAT 3,507

c/d 1,652

Loan
b/d 6,523
Cash 799 b/d 700
(1,984-1185) Cash 450
c/d 6,024
c/d 850

Trade payables
b/d 5,390
Cash 18,018 Purchases 16,050

c/d 3,422

Page 16 of 23

270
Sins Take you away from Allah. Salah takes you back to Allah.

Interest payable
b/d 110
Cash 1,185 Expense 1,210

c/d 135

Tax payable
b/d 230

Cash 940 Expense 1,150


c/d 440

Disposal
PPE 152
Cash 112
Loss 40

Extra practice question (IAS 16 and IAS 23)


Q. The following information pertains to Monday Limited (ML):

(i) The balances of property, plant and equipment as on 1 January 2018:


Cost/Revalued Accumulated
Assets amount depreciation
----------- Rs. in million -----------
Office building 240 36
Equipment 190 60

Revaluation surplus related to the office building as at 1 January 2018 amounted to Rs. 8.5
million.

(ii) On 1 September 2018, a new equipment was acquired by making payment of


Rs. 70 million to the supplier. An old equipment was also given in exchange to the
supplier. The fair values of old and new equipment were assessed at Rs. 21 million and
Rs. 93 million respectively. The old equipment had been acquired at a cost of Rs. 40
million on 1 July 2016. Cost incurred on installing the new equipment amounted to Rs. 5
millions.

(iii) On 1 January 2018, ML commenced construction of a manufacturing plant. The whole


process of assembling and installation was completed on 31 October 2018. However, the
work was stopped from 16 to 31 August 2018 due to unexpected rains.

The total cost of Rs. 660 million incurred on the plant was paid as under:
Description Payment date Rs. in million
1st payment 1 February 2018 140
2nd payment 1 April 2018 214
3rd payment 1 September 2018 146
4th payment 1 December 2018 160

Page 17 of 23

271
The plant was financed through a bank loan of Rs. 500 million obtained on 1 March 2018.
The loan carries a mark-up of 18% payable annually. The surplus funds available from the
loan were invested in a saving account and earned Rs. 17 million during capitalization
period.

(iv) On 31 December 2018, the revalued amount of office building was assessed at Rs. 178
million by Precise Valuers, an independent valuation firm.

(v) Other relevant details are as follows:


Assets Depreciation Life/rate Subsequent
Method Measurement
Office building Straight Line 20 years* Revaluation
Equipment Reducing Balance 20% Cost
Manufacturing Plant Straight Line 15 years Cost
* Remaining life at the date of last revaluation

ML accounts for revaluation on net replacement value method and transfers the maximum
possible amount from revaluation surplus to retained earnings on an annual basis.

Required:
In accordance with IFRSs, prepare a note on ‘Property plant and equipment’ for inclusion in ML’s
financial statements for the year ended 31 December 2018.
(Comparatives figures are not required) (17)

Answer
Monday Limited
Notes to the financial statements
For the year ended 31-12-2018
Rs. in million
Office Equipme Plant Total
Building nt
Gross carrying amount
Balance as on 1-1-2018 240 190 - 430
Addition - 96 699.25 795.25
(21+70+5) (W)
Disposal - (40) - (40)
Elimination (48) - - (48)
Revaluation (W) (14) - - (14)
Closing balance 178 246 699.25 1,123.25
Accumulated depreciation
Balance as on 1-1-2018 36 60 96
Depreciation 12 30.48* 7.77 50.25
(240/20) (699.25/15
X2/12)
Disposal - (15.04)** - (15.04)
Elimination (48) - - (48)
Closing balance - 75.44 7.77 83.21
Carrying Amount 178 170.56 691.48 1,040.04

Page 18 of 23

272
Allah wants to forgive us, We need to repent often and ask Allah to forgive us.

Office building Equipment Plant


Measurement basis Revaluation model Cost model Cost model
Useful life/depreciation 20 years 20% 15 years
rate
Depreciation method Straight line Reducing balance Straight line

The last revaluation was performed on 31.12.2018 by Precise Valuers, an independent firm of valuers.

The carrying amount of office building has the cost model being used instead:
240-10(revaluation surplus) = 230
Depreciation per annum = 230/20 = 11.5
230-(11.5 x 4 up to 31.12.2018) = 184

* Depreciation:
Opening WDV [190-60] 130
Less: opening WDv of disposal (28.8)
101.2
X 20% 20.24
+ 28.8 x 20% x 8/12 3.84
+96 x 20% x 4/12 6.40
30.48

**Accumulated depreciation of disposal:


Cost (1-7-2016) 40
X 20% X 6/12 (4)
31-12-2016 36
X 20% (7.2)
31-12-2017 28.8
X 20% X 8/12 (3.84)
1-9-2017 24.96
Acc. Dep [40-24.96] 15.04

Cost on WIP 660


(140+214+146+160)

Borrowing Cost : 56.25


500 X 18% X [(8-0.5)/12]
Less: Investment Income (17)
39.25
Total 699.25

Detail of payments
Owned Loan
1-2 140 140 -
1-4 214 - 214
1-9 146 - 146
1-12 160 20(bal) 140

Page 19 of 23

273
Revaluation [31-12-2018]
WDV [240- (36+240/20)] 192
FV 178
Revaluation Loss 14

R. Surplus 8 R. Loss 6
(8.5-0.5)

Rev. Surplus on 1-1-2018


240/20 = 12 X 3 = 36 [Three year after revaluation]

8.5 is the remaining balance after three years. So 8.5/17 = 0.5 will be transferred in current year.

Total surplus on last revaluation date would have been 8.5/17 x 20 = 10

Page 20 of 23

274
Q.1[IAS 8 with investment property] Change yourself to please ALLAH but not to please the people.
Following information have been
extracted from the financial statements of Fakhr Limited (FL) for the year ended 31 December 2019:

(i) 2019 2018 2017


Draft Audited Audited
--------- Rs. in million ---------
Net profit 84 98 72
Revaluation surplus arising during the year* 25 (14) -

*Transfer to retained earnings is made upon de-recognition of related asset.

(ii) Share capital and reserves as at 1 January:

2018 2017
----- Rs. in million -----
Share capital (Rs. 10 each) 300 300
Revaluation surplus 102 102
Retained earnings 348 276

(iii) On 1 March 2018, FL declared a final cash dividend of 10% for the year ended 31 December 2017.
On 1 November 2018, FL issued 40% right shares to its ordinary shareholders at Rs. 24 per share.
On 1 August 2019, an interim bonus of 15% was declared.

Following matter need to be incorporated in the draft financial statements of FL:


To provide more relevant and reliable information about investment property, it has been decided to
change the measurement basis for investment property from cost model to fair value model.
The only investment property of FL is a building purchased on 1 January 2016 at a cost of Rs. 150
million. 60% of the cost represents building component having estimated useful life of 20 years and
residual value of Rs. 10 million. The depreciation is included in the above draft financial statements. The
fair value of the investment property has increased by 6% in each year since acquisition.

Required:
Prepare FL’s statement of changes in equity (including comparative figures) for the year ended 31
December 2019. (‘Total’ column is not required) (18)

Page 21 of 23

275
A.1 Fakhr Limited
Statement of Changes In Equity
For the year ended 31 December 2019

Ordinary Share Revaluation Retained


share premium surplus earnings
capital
---------- Rs. in million ----------
Balance as at 31 December 2017 (As 300.00 - 102.00 348.00
given)
Effect of change in accounting policy - - - 26.54
[ 13 + 13.54 ] (Workings)
Balance as at 31 December 2017 300.00 - 102.00 374.54
– Restated
Final cash dividend @ 10% for 2017 - - - (30.00)
( 300 x 10% )
Right issue @ 40% 120.00 168.00 - -
( 300 / 10 x ( 300 / 10
40% x 10) x
40% x 14)
– Net profit: Restated (98 +14.11) - - - 112.11
(Workings)
- Revaluation Surplus - - (14.00) -
Balance as at 31 December 2018 420.00 168.00 88.00 456.65
– Restated
Interim bonus dividend @ 15% 63.00 - - (63.00)
for 2019 (420 x 15%)
Total comprehensive income for the year
ended 31 December 2019
– Net profit (84 + 14.72) - - - 98.72
- Revaluation Surplus - - 25.00 -
Balance as at 31 December 2019 483.00 168.00 113.00 492.37

(i) Change in Policy of Investment Property :

At Cost Model : At FV Model :


1-1-2016 1-1-2016
Investment Property 150 Investment Property 150
Cash 150 Cash 150
31.12.2016 31.12.2016
Depreciation 4 X
Accumulated Depreciation 4
(150 x 60% - 10/12)
31.12.2016
Investment Property 9
X FV Gain
9
FV [150 X 1.06] = 159
C.A = 150
FV Gain = 9

Page 22 of 23

276
31.12.2017 X
Depreciation 4
Accumulated Depreciation 4
31.12.2017
Investment Property 9.54
X FV Gain 9.54
FV [ 159 x 1.06 ] = 168.54
C.A = 159.00
9.54
31.12.2018
Depreciation 4 X
Accumulated Depreciation 4
31.12.2018
Investment Property 10.11
X FV Gain 10.11
FV [168.54 x 1.06] = 178.65
C.A = 168.54
10.11
31.12.2019
Depreciation 4 X
Accumulated Depreciation 4
31.12.2019
Investment Property 10.72
X FV gain 10.72
FV [ 178.65 x 1.06 ] = 189.369
C.A = 178.65
10.72

Effect on Profits :

2016 2017 2018 2019


Reversal of Dep 4 Cr. 4 Cr. 4 Cr. 4 Cr.
Recording of gain 9 Cr. 9.54 Cr. 10.11 Cr. 10.72 Cr.
Net effect on Profit 13 Cr. 13.54 Cr. 14.11 Cr. 14.72 Cr.

Page 23 of 23

277
IAS 40
Why separate classification is necessary? [IAS 40: 7]
Investment property is held to earn rentals or for capital appreciation or both. Therefore, an investment property
generates cash flows largely independently of the other assets held by an entity. This distinguishes investment
property from owner‑ occupied property.
The production or supply of goods or services (or the use of property for administrative purposes) generates
cash flows that are attributable not only to property, but also to other assets used in the production or supply
process. IAS 16 applies to owned owner‑ occupied property.

Impact of ancillary services [IAS 40: 11 to 14]


In some cases, an entity provides ancillary services to the occupants of a property it holds.

If the Services are Classification: Investment Property


Insignificant to the Example: The owner of an office building provides security and maintenance
arrangement as a whole

A GLANCE
services to the tenants who occupy the building.

SPOTLIGHT
If the Services are Classification: Not Investment Property
Significant to the Example: An entity owns and manages a hotel and services provided to guests
arrangement as a whole are significant to the arrangement as a whole.

AT
It may be difficult to determine whether ancillary services are significant to the arrangement as a whole and
judgement is needed to determine whether a property qualifies as investment property. Therefore, an entity is
required to develop (and disclose) criteria for investment property classification so that it can exercise that
judgement consistently.
IAS 40 requires all entities to measure the fair value of investment property, for the purpose of either
measurement (if the entity uses the fair value model) or disclosure (if it uses the cost model). An entity is

NOTES
encouraged, but not required, to measure the fair value of investment property on the basis of a valuation by an

SPOTLIGHT
independent valuer who holds a recognised and relevant professional qualification and has recent experience in
the location and category of the investment property being valued.

STICKY
Disposal [IAS 40: 66 & 69]
An investment property shall be derecognised:
• on disposal; or
• when the investment property is permanently withdrawn from use and no future economic benefits are
expected from its disposal.
Gains or losses arising from the retirement or disposal of investment property shall be determined as the
difference between the net disposal proceeds and the carrying amount of the asset and shall be recognised in
SPOTLIGHT

profit or loss in the period of the retirement or disposal.


Inability to measure fair value reliably [IAS 40: 53 & 55]

There is a rebuttable presumption that an entity can reliably measure the fair value of investment property on a
continuing basis. However, in exceptional cases following guidance is applicable:

Investment property If an entity determines that the fair value of investment property under
under construction construction is not reliably measurable but expects the fair value of the property to
be reliably measurable when construction is complete, it shall measure that
investment property under construction at cost until either its fair value becomes
reliably measurable, or construction is completed (whichever is earlier).
STICKY NOTES

278
Investment property If an entity determines that the fair value of an investment property (other than an
(not under investment property under construction) is not reliably measurable on continuing
construction) basis (may be because at a remote location), the entity shall measure that
investment property using the cost model in IAS 16.
The residual value of such investment property shall be assumed to be zero.
The entity shall continue to apply IAS 16 until disposal of such investment property.

Previously measured If an entity has previously measured an investment property at fair value, it shall
at Fair value continue to measure the property at fair value until disposal (or transfer to owner-
occupied or Inventory) even if comparable market transactions become less
frequent or market prices become less readily available.

Example:
Entity A has investment property carried at its fair value of Rs. 1,000,000 on 1 January 2019 with
remaining useful life of 10 years. Entity A uses fair value model under IAS 40.
On 30 June 2019, it was decided to use the building for administration rather than keeping it for
investment potential. At this date the fair value was Rs. 1,200,000.
Entity A uses cost model under IAS 16. On 31 December 2019 (year-end), the value of property
has increased to Rs. 1,300,000.
Required: Journal entries for the year ended 31 December 2019.

⯈ Answer:
Journal entries

Debit Credit
Date Particulars
Rs. Rs.
30 Jun 2019 Investment property 200,000
Investment income/gain (PL) 200,000
30 Jun 2019 Property, plant and equipment 1,200,000
Investment property 1,200,000
31 Dec 2019 Depreciation [Rs. 1,200,000 / 9.5 x 6/12] 63,158
AT A GLANCE

Accumulated depreciation 63,158

Here, Entity A applies cost model. In case Entity A had policy of applying revaluation model, the
gain of Rs. 163,158 (i.e., Rs. 1,300,000 – (1,200,000 – 63,158) would be recognised in other
comprehensive income.

⯈ Example:
Entity B has investment property carried at its fair value of Rs. 1,000,000 on 1 January 2019 with
remaining useful life of 10 years. Entity B uses fair value model under IAS 40.
On 30 June 2019, board of directors decided to develop the property and use it for sale of plots.
At this date the fair value was Rs. 1,200,000. On 31 December 2019 (year-end), the value of
SPOTLIGHT

property has increased to Rs. 1,300,000.


Required: Journal entries for the year ended 31 December 2019.

279
⯈ Answer:
Journal entries

Debit Credit
Date Particulars
Rs. Rs.
30 Jun 2019 Investment property 200,000
Investment income/gain (PL) 200,000
STICKY NOTES

30 Jun 2019 Inventories 1,200,000


Investment property 1,200,000

No adjustment required at year-end unless NRV of inventory is lower as compared to its cost of
Rs. 1,200,000.
⯈ Example:
Entity C has property being used as warehouse carried at Rs. 1,000,000 on 1 January 2019 with
remaining useful life of 10 years. Entity C uses cost model under IAS 16 for its properties.
On 30 June 2019, property was vacated, and management decided to keep it for investment
potential. At this date the fair value was Rs. 1,200,000. Entity C uses fair value model under IAS
40.
On 31 December 2019 (year-end), the value of property has increased to Rs. 1,300,000. Transfer
from revaluation surplus to retained earnings is made at the time of disposal only.
Required: Journal entries for the year ended 31 December 2019.

⯈ Answer:
Journal entries

Debit Credit
Date Particulars
Rs. Rs.
30 Jun 2019 Depreciation [Rs. 1,000,000 / 10 x 6/12] 50,000
Accumulated depreciation (PPE) 50,000
30 Jun 2019 PPE [Rs. 1,200,000 – (1,000,000 – 50,000)] 250,000
Gain on revaluation (OCI) 250,000
30 Jun 2019 Investment property 1,200,000
Property, plant and equipment 1,200,00
31 Dec 2019 Investment property 100,000
Investment income/gain (PL) 100,000

280
AT A GLAN
⯈ Example:
Entity D has commercial shop held for resale in its ordinary course of property business carried
at Rs. 1,000,000 on 1 January 2019.
On 30 June 2019, it was given on rent to a local business rather than keeping it for resale. At this
date the fair value was Rs. 1,200,000. On 31 December 2019 (year-end), the value of property
has increased to Rs. 1,300,000. Entity D uses fair value model under IAS 40.

SPOTLIGHT
Required: Journal entries for the year ended 31 December 2019.

⯈ Answer:
Journal entries

Debit Credit
Date Particulars
Rs. Rs.
30 Jun 2019 Investment property 1,200,000
Inventory 1,000,000

STICKY NOTES
Fair value gain [Profit or loss] 200,000
31 Dec 2019 Investment property 100,000
Investment income / gain (PL) 100,000

Note: The rent income shall also be recognised in profit or loss.

Completion of self-constructed investment property [IAS 40: 65]


When an entity completes the construction or development of a self‑ constructed investment
property that will be carried at fair value, any difference between:
• the fair value of the property at that date; and
• its previous carrying amount
shall be recognised in profit or loss.

281
Test Revaluation Date: -- / -- / --

QUESTION
The following information pertains to the property, plant and equipment of Orchid Limited (OL), a listed company:
Description Date of Cost Rs. In Original useful Depreciation Subsequent
purchase millions life method measurement
model
Buildings 01.01.2015 600 30 years Straight line Revaluation
Plant 01.01.2015 475 25 years Straight line Cost

Buildings
The revalued amount of buildings as determined by Shabbir Associates, an independent valuer, on 31.12.2015 and
31.12.2017 was Rs. 700 million and Rs. 463 million respectively.
On 30.06.2017 a building having original cost of Rs. 66 million was sold to Baqir limited for Rs. 85 million. It was last
revalued at 87 million. OL incurred a cost of Rs. 2 million on disposal.
OL transfers maximum possible amount from revaluation surplus to retained earnings on an annual basis.
Plant
During 2017, OL has decided to change the depreciation method for plant from straight line to reducing balance
method. The new depreciation rate would be 10%.

Required:
In accordance with International Financial Reporting Standards, prepare a note on ‘Property plant & equipment’
(including comparative figures) for inclusion in OL’s financial statements for the year ended 31 December 2017. (18)

Page 1 of 55

282
A.
Orchid Limited
Notes to financial statements
For the year ended 31-12-2017

Property, plant and equipment


2017
Building Plant Total
-------- Rs. In million ---------
Gross carrying amount:
Opening balances 700 475 1,175
Addition … … …
Disposal (87) … (87)
Elimination (42.28) … (42.28)
Revaluation:
Revaluation surplus (90.12) … (90.12)
Revaluation loss (17.6) … (17.6)
Closing Balance 463 475 938
Accumulated depreciation
Opening balance 24.14 38 62.14
Depreciation 22.64 43.7 66.14
Disposal (4.5) … (4.5)
Elimination (42.28) … (42.28)
Closing … 81.7 81.7
Carrying amount 463 393.3 856.3

Property, plant and equipment


2016
Building Plant Total
-------- Rs. In million ---------
Gross carrying amount:
Opening balance 700 475 1,175
Addition/Deletion … … …
Closing balance 700 475 1,175
Accumulated depreciation
Opening balance … 19 19
For the year 24.14 19 43.14
Closing balance 24.14 38 62.14
Carrying amount 675.86 437 1,112.86
Disclosures:
Measurement base Revaluation model Cost model
Useful life 30 years 10%
Method of depreciation Straight line Reducing balance
The last revaluation was performed by Shabbir Associates, an independent firm of valour’s on 31-12-2017.
Carrying Amount of building if carried at cost model:
2017 2016
Cost 534 600
Accumulated depreciation (53.4) (40)
Carrying Amount 480.6 560

Page 2 of 55

283
Working for the above figures
Building A/c
1-1-2015 Cash 600
c/d 600
b/d 600
c/d 600
b/d 600 30-6-2017 Disposal 66
c/d 534

Accumulated Depreciation A/c


31-12-2015 Depreciation 600/30 20
c/d 20
b/d 20
c/d 40 31-12-15 Depreciation600/30 20
30-6-2017 Disposal[66/30x2.5] 5.5
b/d 40
c/d 53.4 Depreciation[600- 18.9
66)/30+(66/30x6/12)]

Ledgers according to question:


Building
1-1-2015 b/d ----
1-1-2015 Cash 600 31-12-2015 Acc Depreciation 20
31-12-2015 Revaluation Surplus(w-1) 120 c/d 700

1-12-2016 b/d 700 c/d 700


b/d 700 31-06-2017 Disposal 87
31-12-2017 Acc. Depreciation 42.28
R.surplus 90.12
R.loss(W-2) 17.6
31-12-2017 c/d 463

Accumulated Depreciation
1-1-2015 b/d ----
31-12-2015 Building 20 31-12-2015 Depreciation (600/30) 20
c/d -
b/d -
c/d 24.14 Depreciation(700/29) 24.14
30-06-2017 Disposal(87/29X1.5) 4.5 b/d 24.14
Building 42.28 31-12-2017 Depreciation (1.5+21.14) 22.64
c/d -

Revaluation Surplus
1-1-2015 b/d ----
31-12-2015 Building 120
c/d 120

Retained earnings 4.14 b/d 120


[120/29]
c/d 115.86
30-06-2017 Retained earnings 0.4 b/d 115.86
30-06-2017 Retained earnings 22
31-12-2017 Retained earnings 3.34
Page 3 of 55

284
31-12-2017 Building 90.12
c/d -

Workings :

W-1) 31.12.2015
WDV [ 600 – 20] = 580
FV = 700
Rev. Surplus = 120

W-2) 30.6.2017 (Rev. Surplus related to disposal)


WDV [ 66 – (66/30)] = 63.8
FV = 87.0
Rev. Surplus = 23.2

W-3) 31.12.2017
WDV
FV [ 700 – 87 ] = 613
Accumulated Depreciation = 42.28
[ 613 / 29 x 2 ] 570.72

FV = 463.0
Rev. Loss 107.72

Journal Enteries : (for the year ended 31-12-2017)


Dr. Cr.
30-06-2017
Depreciation 1.5
Accumulated Depreciation 1.5
[ 87 / 29 x 6/12 ]

Revaluation Surplus 0.4


Retained Earnings 0.4
[ 23.2(w-2) / 29 x 6/12 ]

Cash ( 85 – 2 ) 83
Accumulated Depreciation 4.5
( 87 / 29 x 1.5 years )
Building 87
Gain ( P.L ) 0.5

Revaluation Surplus 22
Retained Earnings 22
[ 23.2 – (23.2 / 29 x 1.5 ] = 22

31-12-2017
Depreciation 21.14
Accumulated Depreciation 21.14
[ 700 – 87 ] / 29 = 21.14

Page 4 of 55

285
31-12-2017
Rev.Surplus 3.34
Retained Earnings 3.34
[ 120 – 23.2 ] / 29 = 3.34

31-12-2017
Accumulated Depreciation 42.28
Building 42.28

31-12-2017 90.12
Revaluation Surplus 17.60
Revaluation Loss ( P.L ) 107.72
Building ( W-3 )

Workings for Plant:


Plant
1-1-2015
Cash 475
c/d 475
1-1-2016
b/d 475
c/d 475
1-1-2017
b/d 475
c/d 475

Accumulated depreciation
1-1-2015
b/d -

31-12-2015
Depreciation 19
(475 /25)
c/d 19

b/d 19

31-12-2016
Depreciation 19
c/d 38
b/d 38

31-12-2017
Depreciation 43.7
c/d 81.7

*[475 – 38] x 10% = 43.7

Page 5 of 55

286
Test revaluation:

Q.1 The following information pertains to Gava Limited (GL):

Equipment

▪ Date of acquisition 1 January 2015 1 July 2015


▪ Cost Rs. 500 million Rs. 360 million
▪ Estimated useful life 10 years 12 years
▪ Residual value Rs. 60 million Nil
▪ Depreciation method Straight line method Straight line method

▪ Fair value Rs. 526 million Rs. 280 million


▪ Residual value Rs. 78 million Nil

▪ Fair value Rs. 310 million Rs. 230 million


▪ Residual value Rs. 64 million Nil

Additional information:
(i) GL uses revaluation model for subsequent measurement and accounts for
revaluation on net replacement value method.
(ii) There is no change in useful life of plant. The remaining useful life of equipment
was estimated as 14 years and 10 years after revaluation on 31.12.2016 and
31.12.2018 respectively.
(iii) GL transfers maximum possible amount from the revaluation surplus to retained
earnings on an annual basis.
(iv) GL’s financial year ends on 31 December.

Required:
(a) Calculate depreciation on each asset for 2015 to 2018. (08)
(b) Prepare entries to record revaluation in 2018. (Entries to record depreciation expense,
incremental depreciation and elimination of accumulated depreciation are not required.
Further, entries prior to 2018 are also not required.) (08)

Page 6 of 55

287
Ans.1 Guava Limited

a)
Rs. in Million
Plant
Cost [1.1.2015] 500
Depreciation 2015 - [500 – 60 / 10] (44)
Carrying Amount [31.12.2015] 456
Depreciation 2016 - [500 – 60 / 10] (44)
Carrying Amount [31.12.2016] 412
Revaluation Surplus (bal.) 114
Fair Value [31.12.2016] 526
Depreciation 2017 - [526 – 78 / 8] (56)
Carrying Amount [31.12.2017] 470
Depreciation 2018 - [526 – 78 / 8] (56)
Carrying Amount [31.12.2018] 414

Rs. in Millions
Equipment
Cost [1.7.2015] 360
Depreciation 2015 - [360 / 12 x 6/12] (15)
Carrying Amount [31.12.2015] 345
Depreciation 2016 - [360 / 12] (30)
Carrying Amount [31.12.2016] 315
Revaluation Loss (P.L) (bal.) (35)
Fair Value [31.12.2016] 280
Depreciation 2017 - [280 / 14] (20)
Carrying Amount [31.12.2017] 260
Depreciation 2018 - [280 / 14] (20)
Carrying Amount [31.12.2018] 240

b) Accounting enteries to record revaluation:


31.12.2018 Dr. Cr.
Revaluation Surplus 85.5
[114 – (114/8 x 2)]
Revaluation Loss (P.L) (bal.) 18.5
Plant (W-1)
104
31.12.2018
Revaluation Loss (P.L) 10
Equipment (W-1) 10

Page 7 of 55

288
Workings:

W-1) 31.12.2018

Plant:
Carrying Amount [From (a)] 414
Fair Value [Given] 310
Revaluation Loss 104

Equipment:
Carrying Amount [From (a)] 240
Fair Value [Given] 230
Revaluation Loss 10

289
Page 8 of 55
Test Statement of changes in Equity Date: -- / -- / --

Q.1 The following information pertains to Wednesday Limited (WL) for the year ended
30 une 2019:
(i) Shareholders' equity as at 1 July 2018:

Rs. In million
Share capital (Rs. 100 each) 200
Share premium 85
Retained earnings 124
Revaluation surplus 65

(ii) On 30 November 2018, WL issued 30% right shares at a premium of Rs. 120 per share.
(ii) Cash dividend and bonus shares for the last two years:
Final dividend *Interim dividend
For the year ended
Cash Bonus Cash Bonus
30 June 2018 18% - 20% -
30 June 2019 - 25% - 10%

*Declared with half yearly accounts


(iii) Profit for the year amounted to Rs. 95 million.

(iv) Revaluation surplus arising during the year amounted to Rs. 35 million whereas transfer of
incremental depreciation for the year was Rs. 9 million.

Required:
Prepare WL’s Statement of Changes in Equity for the year ended 30 June 2019. (07)
(Column for total and comparative figures are not required)

290
Page 9 of 55
A.1 Wednesday Limited
Statement of changes in equity
For the year ended 30 June 2019
Share Share Retained Revaluation
capital premium earnings surplus
--------------- Rs. in million ---------------
Balance as at 1 July 2018 (As given) 200 85 124 65
Final cash dividend @ 18% for 2018
(200×18%) (36)
Right issue @ 30% 60 72
(200/100 x 30%) (0.6 x 100) (0.6×120)
Interim bonus dividend @ 10% for 2019
(260×10%) 26 (26)
Profit for the year 95
Increase in revaluation surplus 35
Transfer of incremental depreciation 9 (9)
Balance as at 30 June 2019 286 157 166 91

291
Page 10 of 55
Test SOCE Date: -- / -- / --
Q. 1
The following information pertains to draft financial statements of Pak Ocean Limited (POL) for the year ended 31
December 2014.
(i)
2014 2013
Rs. In million
Profit after tax 90 47
Incremental depreciation on revaluation of property, plant and
equipment (means transfer of Revaluation surplus) 1.5 2.3

(ii) Installation of an assembly plant was completed in December 2013 at a cost of Rs. 60 million and it was ready for
use on 1 February 2014. However, depreciation for the year ended 31 December 2014 amounting to Rs. 4.5
million was worked out from the date of production i.e. 1 April 2014.
(iii) Shareholders’ equity as at 1 January 2013 was as follows:

Rs. In million
Share Capital (Rs. 100 Each) 200
Retained Earning 45
Revaluation surplus 100

On 30 November 2014, POL issued 25% right shares to its ordinary shareholders at Rs. 120 per share.
(iv) Cash dividend and bonuses declared/paid during the last three years:

Final *Interim
For the year ended
Cash Bonus Cash Bonus
31 December 2012 - 15% 16% -
31 December 2013 18% - 20% -
31 December 2014 - 25% - 10%

*Declared with half yearly financial statements


Required:
Prepare Statement of Changes in Equity for the year ended 31 December 2014 in accordance with the requirements of
the Companies Act 2017 and International Financial Reporting Standards. (15)
(Ignore taxation)

292
Page 11 of 55
Ans. 1

PAK OCEAN LIMITED


Statement of changes in Equity
For the year ended 31-12-2014
“Rs In Millions”
Share Share Retained Revaluation Total
Capital Premium Earnings Surplus
Balance as on 1-1-2013 200 - 45 100 345
Bonus Share-2012 (200 X 15%) 30 (30) -
Interim Cash dividend-2013 (230 X 20%) (46) (46)
Profit after tax 47 47
Transfer of Surplus 2.3 (2.3) -
Balance as on 1-1-2014 230 - 18.3 97.7 346
Final cash dividend-2013 (230 X 18%) (41.4) (41.4)
Interim bonus-2014(230 X 10%) 23 (23) -
Issue of right shares- (W-2) 63.25 12.65 - 75.90
Profit after tax (W 1) 89 89
Transfer of surplus 1.5 (1.5)
Balance as on 31-12-2014 316.25 12.65 44.4 96.2 469.5

W-1 Current period error


2014
Profit After tax-Given 90
Effect of depreciation* (1)
Profit after tax [adjusted] 89

4.5/9 X 2 = 1 (effect of depreciation of Feb & March 2014).

W-2) [230+23] /100 = 2.53 M shares X 25 % = 0.6325 M shares


0.6325M X 100 = 63.25
0.6325 M X 20 = 12.65

293
Page 12 of 55
Test statement of changes in equity:
Q.1 Following information have been extracted from the financial statements
of Fakhr Limited (FL) for the year ended 31 December 2019:

(i) 2019 2018 2017


Draft Audited Audited
--------- Rs. in million ---------
Net profit 84 98 72

(ii) Share capital and reserves as at 1 January:

2018 2017
----- Rs. in million -----
Share capital (Rs. 10 each) 300 300
Retained earnings 348 276

(iii) On 1 March 2018, FL declared a final cash dividend of 10% for the year ended 31 December
2017. On 1 November 2018, FL issued 40% right shares to its ordinary shareholders at Rs. 24 per
share. On 1 August 2019, an interim bonus of 15% was declared.

Following information need to be incorporated in the draft financial statements of FL:

On 1.1.2019 company changed the method of valuation of subsequent measurement of property plant and
equipment from cost model to revaluation model.

Useful life Cost Acc. Depreciation Revalued Amount


Factory Rs in millions
Buildings 30 years 300 75 280

On 31.8.2019, one of the factory building having written down value of Rs 20.5875 millions and revalued amount
of Rs.25.65 millions on 01.01.2019 was sold for Rs.22 millions.

The cost of disposal was 0.5 millions.

On 31.12.2019 remaining factory buildings were revalued at Rs.205 millions.

There is no change in the estimated useful life and company transfers maximum amount of revaluation surplus
to retained earnings on annual basis.

Required:
Prepare FL’s statement of changes in equity (including comparative figures) for the year ended 31 December
2019. (‘Total’ column is not required) (18)

294
Page 13 of 55
Ans.1 Fakhr Limited
Statement of Changes In Equity For the
year ended 31 December 2019

Ordinary share Share Revaluation Retained


capital premium surplus earnings
---------- Rs. in million ----------
Balance as at 31 December 2017 (As 300.00 - 348.00
given)
Final cash dividend @ 10% for 2017 - - - (30.00)
( 300 x 10% )
Right issue @ 40% 120.00 168.00 - -
( 300 / 10 x ( 300 / 10 x
40% x 10) 40% x 14)
– Net profit for 2018 - - - 98
Balance as at 31 December 2018 420.00 168.00 - 416
Interim bonus dividend @ 15% for 63.00 - - (63.00)
2019 (420 x 15%)
Revaluation Surplus (workings) 55
– Net profit (workings) - - - 68.55
- Transfer of Surplus (workings) - - (7.2825) 7.2825
-Revaluation Loss (workings) - - (38.05) -
Balance as at 31 December 2019 483.00 168.00 9.6675 428.8325

Workings

Date Particular Debit Credit


1.1.2019 Accumulated depreciation Factory 75
Building 75
1.1.2019 Factory building 55
Revaluation Surplus 55
C.A [300 – 75 ] = 225
F.V = 280
Revaluation Surplus = 55
31.08.2019 Depreciation 0.76
Accumulated depreciation 0.76
[25.65 / 22.5(Working) x 8/12]
31.8.2019 Revaluation Surplus 0.15
Retained Earnings 0.15
[25.65 – 20.5875 = 5.0625 / 22.5 x 8/12]
31.8.2019 Cash (22 – 0.5) 21.5
Accumulated depreciation 0.76
Loss(P.L) 3.39
Factory building 25.65
31.8.2019 Revaluation surplus 4.9125

Retained earnings [5.0625 – 0.15] = 4.9125 4.9125


31.12.2019 Depreciation 11.30
Accumulated Depreciation(280-25.65) /22.5 11.30
31.12.2019 Revaluation Surplus 2.22
Retained Earnings (55 – 5.0625) / 22.5 = 2.22 2.22
31.12.2019 Accumulated Depreciation 11.30

295
Page 14 of 55
Factory buildings 11.30
31.12.2019 Revaluation Surplus (W-2) 38.05
Factory Buildings 38.05

Workings:
(W-1)
Remaining useful life on 01.01.2019 after revaluation:
300/30 = 10 per anum
75/10 = 7.5 years
30 – 7.5 = 22.5 remaining years
Carrying Amount (280 – 25.65) – 11.30 = 243.05
Fair Value = 205.00
Revaluation Loss 38.05

Remaining revaluation surplus before revaluation on 31.12.2019 [55 – 5.0625 – 2.22] = 47.7175
After this revaluation on 31.12.2019, fair value of Rs.205 million will be depreciated over remaining useful life of 21.5
years from next year onwards. In addition remaining surplus of Rs.9.6675 (47.7175 – 38.05) will be transferred to
retained earnings over remaining useful life of 21.5 years from next year onwards.
Effect on net profit for the year ended 31.12.2019
Given profit 84 Cr.
Depreciation 0.76 Dr.
Loss 3.39 Dr.
Depreciation 11.30 Dr.

Adjusted profit 68.55 Cr.

(W-3)

Revaluation Surplus
31.08.2019 Retained Earnings 0.15 01.01.2019 Factory Building 55
31.08.2019 Retained Earnings 4.9125
31.12.2019 Retained Earnings 2.22

31.12.2019 Factory Building 38.05


c/d 9.6675

Total transfer of revaluation surplus during the year: 0.15 + 4.9125 + 2.22 = 7.2825

296
Page 15 of 55
Q.1 Octagon Pakistan Limited (OPL) in is process of preparation of financial statements for the year ended 31
December 2018. During the year, OPL completed the construction of its head office building. Relevant
details in this respect are as follows:

(i) Payments related to the construction of the building were as follows:


Description Date of payment Rs. In million
Construction permit fee 1-Jan-18 30
Advance to contractor 1-Jan-18 80
1st bill of contractor 1-Feb-18 250
2nd bill of contractor 1-May-18 360
3rd bill of contractor 1-Sept-18 170
Last bill of contractor (Received on 25-12-2018) 1-Jan-19 150
1,040

(ii) The project was financed through the following sources:


Excess cash of Rs. 200 million available with OPL on 1 January 2018 in a saving account at 10% per annum.
Loan of Rs. 350 million at the rate of 16% per annum obtained on 1 February 2018. The principal is payable in 5
equal annual instalments alongwith interest, from 1 February 2019. The surplus funds available from the loan
were invested in a saving account at 10% per annum.
Withdrawals from running finance facilities arranged on 1 Jan 2018. The facilities were also used to finance other
needs of OPL. Details of these facilities are as follows:

Name of bank Balance as on Limit Average Finance


31 December 2018 Balance cost
------------------------ Rs. in million ------------------------
Bank X 130 150 140 15.4
Bank Y 340 600 390 44.2

Payment of 3rd bill of contractor includes Rs. 10 million which was charged by the contractor for damages
sustained at the site on account of unexpected rains.

The work was stopped from 16 to 31 May 2018 to meet mandatory technical requirements. Further, on 16
September 2018, the building control authority stopped the construction work as it raised objections on the
design of the building. The matter was resolved on 30 September 2018.

Construction of the building was completed on 31 October 2018. However, it was inaugurated on 1 December
2018. The building has an estimated useful life of 30 years.

Required:
Prepare relevant extracts from OPL's statement of profit or loss for the year ended 31 December 2018 and statement of
financial position as on that date.
(Notes to the financial statements are not required. Borrowing costs are to be calculated on the basis of number of
months) (17

297
Page 16 of 55
Test Solution
Ans.1 Octagon Limited
Statement of financial position as on 31 December 2018
Rs. in million
Non-current assets:
Building (W-1) 1,069.95

Non-current liabilities:
Term loan 350–70 280.00

Current Liabilities:
Loan 350÷5 70.00
Running finance 130+340 470.00
Interest payable on loan 350×16%×11/12 51.33
Payable to contractor 150.00

Statement of profit or loss for the year ended 31 December 2018

Investment income (200 – 30 – 80 = 0.75


90 x 10% x 1/12)
Interest expense: (60.26)
▪ Specific loan (350×16%×2.5/12) 11.67
(2.5 = N, D & From 16 – 9 to
30-9)
▪ General borrowings (15.4+44.2 – 11.01) 48.59
Depreciation expense (5.98)
Other expenses (damages due to rain) (10.00)

W-1: Building Rs. in million


Permit fee 30.00
Payment to contractor 80+250+360+170+150–10 1,000.00
Borrowing cost capitalized (W-2) 45.93
1,075.93
Depreciation 1,075.93÷30×2/12 (5.98)
1,069.95

W-2: Borrowing cost capitalized Rs. in million


Specific loan (1 Feb. to 31 Oct. excl. 15* days) 350×16%×8.5/12 39.67
*15= (16-9 to 30-9)
Investment income (W-3) 190×10%×3/12 (4.75)
34.92
General borrowings (W-4) 11.01
45.93
W- 3
Detail of Payments ;

Funds from
Owned Specific fund General
Date Particulars Amount
1-1-2018 Permit fee 30 30
1-1-2018 Advance 80 80
1-2-2018 Ist bill 250 90 160
1-5-2018 2nd bill 360 190 170
1-9-2018 3rd bill 160 160
(170 – 10*)
*Abnormal wastage to be expensed

298
W-4: Capitalization rate
Calculation of capitalization rate
59.6/530 x 100 = 11.25% p.a
General borrowings :
170 x 11.25% x 5.5/12 = 8.76
160 x 11.25 % x 1.5/12 =2.25
11.01

299
Test Cash Flow Date: -- / -- / --
Question 1
The following information has been extracted from the draft financial statements of Alpha Limited for the year ended 31 December
2015.

2015 2014 2015 2014


Assets Equity & Liabilities
Rs. In million Rs. In million
Property, plant & equipment 223 193 Share capital (Rs. 10 each) 180 150
Intangible assets 68 23 Share premium 15 -
Trade receivables 45 33 Retained earnings 114 53
Advances and prepayments 84 70 Long term loan 40 -
Inventories 60 46 Trade payables 57 66
Short-term investments 12 9 Accrued expenses 60 70
Cash at bank 8 7 Tax payable 34 42

500 381 500 381

Following relevant information is available:


(i) On 1 September 2015, the company purchased new machinery costing Rs. 65 million.
(ii) A portion of building costing Rs. 20 million which was purchased on 1 July 2013 was sold for Rs. 20 million on 30 June 2015.The
useful life of building was 20 years.
(iii) Trade receivables written off during the year amounted to Rs. 5 million. It is the policy of the company to maintain the provision
for doubtful debts at 5% of trade receivables.
(iv) Advances and prepayments include advance tax of Rs. 8 million (2014: Rs. 6 million).
(v) Long term loan was obtained on 1 August 2015. Interest on loan @ 13% is payable on 31st July each year. Interest payable for 5
months has been accrued.
(vi) Tax expense for the year was Rs. 17 million. (2014: Rs. 8 million).
(vii) Right shares were issued on 1 December 2015 at Rs. 15 per share in the ratio of 1 right share for every 5 shares held.
Required:
Prepare statement of cash flows for the year ended 31 December 2015 in accordance with the requirements of International Financial
Reporting Standards using the indirect method. (15)

300
Answer 1
Alpha Limited
Statement of Cash Flow
For the year ended 31-12-2015
Cash Flow From Operating Activities:

Profit before tax (61 + 17) 78


Interest expense 2.17
Depreciation 17.00
Gain on disposal (2.00)
Bad debts (5+0.63) 5.63
Profit before working capital changes 100.8

Working Capital Changes:

Trade debtors (17.63)


Inventories (60 – 46) (14.00)
Advances and Prepayments (12.00)
Trade payables (57 – 66) (9.00)
Accrued expense (12.17) (64.8)
Cash for generated from operations 36
Tax paid (27)
Net cash from operating Activities 9.00
Cash flow From Investing Activities:
Purchase of machinery (65)
Receipt from disposal 20
Acquisition of intangibles (45)
Purchase of investments (12 – 9) (3)
Net Cash From Investing Activities (93)

Cash Flow From Financing Activities:


Proceeds from issue of right shares (150/10 = 15 × 1/5 × 15) 45
Proceeds from long term loan 40
85
Net Cash Flow 1
Cash and cash equivalents at the beginning of the period 7
Cash and cash equivalents at the end of the period 8

301
Workings:
PPE – WDV

b/d 193 Disposal (20 – 2) 18


Cash 65 Depreciation (bal.) 17
c/d 223

20 ÷ 20 × 6/12 = [0.5 + 1 + 0.5] = 2


Intangibles

b/d 23
Cash 45
c/d 68

Receivables

b/d 34.74 Bad debts 5


17.63
c/d 47.37

33/95% = 34.74
45/95% = 47.37
Provision

b/d 1.74
Bad debts 0.63
c/d 2.37

Advances

b/d (70 – 6) 64
12
c/d (84 – 8) 76

Advances tax + tax Payable

b/d 6 b/d 42
Cash 27 Expenditure 17
c/d 34 c/d 8

Share Capital

b/d 150
Cash 30
c/d 180

Share Premium

b/d --
Cash 15
c/d 15

Retained Earnings

b/d 53
PAT 61
c/d 114 302
Loan

b/d --
40
c/d 40

Accrued Expenses

12.17 b/d 70

c/d (60 – 2.17*) 57.83

*(40 × 13% × 5/12 = 2.17)


Interest Payable

b/d --
Expenses 2.17
c/d 2.17

Disposal A/c

PPE 18 Cash 20

Gain 2

303
Test Cash Flow (Direct Method) Date: -- / -- / --
Q.Following are the extracts from the financial statements of Emporium Mall (EM) for the year ended 30 June 2018:
Statement of Financial Position as on 30 June 2018
201 2017 2018 201
Equity & Liabilities 8 Asset 7
Rs. in s Rs. in
000 000
Share capital (Rs.10 700,000 300,000 Property, plant and 740,188 228,000
each) equipment
Share premium 70,000 - Stock in trade 116,000 90,000
Retained earnings 58,868 43,000 Trade receivables 72,812 112,000
Revaluation Surplus 20,000 - Cash 95,868 96,000
Debentures 36,000 52,000 Service income receivable 4,000 8,000
Interest payable 2,000 5,000 Prepaid Expenses 2,000 4,000
Trade payables 84,000 86,000
Accrued Expense 40,000 36,000
Provision for taxation 20,000 16,000
1,030,868 538,000 1,030,868 538,000

Statement of profit or loss for the year ended 30 June 2018


Rs. in ‘000’
Sales 546,000
Cost of sales (375,000)
Gross profit 171,000
Operating expenses (93,532)
Other income 22,400
Profit before interest and tax 99,868
Interest expense (4,000)
Profit before tax 95,868
Tax expense (30,000)
Profit after tax 65,868

Additional information:
i. 85% of sales were made on credit.
ii. EM maintains a provision for doubtful receivables at 3%. During the year, trade receivables of Rs.14 million
were written off.
iii. Depreciation expense for the year was Rs.45 million. 55% of the depreciation was charged to cost of sales.
iv. On January 1, 2018 EM made a bonus issue of one share for every six held and 35 million right shares
were issued on March 31, 2018 at Rs.12 per share.
v. Other income comprises of:
• Gain of Rs.6 million on disposal of vehicles for Rs.24 million;
• Service income of Rs 16.4 million
Required:
Prepare EM’s statement of cash flows for the year ended 30 June 2018 using direct method. (15)

304
A.
Emporium Mall
Statement of Cash
Flow
For the year ended 30-6-2018
Cash Flow from Operating Activities:
Cash receipt from customer [546,000 x 15% + 572,400
490,500]
Payment to suppliers (W) (378,250)
Payment for expenses (W) (54,494)
Receipt from services income (W) 20,400
Cash generated from operations 160,056
Interest paid (7,000)
Tax paid (26,000)
Net cash from operation activities 127,056
Cash Flow from Investing Activities:
Receipt from disposal 24,000
Purchase of PPE (555,188)
Net cash from investing activities (531,188)
Cash Flow from Financing Activities:
Receipt from issue of shares (35,000 x 12) 420,000
Repayment of debentures (16,000)
Net cash from financing activities 404,000
Net cash flow (132)
Cash and cash equivalents at the beginning of the 96,000
period
Cash and cash equivalents at the end of the period 95,868
Workings:
Cash and Cash Equivalents:
2018 2017
Cash 95,868 96,000

PPE
b/d 228,000 Disposal 18,000
R.S 20,000 Depreciation 45,000
Cash 555,188
c/d 740,188

Stock
b/d 90,000 Cost of sales 350,250
[375,000 – (45,000 × 55%)]
Purchases(bal) 376,250
c/d 116,000

Provision A/C
b/d (112,000/97 x 3) 3,464
Expense 1,212
c/d (72,812/97 x 3)
2,252

Trade Receivables
b/d 112,000/97 x 100] 115,464 Bad debts 14,000
Sales (546,000 x 85%) 464,100 Cash 490,500
c/d [72,812/97 x 100] 75,064

305
Share Capital
b/d 300,000

Dividend [Bonus Share(300,000/10 = 50,000


30,000 x 1/6 x 10)]
Cash (35,000 x 10) 350,000
c/d 700,000

Share Premium
b/d
Cash (35,000 x 2) 70,000
c/d 70,000

R/Earnings
Dividend 50,000 b/d 43,000
PAT 65,868
c/d 58,868

R.Surplus
b/d --
PPE 20,000
c/d 20,000

Debentures
b/d 52,000
Cash 16,000
c/d 36,000

Prepaid and Accrued Expences


b/d 4,000 b/d 36,000
Cash 54,494 Expense (93,532 -14,000 – 45,000 x 45 % + 1,212) 60,494
c/d 40,000 c/d 2,000

Service Income Receivable


b/d 8,000
Income 16,400 Cash 20,400
c/d 4000

Interest Payable
b/d 5,000
Cash 7,000 Exp 4,000
c/d 2,000

Trade Payable

b/d 86,000
Cash 378,250 Purchases 376,250
c/d 84,000

306
Disposal A/c

PPE 18,000 Cash 24,000


Gain 6,000
Provision for Tax
b/d 16,000
Cash 26,000 Expence 30,000
c/d 20,000

Test IAS-23 Date: -- / -- / --

Q. On 1 July 2015, Minhas Manufacturers Limited (MML) commenced construction of its new factory
building and completed the work on 30 June 2016. Following information is available in this respect:

(i) The agreed price of the contract was Rs. 100 million which was financed through the following
sources:
▪ Bank loan of Rs. 80 million was obtained on 1 June 2015. The loan carries a mark-
up of 10.9436% per annum and is repayable in five semi-annual instalments of Rs.
16 million each along with interest commencing from 30 November 2015.
▪ The remaining amount was financed through cash withdrawals from MML’s
existing running finance facilities. Details of these facilities are as follows:

Name of Running finance


Balance as on Average Mark-up %
Bank
Limit 30 June 2016 Balance
-------Rs. in million -------
Bank A 50 33 40 11%
Bank B 40 5 30 13%

(ii) Due to delay in supply of construction material, construction work was suspended from 1
November 2015 to 30 November 2015.
(iii) The following payments were made to the contractor net of 5% retention money which is
refundable one year after completion of the building:
Date of payment 1-06-2015 1-08-2015 1-12-2015 1-04-2016 1-08-2016
Net amount paid
(Rs. in million) 9.5 28.5 28.5 19.0 9.5

(iv) Surplus funds, if any, were invested @ 7% per annum.

Required:
Show how the above information would be disclosed in MML’s statement of financial position as on
30 June 2016 in accordance with the International Financial Reporting Standards.
(17)
(September 2016)

307 Page 1 of 6
A.
Minhas Manufacturing Limited
Statement of Financial Position (Extracts)
As on 30-6-2016
2016
Rs. In
Million
Non-Current Assets
Factory Building (Not a CWIP now asset is now complete) (W-1) 106.943

Non-current Liabilities
Bank Loan 16.00

Current Liabilities
Current Portion of loan (16 x 2) 32.00
Short term running finance (33 + 5) 38.00
Bills payable (Payable to contractor) 9.5*
Retention money payable (9.5 + 28.5 +28.5 + 19 + 9.5) ÷ 95% x 5% 5.0*
Interest payable (48 x 10.9436% x 1/12) 0.44

Entry of last bills: (As the work is completed on 30-6-2016, therefore last bill should be accrued in this year).
CWIP-Building (9.5 ÷ 95%) 10
Retention money 0.5
Payable to contractor 9.5
W-1) Cost of building:
*Progress bills (10 + 30 + 30 + 20 + 9.5/95 x 100) 100
Interest on specific borrowings (w-4) 6.86
Investment income on surplus funds (w-5) (1.15)
Interest on general borrowings (w-6) 1.233
106.943
Not required; just for extra information:
As the work is commenced on 1-7-2015, therefore interest related to June 2015 and related investment income
should not be considered; i.e.
Interest incurred:
80 x 10.9436% x 1/12 = 0.73
Interest income:
(80 – 9.5) 70.5 x 7% x 1/12 = 0.41
The above two amounts should be recognized as interest expense and interest income respectively during the year
ended
30-6-2015.

W-2) Computation of capitalization rate:


Average bal. Interest rate Interest (A x B)
Bank A 40 11% 4.4
Bank B 30 13% 3.9
70 8.3
8.3/70 x 100 = 11.857%

308 Page 2 of 6
W-3) Detail of payments:

Total Used from Used from


Date Particular Net Amount
Amount specific general
1-6-2015 1st Bill 10 9.5 9.5 -
1-7-2015 Current year
1-8-2015 2nd Bill 30 28.5 28.5 -
Repayment of loan (16 + 80 x 20.377 20.377
30-11-2015 .109436 x 6/12) 20.377 -
1-12-2015 3rd Bill 30 28.5 21.623 6.877
1-4-2016 4th Bill 20 19 - 19
Repayment of loan (16 + 64 x 19.50 19.50
31-5-2016 .109436 x 6/12) - 19.50

W-4) Specific Borrowings:


80 x 10.9436% x 4/12 (J, A, S, O) = 2.92
64 x 10.9436% x 6/12 (Dec to May) = 3.5
48 x 10.9436% x 1/12 (June) = 0.44
6.86

W-5) Investment Income on surplus funds:


70.5 (80-9.5) x 7% x 1/12 ( July) = 0.41
42(80-9.5-28.5) x 7% x 3/12 (A, S, O) = 0.74
21.623 (80-9.5-28.5-20.377) x 7% x 0/12 = -
1.15

W-6) Interest on General loan:


6.877 x 11.857% x 7/12 (Dec to June) = 0.48
19 x 11.857% x 3/12 (April to June) = 0.563
19.5 x 11.857% x 1/12 (June) = 0.19
1.233

W-7) Interest expense of November and investment income of November is ignored in above workings because of
suspension period

309 Page 3 of 6
Test Cash Flow(Indirect Method) Date: -- / -- / --

Q. 1 MARVEL ENGINEERING LIMITED


Following are the extracts from the draft financial statements of Marvel Engineering Limited (MEL), a listed
company, for the year ended 30 June 2015:

Statement of Financial Position


Rs. in million
2015 2014 2015 2014
Non current assets Share capital and
reserves
Property, plant and equipment 633 410 Share capital (Rs. 10 each) 494 440
Long term investments 130 100 Share premium 8 -
Retained earnings 133 110
763 510 635 550
Current assets
Stock-in-trade 97 68 Non current liabilities
Trade debts 133 57 Long term loans 400 181
Other Assets 100 120
Cash at bank 31 39

361 284

Current liabilities

Trade and other 73 56


payables
Tax payable 12 5
Dividend payable 4 2
89 63
1,124 794 1,124 794

Statement of profit or loss


2015
Rs. in million
Revenue 654
Cost of sales (458)
Gross profit 196
Operating expenses (68)
Financial charges (75)
Other income 35

(108)
Profit before tax 88
Income tax expense (21)
Profit after tax 67

Additional information:
i.

310 Page 4 of 6
Rs. m
Dividend income 30
Gain on sale of vehicles (carrying value of Rs. 5 million) 2
Gain on sale of investments (carrying value of Rs. 10 million) 3
35
Required
Prepare the statement of cash flows for Marvel Engineering Limited for the year ended 30 June 2015.
(15)
Q.2 APOLLO INDUSTRY LIMITED
Following are the relevant extracts from the financial statements of Apollo Industry Limited, a listed company, for
the year ended December 31, 2015.
Statement of financial position as at December 31, 2015
2015 2014
Rs. 000 Rs. 000
Issued, subscribed and paid up capital 25,000 20,000
Unappropriated profit 20,900 22,000
Surplus on revaluation of property, plant & equipment 7,000 8,000
52,900 50,000
Non-current liabilities
Long term Loan 1,990 1,190
Trade and other payables 4,200 6,250
59,090 57,440
Non current Assets:
Property, plant and equipment 40,500 35,500
Intangible assets 1,100 1,140
41,600 36,640
Long term deposits and prepayments 400 650
42,000 37,290
Current Assets
Tax refundable / Advance tax 950 800
Other current assets 15,700 12,125
Cash and bank balances 440 7,225
17,090 20,150
59,090 57,440

Statement of comprehensive Income for the year ended December 31, 2015
2015
Rs. 000
Sales 146,700
Cost of sales (127,500)
Gross profit 19,200

Operating expenses (15,000)


Financial charges (500)
Other income 2,800
(12,700)
Profit before tax 6,500
Tax for the year (5,600)
Profit after tax 900

Other relevant information is as under:


i.

311 Page 5 of 6
iv.

312 Page 6 of 6
A1
Marvel Engineering Limited
Cash Flow Statement
For the year ended 30 June 2015
2015
Cash flows from operating activities Rs. in million
Profit before taxation 88.00
Adjustment for non-cash charges and other items:
Depreciation 61.00
Financial charges 75.00
Bad debts(6+4) 10.00
Gain on sale of fixed assets (2.00)
Gain on sale of investments (3.00)
Dividend income (30.00)
Working capital changes
Stock-in-trade (97 - 68) (29.00)
Trade debts (86.00)
Other assets (100 - 120) 20.00
Trade and other payables (73 – 7) – (56 - 3) 13.00
Cash generated from operations 117.00
Financial charges paid (3 + 75 - 7) (71.00)
Income tax paid (14.00)
Net cash generated from operating activities 32.00

Cash flows from investing activities


Purchase of property, plant and depreciation (289.00)
Proceeds from sale of property, plant and equipment (5+2) 7.00
Proceeds from sale of investments (10+3) 13.00
Purchase of long term investments (130-100+10) (40.00)
Dividend received 30.00
Net cash used in investing activities (279.00)

Cash flows from financing activities


Insurance of ordinary shares (8+32) 40.00
Proceeds from long term loan (400 - 181) 219.00
Payment of dividend (2 + (440 x 5%) - 4) (20.00)
Net cash from financing activities 239.00
Net decrease in cash and cash equivalents (8.00)
Cash and cash equivalent at the beginning of the year 39.00
Cash and cash equivalent at the end of the year 31.00

WORKINGS (All amount in million rupees)

Provision for Trade


bad debts Debtors
Closing balance (133 ÷ 0.93)- 133 7.00 (133 ÷ 0.95) 140.00
Add: Bad debts written off 6.00 6.00
Less: Opening balance (57÷ 0.95)- 57 (3.00) (57 ÷ 0.95) (60.00)
10.00 86.00

PPE-WDV
b/d 410
Depreciation 61
Cash 289 Disposal 5
c/d 633
313 Page 7 of 6
Share Capital
b/d 440
Dividend 22
c/d 494 Cash 32

Retained Earnings
Dividend (Cash) 22 b/d 110 Dividend Payable
(440*5%) b/d 2
Dividend (Bonus) 22 Profit after tax 67 Cash 20 Dividend 22
(440*5%) c/d 4
c/d 133

Share Premium Interest Payable


b/d - b/d 3
c/d 8 Cash 8 Cash 71 Expense 75
c/d 7
Long term investment
Income Tax Payable b/d 100 disposal 10
Cash 14 b/d 5 Cash 40 c/d 130
c/d 12 Expense 21

Debtors Provision
b/d 60 Bad 6 b/d 3
debts Bad 4
Increase 86 c/d 140 debts
c/d 7
A.2
Apollo Industry Limited
Statement of cash flows
For the year ended December 31, 2015 Rs. in ‘000
Cash used in operating activities
Profit before taxation 6,500
Adjustment for: (non-cash items / separately disclosed items)
Depreciation for the year (7,000-90) 6,910
Amortization for the year 90
Profit on sale of fixed assets (2,800-1,000) (1,800)
Interest income (1,000)
Interest Expense 500
Operating profit before working capital changes 11,200
Trade and other payable(6,250-4,200) (2,050)
Other current assets(12,125-15,700) 3,575)
Cash generated from operations 5,575
Payment for taxation (950 + 5,600 - 800) (5,750)
Interest paid (500)
Net cash from operating activities (675)
Cash used in investing activities
Purchase of PPE (13,110)
Proceeds from sale of PPE (1,200 + 1,800) 3,000
Acquisition of intangible assets (50)
Interest received 1,000
Receipt from Long term deposits (400-650) 250
(9,410)
Cash used in financing activities
Issue of ordinary share capital (25,000-2,000-20,000) 3,000
Dividend paid (1,000)
Long term loan(1,990-1,190) 800
2,800
Net decrease in cash and cash equivalents (6,785)
Opening balance: cash and cash equivalents 7,225
Closing balance: cash and cash equivalents 440
314 Page 8 of 6
Property, plant & equipment
b/d 35,500 Disposal 1,200
Depreciation 6,910
(7,000 - 90)
Cash 13,110
c/d 40,500

Intangible Assets
b/d 1,140 Amortization(bal) 90
Cash 50 c/d 1,100

Tax
b/d 800 Expense 5,600
Paid (Bal) 5,750 c/d 950
Retained Earning
Dividend(S.C) 2,000* b/d 22,000
Dividend(bal) 1,000 PAT 900
c/d 20,900 R.S 1,000

*(20,000*10%)
Disposal A/C
PPE 1,200 Cash(bal) 3,000
Gain 1,800

Share Capital
b/d 20,000
Dividend 2,000
Cash (Bal) 3,000
c/d 25,000
Revaluation surplus
R.E(bal) 1,000 b/d 8,000
c/d 7,000

315 Page 9 of 6
Test Ratios Date: -- / -- / --

Q.1 Progressive Steel Limited (PSL) commenced business in 2015. The following comparative data pertains to
the year ended 30 June 2017:

PSL Industry
Description
2017 2016 2017
Gross profit margin 13% 13% 16%
Net profit margin 8% 7% 10%
Return on shareholders’ equity 22% 18% 25%
Current ratio 1.2 1.6 1.5
Debt to equity ratio 40:60 30:70 50:50
Cash operating cycle in days 119 135 118

Required:
For each ratio/data give possible reasons for variation from comparative and industry data. (12)

316 Page 10 of
6
Ans.

Ans.1 Reasons for fluctuation with


Ratios Reason for fluctuation with Industry
previous year
Gross profit In line with previous year. No variation. Lower than industry
margin The company is in initial phase and may
have kept the selling prices lower than
the
industry to gain the market share.
The company may not have been able to
purchase raw material at prices which is
available to its competitors.
The company may not have been able to
obtain economies of scale in its
production which may have been
obtained
by its competitors.
Possibility of higher production costs.
Lower than industry however, the
Net profit Higher than previous year: difference
margin Tight control over operating costs. is mainly attributed to lower gross profit
Increase in other income. margin.
Decrease in fixed cost per unit due to
increase in sale.
Return on Higher than previous year: Lower than industry
Reduction in tax
shareholder's rates. Lower gross profit and net profit margins.
equity Reduction in interest rates. Lower leverage.
Decrease in equity might be due to Higher net assets resulting in higher
buyback of shares. equity.
Distribution of profits from previous year
which resulted in decrease in equity.
(means more dividend)
Increase in profit due to increase in
. revenue or decrease in expenses
Current ratio Lower than previous year: Lower than industry
The company might have obtained Since the debt equity ratio is lower than
running finance facility to fund it's the industry, company might have
operations in the current year. obtained running finance or might have
availed extended credit terms from
Long term loan payments might have
suppliers.
become due in the next 12 month, which
decreases the current
ratio.

Decrease in current assets due to better


inventor
y management/ reduction in
credit period of debtors.
Debt to equity Higher than previous year Lower than industry
ratio Decrease in reserves due to dividend pay- Being a new entrant the company may be
out. in the phase of expansion thereby raising
Further debt obtained during the period. debt accordingly.
. Less loans than an average company in this
Decrease in equity might be due to industry.
buyback of shares.
Cash operating Lower than previous year In line with industry.
cycle Increase in creditor days might be due
to increase in credit
317 Page 11 of
6
period.
Decrease in stock turnover days which
might be
due to greater stock turnover or better
inventory management
(means stock turnover
days are lower)

By giving lower credit days to debtors.

Test Ratios Date: -- / -- / --


Q.1
An investor wants to analyze the performance of Zee Limited for which he has collected the following information for
the year ended 30 June 2015 and 2014:

2015 2014
Rs. In million
Profit after interest and tax 100.00 75.00
Interest expense at 12% per annum on a long-term loan acquired on 1 January
2014 (9.60) (4.80)
Tax expense (44.00) (43.00)
Interim bonus issue 12% 10%
Final cash dividend (2013: 30%) 20% 25%

The break-up of shareholders’ equity as at 1 July 2013 was as under:

Rs. In million
Share capital (Rs. 10 each) 200
Share premium 20
Retained earnings 40
260

Required:
Compute Return on Capital employed and Return on shareholders’ Equity for the year ended 30 June 2015. (07)

318 Page 12 of
6
A.1
(i) Return on capital employed

Rs. In million
Average capital employed as at 30 June 2015:
Average equity (W-1) (275 + 320) ÷ 2 297.50
Average long term loan (9.6 ÷ 12%)=80+80/2 80.00
A 377.50
Profit before interest and tax (100+9.6+44) B 153.60
Return on capital employed B÷A 40.69%

(ii) Return on shareholders’ equity

Rs. In million
Average shareholders’ equity C 297.50
Profit after interest and tax D 100.00
Return on shareholders’ equity D÷C 33.61%

(W-1) Average equity

Share Share Retained Total


Capital Premium Earnings Equity
Balance as at 1 July 2013 200.00 20.00 40.00 260.00
Final cash dividend for 2013 at 30% (200 x 30%) - - (60.00) (60.00)
10% Interim bonus issue 20.00 - (20.00) -
Profit after tax for the y.e. 30 Jun. 2014 - - 75.00 75.00
Balance as at 30 June 2014 220.00 20.00 35.00 275.00
Final cash dividend for 2014 at 25% (220 × 25%) - - (55.00) (55.00)
12% Interim bonus issue (220 × 12%) 26.40 - (26.40) -
Profit after tax for the y.e. 30 Jun. 2015 - - 100.00 100.00
Balance as at 30 June 2015 246.40 20.00 53.60 320.00

319 Page 13 of
6
Spring 2020
Q.2 You are working as Finance Manager in Broad Peak Limited (BPL). Faraz has recently joined BPL as an
internee for three months. You have asked him to develop an understanding of the statement of cash
flows. After going through few statements, he has raised the following queries:

(i) Depreciation is not a cash flow but was still appearing as an addition in the statement of cash
flows.
(ii) In the statement of cash flows of a competitor, interest paid was shown as a financing activity but
BPL showed it in operating activities.
(iii) BPL purchased inventories throughout the year but total purchases of inventory were not shown in
the statement. However, only decrease in inventory was added.
(iv) Cash and bank balance in the statement of financial position was not in agreement with the
opening and closing balances at the end of statement of cash flows.

(v) In statement of cash flow, tax paid is deducted in operating activities but tax expense is not added
back I n profit before tax.

(vi) The working capital changes does not include increase/decrease in all current assets and current
liabilities.

(vii) What is the treatment of transactions between the components of cash and cash equivalents e.g.
cash deposited /withdrawn from bank.

Required:
Briefly answer the queries raised by Faraz. (08)

Q.3 Briefly describe the measurement bases that may be used to measure the value of assets in the financial
statements. (06)

320 Page 14 of
6
Q.4 Select the most appropriate answer(s) from the options available for each of the following Multiple
Choice Questions (MCQs).

(i) Which of the following companies is most likely to face cash flow problems?
(a) A loss making government organisation
(b) A company which has recently sold part of its operations so as to concentrate on its core
areas

(c) A reasonably profitable and long established company with no expansion plans
(d) A profitable retailer about to embark on ambitious expansion plans (01)

(ii) A plant has a carrying amount of Rs. 1,500,000 as at 31 December 2019. Its fair value is Rs. 900,000
and costs of disposal are estimated at Rs. 50,000. A new plant would cost Rs. 2,500,000. Cash flows
from the plant for the next four years are estimated at Rs. 350,000 per annum. Applicable discount
rate is 10%.

What is the approximate impairment loss on the plant to be recognised in the financial statements
as at 31 December 2019?

(a) Rs. 650,000 (b) Rs. 390,000


(c) Rs. 1,000,000 (d) Nil (02)

(iii) A debit balance on the retained earnings account indicates that the company has:

(a) made more dividend payments than the profit earned


(b) redeemed some of its share capital
(c) accumulated losses
(d) issued bonus shares (01)

(iv) The correct accounting treatment of initial operating losses incurred during the commercial
production due to under-utilization of the plant would be to:

(a) capitalise as a directly attributable cost


(b) defer and charge to profit or loss account when profit is earned from the plant
(c) charge directly to retained earnings since these are not considered to be normal operating
losses
(d) charge to profit or loss account (01)
(v) In measuring value in use, the discount rate used for discounting the cash flows
should be the:

(a) pre-tax rate that reflects the market assessment of time value of money and
risks specific to the asset
(b) pre-tax rate that reflects the market assessment of time value of money and
risks specific to the entity
(c) post-tax rate that reflects the entity’s assessment of time value of money and
risks specific to the asset
(d) pre-tax rate that reflects the entity’s assessment of time value of money and risks
specific to the asset (01)

(vi) Which of the following is NOT considered as an item of property, plant and
equipment?

321
(a) A standby generator expected to be used for seven years
(b) A plot of land held for resale
(c) A bus for pick and drop of staff members
(d) A generator for rental to others (01)

(vii) Capitalization of borrowing costs should be suspended:


(a) when substantially all the activities necessary to prepare a qualifying asset for
its intended use or sale are complete
(b) during a temporary delay which is a necessary part of the process of getting
an asset ready for its intended use or sale
(c) during extended periods in which active development of a qualifying asset is
interrupted
(d) all of the above (01)

Q.6
Following are the summarised financial statements of Shispare Limited (SL) and its competitor Trivor
Limited (TL) for the year ended 31 December 2019:

Statement of
financial position
SL TL SL TL
Assets Equity & liabilities
Rs. in million Rs. in million
Fixed assets 5,400 7,800 Capital and reserves 8,400 9,450
Current assets: Long-term loan 1,900 4,600
Inventory 4,800 7,100 Current liabilities:
Debtors 2,700 3,200 Creditors 2,900 4,500
Cash 1,200 800 Accrued expenses 900 350
8,700 11,100 3,800 4,850
14,100 18,900 14,100 18,900

Statement of profit
or loss
SL TL
--- Rs. in million ---
Sales 16,700 35,400
Cost of goods sold (11,400) (27,800)
Gross profit 5,300 7,600
Operating expenses (3,500) (4,900)
Finance cost (250) (600)
Net profit 1,550 2,100

Required:
Compute relevant ratios for SL and TL to assess which company seems to:
(i) Give more incentives to its customers to pay on time
(ii) Avail extended credit terms from its suppliers
(iii) Be more efficient in the use of capital

322
(iv) Keep lower selling prices to gain the market share
(v) Have better liquidity position
(vi) Have higher ability to convert its assets into profit
(vii) Control operating expenses more efficiently
(viii) Have higher ability to raise bank loan in future (16)

Q.8
Following information pertains to non-current assets of Distaghil Limited (DL):

(i) DL purchased specialised vehicles for Rs. 370 million on 1 July 2017. The vehicles have an
estimated useful life of 10 years with residual value of Rs. 30 million.

The revalued amounts of the vehicle as at 31 December 2018 and 2019 were determined at Rs.
302 million and Rs. 290 million respectively. There was no change in useful life or residual value.

(ii) DL setup a manufacturing plant in a remote area at a cost of Rs. 280 million. The plant had a useful
life of 8 years. The plant was purchased on 1 January 2018 and was available for use on 1 April
2018. The commercial production started on 1 June 2018.

On 1 July 2018, DL received a government grant of Rs. 120 million towards the cost of the plant.
The sanction letter states that if DL ceases to use the plant in the remote area before 31 December
2021, DL would be required to repay the grant in full.

(iii) A warehouse was given on rent on 1 January 2018. Previously, the warehouse was in use of DL.

On 1 January 2018, carrying value and remaining useful life of the warehouse was Rs. 80 million
and 16 years respectively. Fair value of the warehouse on various dates are as follows:

Rs. in million
01 January 2018 104
31 December 2018 96
31 December 2019 115

Other information:
• DL uses cost model for subsequent measurement of property, plant and equipment
except for specialised vehicles for which revaluation model is used.
• DL transfers the maximum possible amount from the revaluation surplus to retained
earnings on an annual basis.
• Government grant is recorded as deferred income and a part of it is transferred to income
each year.
• Investment property is carried at fair value model.

Required:
Prepare relevant extracts from DL’s statement of profit or loss and other comprehensive income for the
year ended 31 December 2019 and statement of financial position as on that date. (Show comparative
figures) (20)

323
A.2
(i) A statement of cash flows begins with net profit which is arrived after deducting depreciation
expense. So to convert the net profit into net cash flow the deduction of depreciation is
reversed (i.e. added).
(ii) As per IAS 7, interest paid can be shown as either cash flow from financing activities or cash
flow from operating activities. Both classifications are correct as long as they are consistently
applied by an entity.
(iii) A statement of cash flows begins with net profit which is arrived after deducting cost of sales.
So to convert the effect of cost of goods sold into outflow for purchases of inventory, change in
inventory is adjusted i.e. increase is deducted and decrease is added.
(iv) Statement of financial position shows cash and bank balances while the statement of cash
flows ends with cash and cash equivalents which may differ from cash and bank balances due to
existence of bank overdraft and short term investments.
(v) As the operating activities are started with profit before tax and tax expense is therefore not
deducted so there is no reason to add back the tax expense.
(vi) The purpose of working capital changes is to convert the items of income and expenses within
the profit before tax into receipts and payments indirectly. Therefore,
• Debtors and advance from customers should be considered to convert sale into
receipts from customers.
• Inventories should be considered to convert cost of sales in to purchase.
• Creditors and advance to suppliers should be considered to convert purchases into
payment to suppliers.
• Similarly expenses payable and prepaid expenses should be considered to convert
expenses into payment for expenses.
Other items e.g. interest payable, tax payable and dividend payable should not be considered because total
payment is to be presented.
Cash and cash equivalents should be considered at the end of the statement of cash flow. Short term
investments should either be classified as investing activities or cash and cash equivalents.
Interest and dividend receivable should not be considered because their total recipt is to be presented in
investing activities.
(vii) These transactions cancel out each other therefore not presented anywhere in the statement
of cash flows.
A.3
(i) Historical cost
The historical cost of an asset, when it is acquired or created is the value of the cost incurred in acquiring or
creating the asset, comprising the consideration paid to acquire or create the asset plus transaction cost.
Current value
Current value measures provide monetary information about assets using information updated to reflect
conditions at the measurement date.
Current value measurement bases include:
– Fair value
– Value in use for assets
– Current cost

324
Fair value
Fair value is the price that would be received to sell an asset in an orderly transaction between market
participants at the measurement date. Fair value reflects the perspective of market participants.
Value in use
Value in use is the present value of the cash flows or other economic benefit that an entity expects to derive from
the use of an asset and from its ultimate disposal. Value in use reflect entity specific assumptions rather than
assumptions by market participants.
Current cost
The current cost of an asset is the cost of an equivalent asset at the measurement date comprising the
consideration that would be paid at the measurement date plus the transaction cost that would be incurred at
that date.
Current cost, like historical cost is an entry value; while fair value is an exit value. However, unlike historical cost,
current cost reflects conditions at the measurement date

A.4

The most appropriated answer(s) for Multiple Choice Questions (MCQs)

(i) (d) A profitable retailer about to embark on ambitious expansion plans.


(ii) (b) Rs. 390,000 (W – 1)
(iii) (c) accumulated losses
(iv) (d) charge to profit or loss account
(vi) (a) Pre-tax rate that reflects the market assessment of time value of money and risks specific to
the asset
(vii) (b) A plot of land held for resale
(viii) (c) during extended periods in which active development of a qualifying asset is interrupted.

Workings:

W- 1:
Ans:
Option (b)

31-12-2019
Carrying Amount 1,500,000

Recoverable Amount:
Higher of:
Value in use:

1− (1 + 0.1)−4
[350,000 x ] = 1,109,453
0.1

FV less CTS:
[900,000 – 50,000] = 850,000
Higher of: 1,109,453
Impairment loss (1,500,000 – 1,109,453) = 390,548 (which approximates to 390,000)

325
W – 2:
Ans:
Option (b)
As total in all cost types is different, therefore non of the cost is 100% fixed cost.
Cost per unit in case of A is constant at all levels i.e.
(i) 1,500 ÷ 100 = 15/unit; and
(ii) 2,250 ÷ 150 = 15/unit
Therefore, it is 100% variable cost similarly cost per unit in case of C is constant at all levels, i.e.
(i) 2,000 ÷ 100 = 20/unit; and
(ii) 3,000 ÷ 150 = 20/unit
There it is 100% variable as well
However, in case of B and D cost per unit is also not constant, i.e.

B:
(i) 1,800 ÷ 100 = 18/unit; and
(ii) 2,400 ÷ 150 = 16/unit

Similarly;

D:
(i) 3,000 ÷ 100 = 30/unit; and
(ii) 4,200 ÷ 150 = 28/unit
Therefore, cost type B and D are semi-variable cost.

A.6
Relevant ratios SL TL
(i) Debtors collection period 59.01 days 32.99 days
Debtors 2,700 3,200
= ×365 = ×365 = ×365
Sales 16,700 35,400
TL is giving more incentives to its customers to
pay on time.
(ii) Creditors payment period 92.85 days 59.08 days
Creditors
= ×365 2,900 4,500
Purchases/Cost of sales = ×365 = ×365
11,400 27,800
SL avail extended credits terms
(iii) Return on capital 17.48% 19.22%
Profit before interest 1,550+250 2,100+600
= ×100 = ×100 = ×100
Capital employed 8,400+1,900 9,450+4,600
TL is more efficient in the use of capital
(iv) Gross profit margin 31.74% 21.47%
Gross profit 5,300 7,600
= ×100 = ×100
= ×100 16,700 35,400
Sales
TL is deliberately keeping selling prices lower to
gain the market share.
(v) Current ratio 2.29 2.29
Current assets 8,700 11,100
= = =
Current liabilities 3,800 4,850
Quick ratio 1.03 0.82

326
Current assets-inventory 8,700 − 4,800 11,100 − 7,100
= = =
Current liabilities 3,800 4,850
SL has better liquidity position
(vi) Return on assets 12.77% 14.29%
Profit before interest 1,550+250 2,100+600
= ×100 = ×100 = ×100
Total assets 14,100 18,900
TL has higher ability to convert its assets into profit

(vii) Operating expenses %age 20.95% 13.84%


Operating expenses 3,500 4,900
= ×100 = ×100
= ×100 16,700 35,400
Sales
TL is efficiently controlling the operating expenses.

(viii) Gearing ratio 18% 33%


Debt 1,900 4,600
= = ×100 = ×100
Debt + Equity 8,400 + 1,900 9,450 + 4,600

SL is going to raise a bank loan relatively


easily in future

A.8
Distaghil Limited
Extracts from statement of financial position as on 31 December 2019
2019 2018
---- Rs. in million ----
Non-current assets:
Property, plant and equipment:
▪ Vehicles 290.00 302.00
▪ Plant (253.75 – 35) / (280 – 26.25) 218.75 253.75

Investment property 115.00 96.00

Share capital and reserves:


Revaluation surplus:
▪ Vehicles 5.00 -
▪ Warehouse (22.5 – 1.5) / (24 – 1.5) 21.00 22.50

Non-current liabilities:
Deferred government grant (108.75 – 15-15) / (120 – 11.25-15) 78.75 93.75
Current liabilities
Deferred government grant 15.00 15.00

327
Extracts from statement of profit or loss and other comprehensive income For
the year ended 31 December 2019
2019 2018
------ Rs. in million ------
Profit or loss:
Depreciation:
▪ Vehicles (32.00) (34.00)
▪ Plant (35.00) (26.25)

Revaluation loss – vehicles - (17.00)


Revaluation gain – vehicles 15.00
Grant income – Plant 15.00 11.25
Change in fair value of investment property - warehouse
(2019: 115–96), (2018: 96–104) 19.00 (8.00)

Other comprehensive income:


Revaluation surplus:
▪ Vehicles 5.00
▪ Warehouse (104 – 80) - 24.00
Workings:
i) Vehicles:
1-7-2017
Vehicle 370
Cash 370

31-12-2017 370 - 30/10 = 34 x 6/12 = 17


Dep 17
Acc. dep 17

31-12-2017
Dep 34
Acc. Dep 34

31-12-2018
Acc. dep 51
Vehicle 51

R. Loss 17
Vehicle 17

WDV (370 – 51) = 319


FV = 302
R. Loss 17

31-12-2019
Dep 32
Acc. Dep 32
302 – 30/8.5 = 32

328
31-12-2019
WDV (302 – 32) = 270
FV = 290
R.S 20

Acc. dep 32
Vehicle 32
Vehicle 20
R.S (OCI) 5
Reversal of loss 15

Reversal of loss 17Cr.


Extra dep (34 - 32) 2Dr.
Net reversal of loss 15 Cr.
ii) Plant:
1-1-2018
Plant 280
Cash 280

1-7-2018
Cash 120
Deferred Income 120

31-12-2018
Dep 26.25
Acc. dep 26.25
(280/8 x 9/12) =

31-12-2018
Deferred income 11.25
Other income 11.25
(120 ÷ 8 x 9/12) =

31-12-2019
Dep 35
Acc. Dep 35
(280/8)

31-12-2019
Deferred Income 15
Other Income 15
(120 ÷ 8)

iii) Investment Property:


1-1-2018
Warehouse 24
R. Surplus (OCI) 24

C.A = 80
24
FV = 104

329
1-1-2018
Investment Property(WH) 104
Warehouse (PPE) 104

31-12-2018
FV Loss (P.l) 8
Investment Property 8

C.A = 104
8 Loss
FV = 96

31-12-208
R. Surplus 1.5
R. Earnings 1.5 (24 ÷ 16)

31-12-2019
C.A = 96
19 Gain
FV = 115
Investment property 19
FV gain (P.L) 19

31-12-2019
R. Surplus 1.5
R. Earnings 1.5

330
Test of Cash flow:
Q.1 The statement of Financial Position of Liaquat Industries as at 31 December 2016 is as follows:
Equity and 2016 2015 Assets 2016 2015
Liabilities
….Rupees… ….Rupees…
Owners’s capital 13,938,060 13,665,280 Freehold land 4,778,400 6,600,000
Long term loan 1,000,000 1,000,000 Building-WDV 5,057,600 4,171,200
Short term loan 1,331,200 1,531,200 Vehicle-WDV 600,000 800,000
Accounts payable 417,120 694,320 Equipment-WDV 1,643,100 2,112,000
Accrued interest 105,600 63,360 Capital work in progress 1,478,400 1,821,600
Long term deposits 580,800 448,800
Inventory 685,608 320,628
Accounts receivable 1,273,272 595,452
Cash 694,800 84,480
16,791,980 16,954,160 16,791,980 16,954,160
The following information has been extracted from income statement:
….Rupees…
Depreciation expense 932,500
Finance cost 141,872
Gain on disposal of fixed assets (net) 98,960
Net profit before Tax 1,525,948
Additional information:
i. Details of gain on sale of fixed asset are as follows:

….Rupees…
Gain on sale of freehold land 168,960
Loss on disposal of equipment due to fire (70,000)
98,960
The loss on disposal of equipment represents the WDV of the equipment. The amount of insurance
claim received, amounting to Rs. 30,000 was erroneously credited to accumulated depreciation.
ii. Repairs to building amounting to Rs. 50,000 were erroneously debited to building account 31 December
2016.
iii. Transfers from capital work in progress to building amounted to Rs. 1,200,000.
iv. The owner withdrew Rs. 150,000 per month.

Required:
Prepare statement of cash flows for the year ended 31 December 2016, in accordance with
IAS-7 using indirect method.

331
Answer No. 1

….Rupees…
Cash Flow Operating Activities
Profit before tax [1,525,948 + 30,000 – 50,000] 1,505,948
Adjustments for:
Depreciation [363,600 + 200,000 + 368,900] 932,500
Gain on disposal (168,960)
Loss on disposal [70,000 – 30,000] 40,000
Interest expense 141,872
Profit before working capital changes 2,451,360
Working Capital Changes:
Accounts payable (417,120 – 685,608) (277,200)
Inventory (320,628 – 685,608) (364,980)
Accounts receivable (595,452 – 1,273,272) (677,820)
Cash generated from operations 1,131,360
Interest paid (99,632)
Net cash from Operating Activities 1,031,728
Cash Flow from Investing Activities
Receipt from disposal [1,990,560 + 30,000] 2,020,560
Payment for capital work in progress (856,800)
Payment for long term deposits (132,000)
Net cash from Investing Activities 1,031,760
Cash Flow from Financing Activities:
Capital introduced by owner 546,832
Drawings (150,000 x 12) (1,800,000)
Short term loan paid (200,000)
Net cash from Financing Activities (1,453,168)
Net Cash Flow 610,320
Opening balance of cash and cash equivalents 84,480
Closing balance of cash and cash equivalents 694,800
Workings:
Cash & Cash Equivalents
2016 2015
Cash 694,800 84,480

Capital Account
Drawings 1,800,000 b/d 13,665,280

Profit 1,525,948
Cash (bal) 546,832
c/d 13,938,060
Unadjusted bal 13,938,060
Building 50,000 Equipment 30,000
c/d (adjusted) 13,918,060

Short term Loan


Bank (bal) 200,000 b/d 1,531,200
c/d 1,331,200

332
Accrued Interest
Cash (bal) 99,632 b/d 63,360
Interest 141,872
c/d 105,600

Land
b/d 6,600,000 Disposal (bal) 1,821,600
c/d 4,778,400

Building
b/d 4,171,200 Depreciation (bal) 363,600
Capital work in progress 1,200,000
Cash 50,000 c/d 5,057,600
Unadjusted bal 5,057,600 Capital 50,000
c/d (adjusted) 5,007,600

Vehicle
b/d 800,000 Depreciation (bal) 200,000
c/d 600,000

Equipment
b/d 2,112,000 Disposal 70,000
Cash 30,000
Depreciation (bal) 368,900
c/d 1,643,100
Unadjusted bal 1,643,100
Capital 30,000 c/d (adjusted) 1,673,100

Capital work in progress


b/d 1,821,600 Capital work in progress 1,200,000
Cash (bal) 856,800
c/d 1,478,400

Long Term Deposits


b/d 448,800
Cash (bal) 132,000
c/d 580,800

Disposal-Land
Land 1,821,600 Cash (bal) 1,990,560
Gain 168,960

Disposal-Equipment
Land 70,000 Cash 30,000
Loss 40,000

333
[70,000 – 30,000]

Long term loan


b/d 2,112,000

c/d 1,643,100

Accounts payable
b/d 694,320
277,200
c/d 417,120

Inventory
b/d 320,628
364,980
c/d 685,608

Accounts receivable
b/d 595,452
677,820
c/d 1,273,272

334
Question of depreciation
Kamran Enterprises (KE) provides depreciation on plant and machines at 10% on written-down value.
Depreciation is charged from the month the asset is available for use in operations up to the month
prior to its disposal. Cost of its plant & machines and the accumulated depreciation as on 1 July 2015
were Rs. 75 million and Rs. 17 million respectively.
The following information is available in respect of its plant & machines, for the year ended 30 June
2016:
(i) On 1 October 2015, a second-hand machine was acquired from a Chinese company for Rs. 15
million. The machine was renovated and overhauled at a cost of Rs. 3 million. 25% of this
expenditure was in respect of purchase of consumables (spare parts).
(ii) On 1 November 2015, KE transferred a machine having a list price of Rs. 10 million from its stock-
in-trade to its Engineering Department. KE sells such machines at cost plus 25%.
(iii) On 1 January 2016, certain worn-out parts of a plant were replaced at a cost of Rs. 4 million. The
replaced parts neither enhanced the useful life of the plant nor its operating efficiency. The old
parts were sold for Rs. 0.75 million. The plant was purchased for Rs. 25 million on 1 January 2015.
The old parts were fully depreciated at the beginning of period.
On 1 May 2016, the plant was damaged and remained in-operative for one month. KE spent an
amount of Rs. 3 million on repairs to restore the plant in working condition.
(iv) On 1 April 2016, a machine which was purchased on 1 July 2012 for Rs. 12 million was completely
damaged and was sold for Rs. 1.2 million.
Required:
Prepare accounting entries to record the above transactions in KE’s books for the year ended 30 June
2016. (17)
Answer:
Kamran Enterprises – Adjusting entries:
1-10-2015 Machine (15 + 3 × 75%) 17,250,000
Consumable items 750,000
Bank 18,000,000
11-2015 Machine Account (10,000,000/125 × 100) 8,000,000
Stock 8,000,000
1-1-2016 Plant and Machine 4,000,000
Bank 4,000,000
1-1-2016 cash 750,000
Other Income 750,000
There is no need of cost of old parts because they are fully depreciated.
1-5-2016 Repair and Maintenance 3,000,000
Bank 3,000,000
1-4-2016 Bank 1,200,000
Accumulated depreciation (working 1) 3,908,100
Loss on disposal (bal) [P&l] 6,891,900
Machine 12,000,000
30-6-2016 Depreciation (Working) 7,608,838
Accumulated depreciation 7,608,383

Workings:
Property, Plant & Equipment
b/d 75,000,000 1-4-2016 Disposal 12,000,000
1-10-2015 Bank 17,250,000

335
1-11-2015 Stock 8,000,000
1-1-2016 Bank 4,000,000
c/d 34,250,000

Accumulated Depreciation
1-04-2016 Disposal (W-1) 3,908,100 b/d 17,000,000
30-06- Depreciation (W 2) 7,608,383
2016
c/d 20,700,283
Disposal Account
--- Bank 1,200,000
Machine 12,000,000 Acc. Dep. 3,908,100
Loss 6,891,900
(W-1)
Cost (1-7-2012) 12,000,000
× 10% (1,200,000)
30-6-2013 10,800,000
× 10% (1,080,000)
10-602014 9,720,000
× 10% (972,000)
10-6-2015 8,748,000
× 10% × 9/12 (656,100)
WDV 8,091,900
Accumulated Depreciation:
[12,000,000 – 8,091,900] = 3,908,100
(W-2) Depreciation:
Opening WDV (75,000,000 – 17,000,000) 58,000,000
Less: opening WDV of Disposal (8,748,000)
49,252,000
× 10% 4,925,200
+ 17,250,000 × 10% × 9/12 1,293,750
+ 8,000,000 × 10% × 8/12 533,333
+ 4,000,000 × 10% × 6/12 200,000
+ Depreciation on disposal (8,748,000 × 10% × 9/12) 656,100
Total for the year 7,608,383

336

You might also like